Download as pdf or txt
Download as pdf or txt
You are on page 1of 292

RBI Assistant Prelims All in One E-Book

INTRODUCTION

We are providing you RBI Assistant Prelim Online Exam Practice Work
Book-(English Medium) These year a major change has been introduced in the
Prelims phase of the examination. From this year onward, the RBI Assistant
Prelims phase will have sectional timings (20 mins for each section), whereas
the total timing of the exam remains the same (one hour).

RBI Assistant Preparation | Tips During the Exam

1. If you find any particular question difficult, make sure that you don’t waste
your time trying to solve it. Instead, you should move to the next question and if
time permits come back to this question later.

2. One thing that is crucial is that you have to clear the sectional cut-off of
each section and also the overall cut-off.

3. So, if in the Exam, you find any particular section a bit challenging, attempt
the minimum number of questions that are required to clear the sectional cut-off
of that section.

4. Sometimes, the questions can be very tricky. We advise you to read them
very carefully before answering.

5. In the Exam, avoid using any shortcut that you are not completely sure
about.

6. Avoid Guess work in the Exam

By Dream Big Institution Team

(Deepali Jadhav)

Page 1 Follow us: Official Site, Telegram, Facebook, Instagram, Instamojo


INDEX

Sr No. Title Page


No.
1 RBI Assistant Exam Pattern 3

2 Reasoning Questions 4

3 Quantitative Aptitude Questions 78

4 English Questions 211

Page 2 Follow us: Official Site, Telegram, Facebook, Instagram, Instamojo


RBI Assistant Exam pattern

The RBI Assistant Exam pattern is the same as other bank clerk exams
followed by Preliminary, Main and Language Proficiency Test. This year a new
change has been introduced i.e. the sectional timing in the exam pattern. You
can go through the Prelim & main exam pattern below:

RBI Assistant Exam Pattern for Prelims

Sr. Subject No. of Maximum Time


No. Questions Marks
1 English 30 30 20 min
Language
2 Reasoning 35 35 20 min
Ability
3 Numerical 35 35 20 min
Ability
Total 100 100 60
mins.

RBI Assistant Exam Pattern for Main

Sr. Subject No. of Maximum Total


No. Questions Marks
1 Reasoning Ability 40 40 30
minutes
2 Numerical Ability 40 40 30
minutes
3 English Language 40 40 30
minutes
4 Computer 40 40 20
Knowledge minutes
5 General 40 40 25
Awareness minutes
Total 200 200

Page 3 Follow us: Official Site, Telegram, Facebook, Instagram, Instamojo


REASONING QUESTIONS
SYLLABUS:

Reasoning Ability

 Syllogism
 Seating Arrangement -Circular, Square & Linear
 Puzzle
 Blood Relation
 Inequality
 Data Sufficiency
 Coding & Decoding
 Distance & Direction
 Ranking

Page 4 Follow us: Official Site, Telegram, Facebook, Instagram, Instamojo


Syllogism

(Directions 1 – 10): In the questions below are given some conclusions


followed by five set of statements. You have to choose the correct set of
statements that logically satisfies given conclusions. Assume the given
statements to be true even if they seem to be at variance from commonly
known facts.

1. Conclusions:
I. Some cats are dogs.
II. At least some rats are birds.
III. All rats being cats is a possibility.

Statements:
I. Some birds are dogs. Some dogs are rats. Some rats are cats.
II. All birds are rats. Some rats are cats. No dog is rat.
III. All birds are rats. All rats are dogs. No dog is cat.
IV. Some birds are rats. Some rats are dogs. No dog is cat.
V. Some birds are rats. Some rats are dogs. Some dogs are cats.
1) Only statement I
2) Only statement II
3) Only statement III
4) Only statement IV
5) Only statement V

Answer & Explanation

Answer – 5) Only statement V


Solution:

Page 5 Follow us: Official Site, Telegram, Facebook, Instagram, Instamojo


Page 6 Follow us: Official Site, Telegram, Facebook, Instagram, Instamojo
Page 7 Follow us: Official Site, Telegram, Facebook, Instagram, Instamojo
2. Conclusions:
I. All Parrots are Scissors.
II. Some Scissors are definitely not Combs.

Statements:
I. Some Parrots are Sparrow. Some Sparrows are Scissors. No Comb is
Sparrow.
II. All Scissors are Sparrow. Some Sparrows are Parrots. No Comb is

Page 8 Follow us: Official Site, Telegram, Facebook, Instagram, Instamojo


Sparrow.
III. All Parrots are Sparrow. Some Sparrows are Scissors. Some
Combs are Scissors.
IV. All Parrots are Sparrow. All Sparrows are Scissors. No Comb is
Sparrow.
V. All Parrots are Sparrow. All Sparrows are Scissors. Some Combs
are Scissors.
1) Only statement I
2) Only statement II
3) Only statement III
4) Only statement IV
5) Only statement V

Answer & Explanation

Answer – 4) Only statement IV


Solution:

Page 9 Follow us: Official Site, Telegram, Facebook, Instagram, Instamojo


Page 10 Follow us: Official Site, Telegram, Facebook, Instagram, Instamojo
3. Conclusions:
I. Some heroes are zeroes.
II. At least some top are joker.
III. All top being hero is a possibility.

Statements:
I. Some jokers are top. Some top are zero. Some zeroes are hero. All
top are bottom.
II. Some jokers are zero. Some zeroes are top. Some top are hero. No
joker is bottom.
III. All jokers are top. Some top are bottom. All heroes are bottom. No
zero is bottom.
IV. All jokers are top. All top are zero. No zero is hero. Some bottoms
are zero.

Page 11 Follow us: Official Site, Telegram, Facebook, Instagram, Instamojo


V. Some jokers are top. Some top are zero. No bottom is hero. All zero
are bottoms.
1) Only statement I
2) Only statement II
3) Only statement III
4) Only statement IV
5) Only statement V

Answer & Explanation

Answer – 1) Only statement I


Solution:

4. Conclusions:
I. All flowers being sand is a possibility.
II. No flower is a candle.
III. Some rose are flowers.

Page 12 Follow us: Official Site, Telegram, Facebook, Instagram, Instamojo


Statements:
I. At least some flowers are petals. No petal is a candle. No flower is
sand.
II. All flowers are petals. No petal is a candle. All candles are sand. All
petals are rose.
III. At least some flowers are petals. All flowers are candles. No
flower is sand.
IV. All flowers are petals. No petal is a candle. All flowers are rose.
No petal is sand.
V. Some flowers are petals. All flowers are rose. Some rose are
candles. All candles are sand.
1) Only statement I
2) Only statement II
3) Only statement III
4) Only statement IV
5) Only statement V

Answer & Explanation

Answer – 2) Only statement II


Solution:

Page 13 Follow us: Official Site, Telegram, Facebook, Instagram, Instamojo


Page 14 Follow us: Official Site, Telegram, Facebook, Instagram, Instamojo
5. Conclusions:
I. Some jugs are frogs.
II. At least some cocks are cakes.
III. All cocks being jugs is a possibility.

Statements:
I. Some cakes are cocks. Some cocks are frogs. Some frogs are jugs.
II. Some cakes are frogs. Some frogs are cocks. Some cocks are jugs.
III. All cakes are cocks. Some cocks are jugs. No frog is cock.
IV. All cakes are cocks. All cocks are frogs. No frog is jug.
V. Some cakes are cocks. Some cocks are frogs. No frog is jug.
1) Only statement I

Page 15 Follow us: Official Site, Telegram, Facebook, Instagram, Instamojo


2) Only statement II
3) Only statement III
4) Only statement IV
5) Only statement V

Answer & Explanation

Answer – 1) Only statement I


Solution:

6. Conclusions:
Some road is house is a possibility. Some child is not house.

Statements:
1) All road is child. Some child is toy. All toy is ring. No toy is house.
2) All road is child. Some toy is road. All toy is ring. No child is
house.
3) Some road is toy. All child is toy. No toy is ring. Some house is
ring.
4) All child is road. All road is toy. Some house is ring. No house is
toy.
5) None is correct

Page 16 Follow us: Official Site, Telegram, Facebook, Instagram, Instamojo


Answer & Explanation

Answer – 1) All road is child. Some child is toy. All toy is ring. No
toy is house.
Solution:

7. Conclusion– No river can be bangles. Some beads are Canal.


Statement I- Some river is beads. No bead is stone. Some stone is
Canal. Some rivers are bangles.
Statement II-Some river is beads. Some beads are Canal. All Canals
are stones. Some bangles are beads.
Statement III- All stones is Canal. No Canal is bangle. Some rivers are
bangles. Some river is beads.
Statement IV- All bangles is Canal. Some rivers are Canal. All rivers
are stones. No bead is stones.
Statement V- No bangle is river. Some rivers are stones. Some stone is
Canal. Some beads are canal.
1) Only Statement I
2) Only Statement II
3) Only Statement III

Page 17 Follow us: Official Site, Telegram, Facebook, Instagram, Instamojo


4) Only Statement IV
5) Only Statement V

Answer & Explanation

Answer – 5) Only Statement V


Solution:

Page 18 Follow us: Official Site, Telegram, Facebook, Instagram, Instamojo


Page 19 Follow us: Official Site, Telegram, Facebook, Instagram, Instamojo
8. Statements:
Some bag is Hot.
No Hot is cake.
All cakes are Milk.

Conclusions:
I. Some bag is not cakes.
II. Some Hots can be Milk.
III. Some Milk is not Hot.
1) If only conclusion II follows
2) If both conclusions II and III follow
3) If both conclusions I and III follow
4) If both conclusions I and II follow
5) If all conclusions follows

Answer & Explanation

Page 20 Follow us: Official Site, Telegram, Facebook, Instagram, Instamojo


Answer – 5) If all conclusions follows
Solution:

9. Statements:
Some colours are walls.
All brushes are walls.
All paints are colours.
Some brushes are balls.

Conclusions:
I. Some brushes are paints.
II. Some balls are definitely colours.
III. At least some walls are ball.
1) If both conclusions I and III follow
2) If only conclusion III follows

Page 21 Follow us: Official Site, Telegram, Facebook, Instagram, Instamojo


3) If both conclusions I and II follow
4) If both conclusions II and III follow
5) If all conclusions follows

Answer & Explanation

Answer – 2) If only conclusion III follows


Solution:

10. Conclusion:
Some pages are sheets. All sheets are pens.

Statements:
1) All pages are sheets. All sheets are paper. All papers are pens.
2) All pages are sheets. Some pens are pages. Some sheets are paper.
3) All sheets are paper. All papers are pens. Some pages are pens.
4) Some pages are sheets. No pen is paper. All sheets are paper.
5) None of these

Answer & Explanation

Answer – 1) All pages are sheets. All sheets are paper. All papers
are pens.

Page 22 Follow us: Official Site, Telegram, Facebook, Instagram, Instamojo


Solution:

Page 23 Follow us: Official Site, Telegram, Facebook, Instagram, Instamojo


Seating Arrangement – Circular, Square & Linear

Direction (Q.1-5): Study the following information carefully and answer


the questions given below:
Eight persons P, Q, R, S, T, V and W are playing a game of musical
chairs and facing away from the centre. Each person knows a different
language, viz Marathi, Odia, Tamil, Punjabi, Bengali, Hindi, Telugu and
English, but not necessarily in the same order.
· Neither S nor T speaks English.
· R speaks Hindi and sits third to the left of T.
· Those who speak Tamil and Punjabi are immediate neighbours of each other.
Neither the one who speaks Punjabi nor the one who speaks Tamil is an
immediate neighbour of either T or R.
· The one who speaks Punjabi sits on the immediate left of Q, who speaks
Odia.
· Those who speak Telugu and English are immediate neighbours of each other.
· P speaks Telugu and is not an immediate neighbour of the one who
speaks Hindi.
· V sits third to the left of the one who speaks English.
· Only U sits between the ones who speak Punjabi and Marathi.

1). Who among the following speaks English?


a) R
b) W
c) S
d) V
e) None of these

2). How many persons are there between those who speak Punjabi and
Telugu respectively?
a) None
b) One
c) Three
d) Four
e) None of these

Page 24 Follow us: Official Site, Telegram, Facebook, Instagram, Instamojo


3). Who among the following is sitting third to the left of the one who
speaks Tamil?
a) The one who speaks Punjabi
b) The one who speaks English
c) The one who speaks Bengali
d) The one who speaks Telugu
e) None of these

4). The one who sits second to the right of W speaks which of the following
languages?
a) Marathi
b) Bengali
c) Telugu
d) Hindi
e) Odia

5). Who among the following sits opposite the one who speaks Odia?
a) P
b) The one who speaks Bengali
c) The one who speaks English
d) V
e) None of these

Directions (Q. 6-10): Study the following information carefully and answer
the questions given below:
Ten persons are sitting in two parallel rows containing five persons in
each row. In row 1— D, E, F, G, and H are sitting and all are facing north. In
row 2 — V, W, X, Y and Z are sitting and all are facing south. Each person also
likes a different TV channel, viz StarPlus, Life OK, Sony, Star Gold, Zee TV,
Colors, ETV, SAB TV,&TV and DD, but not necessarily in the same order.
H sits in the middle of the row and faces the one who likes Zee TV. Only
one person sits between Y and the one who likes Zee TV. Y faces one of the
immediate neighbours of the one who likes Colors. Only two persons sit
between E and the one who likes Colors. E faces the one who likes SAB TV. V
faces one of the immediate neighbours of F. V does not like SAB TV. G is an
immediate neighbour of F.
The one who likes Star Gold is an immediate neighbour of V. Z is not an
immediate neighbour of V and faces the One who likes Life OK. X faces the

Page 25 Follow us: Official Site, Telegram, Facebook, Instagram, Instamojo


one who likes&TV. The one who likes DD faces north. V does not face the one
who likes DD. The one who likes Star Plus sits on the immediate right of the
one who likes ETV. Y does not like Star Plus. The one who faces F does not
like ETV and G faces Y.

6). Which of the following pairs represent the people sitting at the extreme
ends of the rows 2?
a) XG
b) YZ
c) WF
d) VD
e) EV

7). Which of the following TV channels does Z like?


a) Star Plus
b) Colors
c) Zee TV
d) ETV
e) SAB TV

8). Which of the following statements is definitely true?


a) F faces X.
b) W likes SAB TV
c) W is an immediate neighbour of V.
d) The one who likes&TV is an immediate neighbour Of V.
e) The one who likes, Sony faces X.

9). Who among the following likes Star Gold?


a) Y
b) V
c) Z
d) X
e) W

10). Who among the following is/are the immediate(s) neighbour of the one
who likes&TV?
a) The one Who likes Sony and G
b) The one who likes ETV and H

Page 26 Follow us: Official Site, Telegram, Facebook, Instagram, Instamojo


c) W and E
d) The one who likes Life OK and H.
e) Z

Answers:
1)b 2)c 3)d 4)b 5)b 6)b 7)e 8)c 9)d 10)d

Explanation:
Directions (Q. 1-5):
All are facing away from the centre.

Directions (Q. 6-10):

Directions (11 – 15): Answer the questions on the basis of the information
given below.

Page 27 Follow us: Official Site, Telegram, Facebook, Instagram, Instamojo


An online shopping company has eight employees, F, G, H, I, J, K, L, and
M. They are seating around a square table facing centre such that 2 persons are
seating on its each side. Also they all have different mobile phones – Samsung
j7, Redmi 4A, Redmi 5A, moto G, oppo, Vivo , Panasonic , and Infocus. They
have different number of order of delivery, 1-8 number.

M has highest number of order, and F has lowest number of order of delivery.
The person who has Redmi 4A, has number 5 as order of delivery and sits third
to right of L. H has number 3 order of delivery and is immediate neighbour of
L. The person who has Moto G, sits second to right of H. G sits third to right of
M, who has neither Redmi 4A nor Moto G. Only one person sits between H and
who has Redmi 5A. The one who has Samsung j7, has one less number of order
of delivery than the one who has Infocus. F and K are immediate neighbours of
each other. Neither F nor K has Redmi 4A. The person who has Infocus sits
second to right of F. Two persons sit between I and who has oppo. I does not
have Redmi 4A. The one who has Redmi 5A has number 6 order of delivery.
The person who has Samsung j7 is not an immediate neighbour of the one who
has Redmi 4A. The person has Vivo sits second to left of F. The one who has
Vivo has one more number of order of delivery than H.

11. What is the number of order of delivery of K?


A) Five
B) Eight
C) Seven
D) Six
E) None of these

View AnswerOption D
Explanation:

Page 28 Follow us: Official Site, Telegram, Facebook, Instagram, Instamojo


12. Who likes Samsung j7 and Redmi 4A respectively?
A) L J
B) L H
C) H J
D) L G
E) None of these

View Answer
Option A

13. Who is third to right of H?


A) I
B) F
C) G
D) M
E) None of these

View Answer
Option A

14. How many persons Sit between M and I?


A) Two
B) Three
C) Four
D) One
E) None of these

View Answer
Option B

Page 29 Follow us: Official Site, Telegram, Facebook, Instagram, Instamojo


15. Who sits 4th to right of L like which of following mobile?
A) Panasonic
B) Redmi 4A
C) Redmi 5A
D) Infocus
E) None of these

View Answer
Option A

Page 30 Follow us: Official Site, Telegram, Facebook, Instagram, Instamojo


Puzzle

Directions (1-5): Study the following information carefully to answer the


given questions.

Eight persons C, K, L, M, R, S and T were born in different months January,


March, April, May, June, August, November and December. Each one of them
works in different company HCL, TCS, IBM, CTS, Accenture, Wipro, Infosys
and Tech Mahindra but not necessarily in the same order.

As many persons were born before M was born after the one who works in
CTS. G was born in a month which has less than 31 days. The one who works
in Tech Mahindra was born immediately after G. M does not work in Tech
Mahindra. Two persons were born between T and the one who works in IBM.
Not more than three persons were born between M and the one who works in
CTS. T was born after G. The one who works in TCS was born immediately
before the one who works in Wipro. L was born before K. Three persons were
born between C and the one who works in Accenture. C was born after S, who
does not work in Wipro. M was born before the one who works in CTS. As
many persons were born between T and the one who works in CTS were born
between K and the one who works in Wipro, who was born after K. The one
who works in IBM was born immediately after R. G does not work in HCL.

1) How many persons were born between K and the one who works in
IBM?

a) One

b) Two

c) Three

d) Four

e) None of these

Page 31 Follow us: Official Site, Telegram, Facebook, Instagram, Instamojo


2) If T is related to Infosys, S is related to Wipro, in the same way M is
related to which of the following?

a) HCL

b) Infosys

c) Tech Mahindra

d) Accenture

e) None of these

3) Which of the following statement is not true?

a) As many persons were born between G and the one who works in Wipro is
same as persons were born between S and the one who works in Tech Mahindra

b) More than three persons were born between M and the one who works in
Tech Mahindra

c) Only one person were born between C and the one who works in HCL

d) Less than two persons were born after C

e) All are true

4) If K is related to June, S is related to December, in the same way R is


related to which of the following?

a) August

b) November

c) May

d) January

Page 32 Follow us: Official Site, Telegram, Facebook, Instagram, Instamojo


e) None of these

5) Which of the following statement is true?

a) More than one person were born between R and the one who works in CTS

b) Three persons were born between S and the one who works in IBM

c) At least one person was born after S

d) S was born before the one who works in TCS

e) None is true

Answers :

Directions (1-5):

As many persons were born before M was born after the one who works in
CTS. Not more than three persons were born between M and the one who works
in CTS. M was born before the one who works in CTS. G was born in a month
which has less than 31 days. The one who works in Tech Mahindra was born
immediately after G. M does not work in Tech Mahindra.

Page 33 Follow us: Official Site, Telegram, Facebook, Instagram, Instamojo


Two persons were born between T and the one who works in IBM. As many
persons were born between T and the one who works in CTS were born
between K and the one who works in Wipro, who was born after K. T was born
after G. The one who works in TCS was born immediately before the one who
works in Wipro. L was born before K.

Page 34 Follow us: Official Site, Telegram, Facebook, Instagram, Instamojo


Case 3 will be dropped because two persons were born between T and the one
who works in IBM. As many persons were born between T and the one who
works in CTS were born between K and the one who works in Wipro, who was
born after K.

T was born after G. L was born before K. Three persons were born between C
and the one who works in Accenture. C was born after S, who does not work in
Wipro. The one who works in IBM was born immediately after R. G does not
work in HCL.

Case 2: A and B will be dropped because C does not work in Wipro. Three
persons were born between C and the one who works in Accenture.

1) Answer: B

2) Answer: D

3) Answer: C

4) Answer: B

5) Answer: C

Directions (6-10): Study the following information carefully to answer the


given questions:

Page 35 Follow us: Official Site, Telegram, Facebook, Instagram, Instamojo


Seven boxes K, L, M, N, P, R, and S are kept above one another in a rack
from top to bottom. Only three boxes are kept between M and N where M is
kept above N. L is kept three boxes below K and L is not kept at the bottom of
the rack. There are three boxes between P and R. Box S is above R. Box P is not
placed immediately above M.

6) How many boxes are kept between P and S?

a) 5

b) 3

c) None

d) 1

e) 2

7) How many boxes are kept above M?

a) 1

b) 2

c) 4

d) 3

e) None

8) Which among the following box is kept immediately below box L?

a) P

b) M

c) S

Page 36 Follow us: Official Site, Telegram, Facebook, Instagram, Instamojo


d) N

e) None of these

9) If R and P interchange their position then which among the following


box will be kept immediately below P?

a) L

b) S

c) N

d) None

e) M

10) Four of the following are alike in a certain way to form a group. Which
among the following does not belong to the group?

a) R-S

b) K-P

c) M-S

d) S-P

e) N-L

Directions (6-10):

6) Answer: d)

7) Answer: a)

8) Answer: c)

Page 37 Follow us: Official Site, Telegram, Facebook, Instagram, Instamojo


9) Answer: d)

10) Answer: c)

Explanation:

Only three boxes are kept between M and N where M is kept above N.

Here we get three cases Case 1, Case 2 and Case 3.

Page 38 Follow us: Official Site, Telegram, Facebook, Instagram, Instamojo


L is kept three boxes below K and L is not kept at the bottom of the rack.

Here we get another case, Case 3a.

There are three boxes between P and R. Box S is above R.

Here Case 1 and Case 3a gets eliminated.

Box P is not placed immediately above M.

Page 39 Follow us: Official Site, Telegram, Facebook, Instagram, Instamojo


Hence, the final arrangement is,

Page 40 Follow us: Official Site, Telegram, Facebook, Instagram, Instamojo


Blood Relation

Directions (1-3): There are ten members P, Q, R, S, T, U, V, W, X and Y in the


family, there are three generations of the family. There is equal male and
female. P is son-in-law of Y. Q sister X have only one brother P. Q and W are
the married couple. Q is the mother-in-law of R, who is sister of T. V is the
mother of Q. W is son-in-law of V. S is the only son of U, who is daughter-in-
law of V.

1. If Z is mother-in-law of P. then how is S related to Z?


A) Son
B) Daughter
C) Grandson
D) Granddaughter
E) Can’t be determined

View Answer
Option C

2. If M is husband of R, then how is W related to M?


A) Father
B) Father- in-law
C) Uncle
D) None of these
E) Can’t be determined

Page 41 Follow us: Official Site, Telegram, Facebook, Instagram, Instamojo


View Answer
Option A
3. If N is father of P, then how is U related to N?
A) Grandson
B) Granddaughter
C) Son
D) Daughter-in-law
E) None of these

View Answer
Option D
Directions (4-5): D is son of E. M has only two children- D and F. T is the
sister of U. F married to V. M has only daughter. W is the mother of E. T
married to D. X married to W, and M is the son of Y.

4. IF G is wife of U, the how is G related to T?


A) Aunt
B) Sister-in-law
C) Sister
D) Cannot be determined
E) None of these

View Answer
Option B
5. IF Y is mother-in-law of E, then how is Y related to F?
A) Grandmother
B) Mother
C) Niece
D) Aunt
E) Cannot be determined

View Answer

Page 42 Follow us: Official Site, Telegram, Facebook, Instagram, Instamojo


Option A

6. Pointing to a photograph, Priya said to Pawan, “I am the only daughter of


this man and his son is your maternal uncle”. How is Pawan’s father related
to Priya?
A) Father
B) Father-in-law
C) Brother
D) Nephew
E) Husband
View Answer
Option E
7. Pointing to a man in photograph Ayush said, “his father’s sister is the only
daughter of my paternal grandfather”. How is man’s father related to
Ayush?
A) Father
B) Father-in-law
C) Brother
D) Cannot be determined
E) None of these
View Answer
Option D
8. If ‘P $ Q’ means ‘P is mother of Q’, ‘P # Q’ means ‘P is father of Q’, and ‘P
@ Q’ means ‘P is husband of Q’, then in the expression “H @ T $ S # D”
what is the relationship of H with D?
A) Maternal Grandfather
B) Maternal Grandmother
C) Paternal Grandmother
D) Paternal Grandfather
E) None of these

View Answer

Page 43 Follow us: Official Site, Telegram, Facebook, Instagram, Instamojo


Option D

Directions (9-10): Study the following information carefully and answer the
questions which follow:

(a) P#Q means P is daughter of Q,


(b) P+Q means Q is brother P,
(c) P=Q means Q is sister of P,
(d) P&Q means P is son of Q,
(e) P*Q means P is father of Q, and
(f) P@Q means P is mother of Q

9. If expression ‘M@N=O+S&Z’ is true, then which of he following is


correct?
A) Z is father of S.
B) N is daughter of M.
C) M is wife of Z
D) N is sister of S.
E) Both A and C are correct

View Answer
Option E
10. Which of following indicates that A is daughter of B?
A) C*P#O@A@B
B) B*D#E@A@C
C) F@N#C*M
D) F@A#C+B
E) None of these

View Answer
Option B

Page 44 Follow us: Official Site, Telegram, Facebook, Instagram, Instamojo


Inequality

Direction (1-6): Relationship between different elements is shown in the


statements. Find if the conclusions also follow or not.

1. Statements: A > L ≥ H > M = D < G ≤ F; U ≤ K = P > M; N= K < S


Conclusions:
I. A > S
II. A = S
A) only I follows
B) only II follows
C) either I or II follows
D) neither I nor II follow
E) both I and II follow

View Answer
Option D
Solution:
A>M<K<S
So relationship cannot be determined between A and S. For either or
condition, all three <, >, = sign should be present in conclusion
2. Statements: B < G ≤ V = F > H; K = L > F ≤ N = P; D > N = S
Conclusions:
I. H < S
II. B < P
A) only I follows
B) only II follows
C) either I or II follows
D) neither I nor II follow
E) both I and II follow
View Answer
Option E

Page 45 Follow us: Official Site, Telegram, Facebook, Instagram, Instamojo


3. Statements: V < L = D < K ≥ H > S; P ≤ K = D > E; K = F ≤ Z
Conclusions:
I. Z > L
II. E < V
A) only I follows
B) only II follows
C) either I or II follows
D) neither I nor II follow
E) both I and II follow
View Answer
Option B
Solution:
Z ≥ K = D = L, So Z ≥ L
4. Statements: K = L < D ≤ G = P; U > E = D ≥ X ≤ B; P > O ≥ V < S
Conclusions:
I. V < X
II. P ≥ X
A) only I follows
B) only II follows
C) either I or II follows
D) neither I nor II follow
E) both I and II follow
View Answer
Option B
5. Statements: K = L < D ≤ G = P; U > E = D ≥ X ≤ B; P > O ≥ V < S
Conclusions:
I. U > K
II. P > B
A) only I follows
B) only II follows
C) either I or II follows
D) neither I nor II follow
E) both I and II follow
View Answer

Page 46 Follow us: Official Site, Telegram, Facebook, Instagram, Instamojo


Option A
6. Statements: B < G ≤ V = F > H; K = L > F ≤ N = P; D > N = S
Conclusions:
I. L > G
II. D > K
A) only I follows
B) only II follows
C) either I or II follows
D) neither I nor II follow
E) both I and II follow
View Answer
Option A
7. Which of the following would replace & and # in the following expression
so that ‘A > N’ holds true?
A > L ≥ H & M = D < G ≤ F; U ≤ K = M > P; N # K < S
A) =, =
B) ≥, >
C) ≥, ≥
D) <, =
E) None of these

View Answer
Option A
Solution:
A > L ≥ H = M = K = N, So A > N
8. In which of the following expressions does the expression ‘G < S’ definitely
hold true?
A) A = S < F ≥ H = K > G > D
B) D > A = G ≥ B = F ≤ S < H
C) A < O > G < H = H < S ≥ B
D) G = U ≤ B = S ≤ H = O < A
E) None of these

Page 47 Follow us: Official Site, Telegram, Facebook, Instagram, Instamojo


View Answer
Option C

9. In which of these expression ‘L ≥ R’ is definitely false?


A) W < R ≥ S ≥ Q < N> A ≥ L > V
B) N > L > M = D ≥ B = A > P = R
C) M ≤ A > L > W ≥ V ≤ B = P < R
D) S > L = C ≥ H = V ≥ P ≤ R = T
E) B > R ≤ A = M = Q ≤ T = L < G

View Answer
Option B
Solution:
in B) – L > R so ‘L ≥ R’ is definitely false
In A), C) and D) – relation between L and R cannot be determined, so
cannot be told that whether ‘L ≥ R’ is definitely false or not.
In E) – L ≥ R is true
10. Which of the following expressions is definitely false if the expression K >
O = G ≤ D > F = P ≥ Q < T is definitely true?
A) K > D
B) F ≥ T
C) F < G
D) D = Q
E) P < O
View Answer
Option D
Solution:
D < Q in given expression, so D = Q is definitely false

Page 48 Follow us: Official Site, Telegram, Facebook, Instagram, Instamojo


Data Sufficiency

Directions (Q. 1-5): Each of the questions below consists of a question and
three statements numbered I, II and III given below it. You have to decide
whether the data provided in the statement are sufficient to answer the question
and give answer.

1. What is R’s rank from the top in a class of 90 students?

I. R is 18 ranks below S, who is 54th from the bottom.


II. Q is 45th from the top and N is 10th from the bottom.
III. R is exactly in the middle of Q and N.

a. Only II
b. Only I and II
c. Either only I or both II and III
d. Only III
e. None of these

2. Six persons A, B, C, D, E and F are sitting around a rectangular


table, three each on longer sides and opposite each other. C and E are
sitting exactly opposite each other. A sits on the immediate right of B.
Is C sitting between two persons?

I. D is not opposite A and sits on the extreme end of the longer side.
II. If A and F exchange their places, A is on the immediate right of E.
III. If A and E exchange their places, E sits on the immediate right of C.

a. Only I
b. Only II
c. Only III
d. All I, II and III together

Page 49 Follow us: Official Site, Telegram, Facebook, Instagram, Instamojo


e. Any two statements

3. Four bikes Discover, Yamaha, ZMR and Honda are each of a


different engine capacity and give a different mileage. Does the bike
with the lowest engine capacity give the least mileage?

I. Discover gives the highest mileage but does not have the highest engine
capacity.
II. ZMR gives more mileage than Honda while Yamaha has a greater engine
capacity than ZMR, whose engine capacity is more than Discover.
III. Honda does not give the least mileage but has the highest engine
capacity.

a. Only I and II
b. Any two statements together are sufficient
c. Any of the three statements
d. Can’t be determined
e. All I, II and III together

4. On which day of the week did Rajeev visit Patna? (Assume that the
week starts from Monday.) I. Rajeev took leave on Wednesday. II.
Rajeev visited Patna the day after his mother’s visit to his house. III.
Rajeev’s mother visited Rajeev’s house neither on Monday nor on
Thursday.

a. Only I and III


b. All I, II and III together
c. Only II and III
d. Only I and II
e. Can’t be determined

Page 50 Follow us: Official Site, Telegram, Facebook, Instagram, Instamojo


5. Who among Ravi, Madhu, Rakhi, Roma, Ritu and Reena, each
having different heights, is the tallest?

I. Madhu is taller than Ravi but shorter than Ritu.


II. Only two of them are shorter than Rakhi.
III. Roma is taller than only Reena.

a. Only I and III


b. Only I and II
c. Only II and III
d. All I, II and III together
e. None of these

Answers:

Direction(1-5):

1. Answer: c)
2. Answer: b)
3. Answer: e)
4. Answer: e)
5. Answer: d)

DIRECTIONS (6-10)
Answer (1) if the data in statement I alone is sufficient to answer the question,
while the data in statement II alone are not sufficient to answer the question
Answer (2) if the data in statement II alone is sufficient to answer the question,
while the data in statement I alone are not sufficient to answer the question
Answer (3) if the data either in statement I alone or in statement II alone are
sufficient to answer the question
Answer (4) if the data even in statement I and II together are not sufficient to
answer the question
Answer (5) if the data in both statement I and II together are necessary to answer
the question

Page 51 Follow us: Official Site, Telegram, Facebook, Instagram, Instamojo


QUESTION 6
The Chairman of a big company visits one department on Monday of every week
expect for the Monday of 3rd week of every month. When did he visit the Purchase
department?
Statement:
I. He visited Accounts department in the 2nd week of September after having
visited Purchase department on the earlier occasion
II. He had visited Purchase department immediately after visiting Stores
department but before visiting Accounts department
Solution
From statement I alone we can deduce that the chairman visited the Purchase
department on Monday of the 1st week of September
From statement II alone we cannot determine the time of visit of any of the
departments. Therefore this statement is not sufficient enough to answer the
question
Therefore, statement I alone is sufficient to answer the question, while the data in
statement II alone are not sufficient to answer the question
Hence Answer: (1)

QUESTION 7
P, Q, R, S and T are sitting around a circular table facing towards the centre. Who
is 2nd to the right of P?
Statement:
I. R is to the immediate left of T and 2nd to the right of S
II. Q is to the immediate right of S and 3rd to the left of P
Solution
From statement I alone we cannot deduce the arrangements
Therefore, the 2nd position to the right of P can be occupied by either Q or T. It
cannot be determined by statement I alone

From statement II alone we can come to the following conclusions


Here the 2nd position to the right of P can be clearly determined as Q
Therefore, statement II alone is sufficient to answer the question, while the data
in statement I alone are not sufficient to answer the question
Hence Answer: (2)

Page 52 Follow us: Official Site, Telegram, Facebook, Instagram, Instamojo


QUESTION 8
In a certain code language, ‘2 9 7’ means ‘tie clip button’. Which number means
‘button’ in that language?
Statement:
I. In the same code language, ‘9 2 6’ means ‘clip your tie’
II. In the same language, ‘1 7 5’ means ‘hole and button’
Solution
From statement I alone we can deduce the common words between the question
and this statement are ‘tie’ and ‘clip’ and the common codes are ‘2’ and ‘9’. So,
‘2’ and ‘9’ are codes for tie and clip. Therefore the code for ‘button’ is ‘7’
From statement II alone we can see that the word ‘button’ is common between
the question and statement II. Similarly ‘7’ is the common code between them.
Therefore, ‘button’ is ‘7’
Therefore, statement I alone or in statement II alone are sufficient to answer the
question
Hence Answer: (3)

QUESTION 9
Among A, B, C, D and E, who earns more than only the least earner among them?
Statement:
I. B earns more than A and D but less than only C
II. A earns more than D who earns less than E

Therefore either E or A earns more than the least earner D and we don’t have any
other clue to determine which one of them is the exact answer
Therefore, data even in statement I and II together are not sufficient to answer the
question
Hence Answer: (4)

QUESTION 10
How is P related to Q?
Statement:
I. R’s sister S has married P’s brother T, who is the only son of his parents
II. Q is the only daughter of T and S
Solution

Page 53 Follow us: Official Site, Telegram, Facebook, Instagram, Instamojo


From statement I alone we know that T is P’s brother and S’s husband. Since T
is the only son of his parents, P is T’s sister
From statement II alone we can understand that Q is T’s daughter
Now considering statements I and II together, we can conclude that P is the sister
of Q’s father i.e., P is Q’s aunt
Here answer cannot be found using one statement alone.
Therefore, data in both statements I and II together are necessary to answer the
question
Hence Answer: (5)

Direction (11-15): In each of the following questions, two statements


numbered I and II are given. There may be cause and effect relationship
between the two statements. These two statements may be the effect of the
same cause or independent causes. These statements may be independent
causes without having any relationship. Read both the statements in each
question and mark your answer as

a). Statement I is cause and Statement II is its effect

b). Statement II is cause and Statement I is its effect

c). Both statement I and II are independent cause

d). Both Statement I and Statement II are effect of independent cause

e). Both statement I and Statement II are effects of some common cause

11). Statement I: Indian Railway department was one of the leading public sector
organisation, spread all over India. It was came out with the huge number of
vacancies recently.

Statement II: India has huge number of educated unemployed candidates. The
candidates was awaiting for the chance to show their potential.

Page 54 Follow us: Official Site, Telegram, Facebook, Instagram, Instamojo


12). Statement I: The three-day annual Krishi Unnati Mela has begun in New
Delhi with a focus on doubling farmers’ income.

Statement II: Now-a-days, famers are suffered a lot by poverty. The Climatic
conditions which we faced in India were reducing the income of farmers.

13). Statement I: Public sector Indian Overseas Bank has joined hands with the
National Housing Bank for implementation of the rural housing interest subsidy
scheme offered under the Ministry of Rural Development.

Statement II: Central government was announced the scheme to provide the
shelter for all the Indian citizens, under the subsidy rate, especially to develop the
rural India

14). Statement I: Facebook has prohibited advertisements that promote financial


services that are associated with misleading or deceptive promotional practices,
naming cryptocurrencies and initial coin offering.

Statement II: Facebook gradually added support for students at various other
universities, and eventually to high school students.

15). Statement I: New India Assurance Company, India’s largest non-life


insurance company launched a comprehensive worldwide mediclaim policy
named ‘New India Global Mediclaim Policy’.

Statement II: Most of the Indian’s are unable to cover their unexpected medical
expenses, because of timely need of huge capital. So, they were looking for the
help of insurance companies to provide it.

Direction (11-15)

Page 55 Follow us: Official Site, Telegram, Facebook, Instagram, Instamojo


11. Answer: C

12. Answer: D

13. Answer: B

14. Answer: D

15. Answer: B

Page 56 Follow us: Official Site, Telegram, Facebook, Instagram, Instamojo


Coding and Decoding

Question 1: If EARTH is written as FCUXM in a certain code. How is MOON


written in that code?
Solution :

Question 2: If DELHI is written as EDMGJ in a certain code. How is NEPAL


written in that code?
Solution :

Question 3: If SYMBOL is written as NZTMPC is a certain code. How is


NUMBER written in that code?
Solution : Acc. to question

Page 57 Follow us: Official Site, Telegram, Facebook, Instagram, Instamojo


Question 4: In a certain code, COMPUTER is written as PMOCRETU, how is
DECIPHER written in that code?
Solution : Acc. to question

Question 5: In a certain code, NEWYORK is written as 111, how is


NEWJERSEY written in that code?
Solution : In NEWYORK
N = 14, E = 5, W = 23, Y = 25, O = 15, R = 18, K = 11
Total = 14 + 5 + 23 + 25 + 15 + 18 + 11 = 111
In NEWJERSEY
Total = 14 + 5 + 23 + 10 + 5 + 18 + 19 + 5 + 25
= 124

Question 6: In a certain code, HARYANA is written as 8197151, how is


DELHI written in that code?
Solution : We used the number of alphabets here.
H=8
A=1
R = 18 = 1+8 = 9
Y = 25 = 2+5 = 7
For DELHI
D=4
E=5
L = 12 = 1+2 = 3
H=8

Page 58 Follow us: Official Site, Telegram, Facebook, Instagram, Instamojo


I=9
Hence, DELHI is written as 45389.

Question 7: In a certain code BOMB is written as 5745 and BAY is written as


529, how is BOMBAY written in that code?
Solution : Acc. to question
Use the numbers to relate the words
BOMB BAY -> BOMBAY
5745 529 574529

Direction (8-12): Read the following information carefully and answer the
questions given below.

In certain codes:

“Newspaper may be choice published” was coded as “lkf ero kof jow rei”

“All newspaper was mobile power” was coded as “ero ljb qwr nbc wea”

“No Mobile was charging power” was coded as “qwr oes ljb wea koq”

“All charging may choice power” was coded as “qwr kof lkf nbc oes”

8). What is the code for the word “no”?

a). kof

b). koq

c). jow

d). rei

Page 59 Follow us: Official Site, Telegram, Facebook, Instagram, Instamojo


e). lkf

9). What is the code for the word “Choice”?

a). lkf

b). oes

c). kof

d). Either b or c

e). Either a or c

10). What is the code for the word “published”?

a). jow

b). rei

c). ero

d). Either a or c

e). Either a or b

11). What is the code for the word “power”?

a). ljb

b). oes

c). nbc

d). wea

e). qwr

Page 60 Follow us: Official Site, Telegram, Facebook, Instagram, Instamojo


12). What is the code for the word “mobile”?

a). kof

b). ljb

c). wea

d). nbc

e). Cannot be determined

Answers:

Direction (8-12)

Newspaper Ero

may / choice kof / lkf

be/ published jow / rei

All Nbc

Power Qwr

mobile/was ljb/wea

Charging Oes

No Koq

8 Answer: B

Page 61 Follow us: Official Site, Telegram, Facebook, Instagram, Instamojo


9. Answer: E

10. Answer: E

11. Answer: E

12. Answer: E

Page 62 Follow us: Official Site, Telegram, Facebook, Instagram, Instamojo


Distance and Direction

(Directions 1 – 5): Study the following information and answer the given
questions.
Seven friends I, J, K, L, M, N and O had their houses situated along a straight
row facing the north. They like different items among Eraser, Pencil, Pen,
Scale, Book, Board and Table but not necessarily in the same order. The
distance between the neighbouring houses was a successive integral multiple of
5km and the distance increased from left to right.
I’s house was third to the left of the friend who likes board. J’s house was
135km to the right of the friend who likes scale. K’s house was exactly between
O’s house and the friend who likes table. The friend who likes book was to the
immediate left of the friend who likes pen. The friend who likes pen was 115km
to the left of L’s house. The friend who likes board was the neighbour of L’s
house. The friend who likes scale was 85km away from M’s house. The friend
who likes eraser had house at one of the extreme ends. O did not like pencil.
The distance between any two houses was less than 80 km.

1. Which item does N likes?


1) Eraser
2) Board
3) Pencil
4) Table
5) Book

Answer & Explanation

Answer – 2) Board
Solution:

Page 63 Follow us: Official Site, Telegram, Facebook, Instagram, Instamojo


Page 64 Follow us: Official Site, Telegram, Facebook, Instagram, Instamojo
Hence, N likes Board.

2. What is the distance between K and the friend who likes Pen?
1) 95km
2) 170km
3) 150km
4) 135km
5) 85km

Page 65 Follow us: Official Site, Telegram, Facebook, Instagram, Instamojo


Answer & Explanation

Answer – 4) 135km
Hence, distance between K and the friend who likes pen is 135km.

3. How many houses are there between I and the friend who likes
pencil?
1) None
2) One
3) Two
4) Three
5) Four

Answer & Explanation

Answer – 4) Three
Hence, three houses are there between I and the friend who likes
pencil.

4. Who among the following sits to the immediate left of M?


1) I
2) The one who likes pen
3) O
4) The one who likes pencil
5) J

Answer & Explanation

Answer – 1) I
Hence, I sits to the immediate left of M.

Page 66 Follow us: Official Site, Telegram, Facebook, Instagram, Instamojo


5. Who among the following has distance of 120km between them?
1) I, The one who likes pen
2) J, L
3) O, The one who likes book
4) K, The one who likes pen
5) None of these

Answer & Explanation

Answer – 3) O, The one who likes book


Hence, O and the one who likes book have distance of 120km between
them.

Direction (6 – 10): Read the following information carefully and answer the
questions given beside.

Six friends I, J, K, L, M and N are sitting in line AB, facing south direction and
distance between two adjacent friends increases when we move from point A to
point B in consecutive integral multiple of 9m. Minimum possible distance
between two friends is 18m in line AB. Two friends sit between K and N. L sits
to the immediate left of J and neither of them is sitting next to the person sitting
at extreme ends of the line. I is first person from the right end. M and N are not
immediate neighbours. Person sitting nearest to point B is 7m away from point
B.
Six friends M, N, O, P, Q and R are sitting in the line XY facing south and
distance between two adjacent friends increases when we move from point X to
point Y in consecutive integral multiple of 5m. Atmost the distance between
two immediate neighbours in line XY is 30m. N sits towards left of O and
distance between them is 75m. M sits at the immediate right of the Q and there
is no person to the right of M. R does not sit towards left of P. Person sitting
farthest from point X is at distance of 104m from point X. O does not sit nearest
to point X.

Page 67 Follow us: Official Site, Telegram, Facebook, Instagram, Instamojo


Based on the above conditions:
I. In line AB, people whose distance from B is an odd number will move to line
CD maintaining their respective positions and others will remain at their
respective positions.
II. In line XY, people whose distance from X ends up with the number with unit
digit 4 will move to line UV maintaining their respective positions and others
will remain at their respective positions.
Length of line AB is 189m, XY is 107m, CD is 42m and UV is 78m.

6. How many friends changed their positions in line AB?


1) Five
2) Three
3) Two
4) Four
5) One

Answer & Explanation

Answer – 4) Four
Solution:

Page 68 Follow us: Official Site, Telegram, Facebook, Instagram, Instamojo


Page 69 Follow us: Official Site, Telegram, Facebook, Instagram, Instamojo
Hence, four friends changed their positions in line AB.

7. After movement, what is the distance between the any two persons
in the line XY and in case only one person left choose option 5.
1) 15m
2) 20m
3) 25m
4) 10m
5) Cannot be determined

Answer & Explanation

Page 70 Follow us: Official Site, Telegram, Facebook, Instagram, Instamojo


Answer – 2) 20m
Hence, distance between O and R is 20m.

8. How many persons move to line UV?


1) None
2) One
3) Two
4) Three
5) Four

Answer & Explanation

Answer – 5) Four
Hence, four persons move to line UV.

9. What is the total distance between M and R?


1) 30m
2) 49m
3) 50m
4) 45m
5) None of these

Answer & Explanation

Answer – 4) 45m
Hence, distance between M and R is 45m.

10. What is the distance of N from point Y?


1) 2m
2) 3m

Page 71 Follow us: Official Site, Telegram, Facebook, Instagram, Instamojo


3) 4m
4) 5m
5) 6m

Answer & Explanation

Answer – 2) 3m
Hence, distance of N from point Y is 3m.

Page 72 Follow us: Official Site, Telegram, Facebook, Instagram, Instamojo


Ranking

1).In a row of 25 children, Nayan is 14th from the right end. Arun is 3rdto the left
of Nayan in the row. What is Arun’s position from the left end of the row?

a) 8th
b) 9th
c) 7th
d) 10th
e) None of these

2).A class of girls stands in a single line. One girl is 19th in order from both the
ends. How many girls are there in the class?
a) 27
b) 37
c) 38
d) 39
e) None of these

3).Surbhi ranks 18th in a class of 49 students. What is her rank from the last?
a) 31
b) 28
c) 35
d) 32
e) None of these

4).There are 35 students in a class. Suman ranks third among the girls in the
class. Amit ranks 5th among the boys in the class. Suman is one rank below
Amit in the class. No, two students hold the same rank in the class. What is
Amit’s rank in the class?
a) 7th
b) 5th
c) 8th

Page 73 Follow us: Official Site, Telegram, Facebook, Instagram, Instamojo


d) Cannot be determined
e) None of these

5).In a row of children facing North, Ritesh is 12th from the left end. Sudhir,
who is 22nd from the right end is 4th to the right of Ritesh. Total how many
children are there in the row?
a) 35
b) 36
c) 37
d) 34
e) None of these

6).In a class of 45 students, a boy is ranked 20th. When two boys joined, his rank
was dropped by one. What is his new rank from the end?
a) 25th
b) 26th
c) 27th
d) 28th
e) None of these

7).Sohan ranks 7th from the top and 26th from the bottom in a class. How many
students are there in the class?
a) 31
b) 32
c) 33
d) 34
e) None of these

8).There are 25 boys in a horizontal row. Rahul was shifted by three places
towards his right side and he occupies the middle position in the row. What was
his original position from the left end of the row?
a) 15th
b) 16th
c) 12th

Page 74 Follow us: Official Site, Telegram, Facebook, Instagram, Instamojo


d) 10th
e) None of these

9).In a row of letters, a letter is 5th from left end and 12thfrom the right end. How
many letters are there in a row?
a) 15
b) 16
c) 17
d) 18
e) None of these

10).Sunita is the 11th from either end of a row of girls. How many girls are there
in that row?
a) 19
b) 20
c) 21
d) 22
e) None of these

Answers:
1). b) 2). b) 3). d) 4). a) 5). c) 6). c) 7). b) 8). d) 9). b) 10). c)

Explanation:

1).Nayan’s position from left = (25 + 1 – 14) = 12th


Arun’s position = (12 – 3) = 9th from left
Answer: b)

2).Clearly, the girl at the 19th position is exactly in the middle of both the ends
Total number of girls = (18 + 1 + 18) = 37
Answer: b)

Page 75 Follow us: Official Site, Telegram, Facebook, Instagram, Instamojo


3).Surbhi’s rank from last = (49 + 1 – 18) = 32
Answer: d)

4).Suman ranks 3rd in the class among the girls,


Amit ranks 5th among the boys.
Suman comes one rank after Amit in the class
It means two girls and four boys rank higher than Amit in the class
So, Amit ranks 7th in the class.
Answer: a)

5).Sudhir’s position from left = (12 + 4) = 16th from left


Clearly, there are three students between Ritesh and Sudhir.
Total number of children = (12 + 3 + 22) = 37
Answer: c)

6).Total number of boys after 2 new boys joined = 47


Since, the rank of the boy dropped by 1, it became 21 st 20th
His new rank from the end = 47 – 21 + 1 = 27th
Answer: c)

7).Total students in the class = 7 + 26 – 1 = 33 – 1 = 32


Answer: b)

8).When Rahul was shifted by three places towards his right side, then he
occupies the middle position i.e., 13th position
Thus, his original position from the left end of the row = 13 – 3 = 10th
Answer: d)

9).Number of letters in the row = 5 + 12 – 1 = 16


Answer: b)

Page 76 Follow us: Official Site, Telegram, Facebook, Instagram, Instamojo


10).Total number of girls = 11 + 11 – 1 = 22 – 1 = 21
Answer: c)

Page 77 Follow us: Official Site, Telegram, Facebook, Instagram, Instamojo


QUANTITATIVE APTITUDE
SYLLABUS:

 Number Series
 Data Interpretation
 Average
 Profit & Loss
 Time & Work
 Data Sufficiency
 Simplification/Approximation
 Quadratic Equation
 Time & Distance
 Partnership
 Ratio & Proportion
 Problem on Ages
 Boats & Streams
 Pipe & Cisterns

Page 78 Follow us: Official Site, Telegram, Facebook, Instagram, Instamojo


Number Series

1). 9, 11, 16, 26, ? 69

Logic: diff off diff 3,5,7,9..

9+2=11

11+5=16

16+10=26

26+17=43

43+26=69

Answer: 43

2). 3, 4, 10, 33, 136, ?

Logic: 3×1+1=4

4×2+2=10

10×3+3=33

33×4+4=136

136×5+5=685

Answer: 685

3). 21, 24, 32, 45, 63, ?

Logic: diff of diff 5. = 86

Page 79 Follow us: Official Site, Telegram, Facebook, Instagram, Instamojo


21+3=24

24+8=32

32+13=45

45+18=63

63+23=86

Answer: 86

4). 6, 3, 3, 6, 24, ?

Logic: 6×.5=3

3×1=3

3×2=6

6×4=24

24×8=192

Answer: 192

5). 7, 10, 16, 28, ? 100

Logic: 7+3=10

10+6=16

16+12=28

28+24=52

52+48=100

Page 80 Follow us: Official Site, Telegram, Facebook, Instagram, Instamojo


Answer: 52

6). 7, 9, 14, 24, ?, 67

Logic: Diff of Diff – +3, +5 , +7, +9

7+2 = 9

9+5=14

14+10=24

24+17= 41

41+26=67

Answer: 41

7). 9, 13, 21, 37, ? 13

Logic: 9+4=13

13+8=21

21+16=37

37+32=69

69+64 = 133

Answer: 69

8). 24, 31, 43, 60, 82, ?

Logic: 24+7=31

31+12=43

Page 81 Follow us: Official Site, Telegram, Facebook, Instagram, Instamojo


43+17=60

60+22=82

82+27=109

Answer: 109

9). 4, 5, 12, 39, 160, ?

Logic: 4×1+1 = 5

5×2+2 =12

12×3+3 = 39

39 x4 +4 = 160

160×5 +5 = 805

Answer: 805

10). 8, 4, 4, 8, 32, ?

Logic: 8x 0.5 = 4

4x 1 = 4

4x2=8

8 x 4 = 32

32 x 8 = 256

Answer: 256

11). 11, 17, 29, 53, ? 197

Page 82 Follow us: Official Site, Telegram, Facebook, Instagram, Instamojo


Logic: 11×2-5= 17

17×2-5 = 29

29×2-5 = 53

53×2-5 = 101

101×2-5 =197

Answer:101

12). 11, 13, 18, 28, ? 71

Logic: Difference of Difference 3,5,7,9

11+2=13

13+5=18

18+10=28

28+17=45

45+36=71

Answer:45

13). 5, 6, 14, 45, 184, ?

Logic: 5×1+1=6

6×2+2=14

14×3+3=45

Page 83 Follow us: Official Site, Telegram, Facebook, Instagram, Instamojo


45×4+4=184

184×5+5=925

Answer:925

14). 14, 7, 7, 14, 56, ?

Logic: 14x 0.5 = 7

7x 1 = 7

7 x 2 = 14

14 x 4 =56

56x 8 = 448

Answer: 448

15). 13, 20, 34, 62, ?, 230

Logic: 13+7×1=20

20+7×2=34

34+7×4=62

62+7×8=118

118+7×16=230

Answer: 230

16). 6, 7, 16, 51, 208, ?

Logic: 6×1+1=7

Page 84 Follow us: Official Site, Telegram, Facebook, Instagram, Instamojo


7×2+2=16

16×3+3=51

51×4+4=208

208×5+5=1045

Answer: 1045

17). 26, 34, 47, 65, 88, ?

Logic: Difference of Difference 5, 5, 5, 5

26+8=34

34+13=47

47+18=65

65+23=88

88+28=116

Answer:116

18). 13, 15, 20, 30, ? 73

Logic: Difference of Difference 3, 5, 7, 9

13+2=15

15+5=20

20+10=30

30+17=47

Page 85 Follow us: Official Site, Telegram, Facebook, Instagram, Instamojo


47+26=73

Answer:47

19). 4, 2, 2, 4, 16, ?

Logic: 4x 0.5 = 2

2x 1 = 2

2x2=4

4 x 4 = 16

16 x 8 = 128

Answer: 128

20). 28, 31, 39, 52, 70, ?

Logic: Difference of Difference 5, 5, 5, 5

28+3=31

31+8=39

39+13=52

52+18=70

70+23=93

Answer:93

21). 14, 13, 24, 69, ?, 1355

Logic: ×1-1,×2-2,×3-3,..

Page 86 Follow us: Official Site, Telegram, Facebook, Instagram, Instamojo


Answer: 272

22). 7, 16, 11, 20, 15,?

Logic: Difference +9,-5,+9,-5

Answer: 24

23). 7, 7, 10, 18, 33, ?

Logic: Difference of difference 3, 5, 7, 9 .

Answer: 57

24). 7, 15, 79, 591, ?

Logic: Difference 8, 8^2, 8^3

Answer: 4687

25). 3, 10, 18, 28, ?

Logic: Difference of Difference 1, 2 ,3

Answer: 41

26). 9 ,9, 12, 20, 35, ?

Logic: Difference of Difference +3, +5, +7, +9

Answer: 59

27). 4, 13, 23, 35, 50, ?

Logic: Difference of Difference +1, +2, +3, +4

Answer: 69

Page 87 Follow us: Official Site, Telegram, Facebook, Instagram, Instamojo


28). 9, 17, 81, 593, ?, 37457

Logic: Difference 8, 8^2, 8^3

Answer: 4689

29). 7, 20, 13, 26, 19, ?

Logic: Difference +13,-7,+13,-7

Answer: 32

30). 9, 16, 13, 20, 17, ?

Logic: Difference +7,-3,+7,3

Answer: 24

31). 5, 11, 18, 27, 38, ?

Logic: x2+1, x2-4, x2-9, x2-16, x2-25

Answer: 51

32). 8, 7, 12, 33, ?, 635

Logic: x1-1, x2-2, x3-3, x4-4

Answer: 128

33). 11 , 11, 14, 22, 37, 61, ?

Logic: Difference of Difference +3, +5, +7, +9

Answer: 96

34). 6, 10, 19, 32, 48, ?

Page 88 Follow us: Official Site, Telegram, Facebook, Instagram, Instamojo


Logic: Difference of difference 5, 4, 3, 2 .

Answer: 66

35). 7, 20, 46, 85, ?

Logic: Difference of difference 13, 13, 13, 13 .

Answer: 137

36). 4, 13, 40, 121, ?

Logic: x3+1, x3+1, x3+1, x3+1

Answer: 364

37). 2180, 2179, 2152, 2027, ?

Logic: -13, -33, -53, -73

Answer:1684

38). 17, 18, 14, 23, 7, ?

Logic: +12 , -22, +32, -42, +52

Answer: 32

39). 97, 98, 90, 117, ?, 178

Logic: +13, -23, +33, -43

Answer: 53

40). 8, 11, 20, 47, ?, 371

Logic: x3, 32, 33, 34, 35

Page 89 Follow us: Official Site, Telegram, Facebook, Instagram, Instamojo


Answer: 128

41). 7, 10, 15, 24, 39, ?

Logic: Difference of difference 2, 4, 6, 8

Answer: 62

42). 5, 14, 41, 122, ?, 1094

Logic: x3-1, x3-1, x3-1, x3-1

Answer: 365

43). 18, 9, 9, 13.5, ?, 67.5

Logic: x0.5, x1, x1.5, x2, x2.5

Answer: 27

Page 90 Follow us: Official Site, Telegram, Facebook, Instagram, Instamojo


Data Interpretation

Directions (1 – 5): Study the following information carefully and answer


the questions given below.

The below graph shows the total number of students placed in campus interview
from two colleges P and Q in five different years.

1) If the respective ratio of total number of students placed in campus


interview from colleges P and Q together in 2017 and 2019 is 5: 7, what was
the total number of students placed from the colleges P and Q together in
2018?

a) 456

b) 530

c) 378

d) 560

e) None of these

Page 91 Follow us: Official Site, Telegram, Facebook, Instagram, Instamojo


2) In 2016, 40% of students placed in campus interview from college P and
30% of students placed in campus interview from college Q were females.
What was the total number of female students placed in campus interview
from colleges P and Q together in 2016?

a) 62

b) 64

c) 66

d) 68

e) None of these

3) Number of students placed in campus interview from college P increased


by what percent from 2014 to 2015?

a) 40%

b) 80%

c) 85%

d) 70%

e) None of these

4) What is the average number of students placed in campus interview


from college Q in 2014, 2015, 2016, 2017 and 2018?

a) 146

b) 141

c) 123

d) 132

Page 92 Follow us: Official Site, Telegram, Facebook, Instagram, Instamojo


e) None of these

5) In 2015, out of the total number of students placed in campus interview


from colleges P and Q together, only 60% of the students accepted the job
offer. What was the total number of students from colleges P and Q
together accepted the job offer in 2015?

a) 198

b) 190

c) 189

d) 160

e) None of these

Directions (Q. 6– 10) Study the following information carefully and answer
the given questions:

Following table shows the total number of graduates attended the interview in
different venues of a certain company. The ratio of male and female among
them and the percentage of selected candidates also given.

Page 93 Follow us: Official Site, Telegram, Facebook, Instagram, Instamojo


6) Total number of male graduates who attended the interview in Mumbai,
Kolkata and Hyderabad together is approximately what percentage of total
number of female graduates who attended the interview in Chennai, Delhi
and Bangalore together?

a) 81 %

b) 72 %

c) 60 %

d) 105 %

e) 95 %

7) Find the difference between the total number of selected candidates in


Chennai, Mumbai and Bangalore together to that of total number of
selected candidates in Delhi, Kolkata and Hyderabad together?

a) 412

b) 460

c) 325

Page 94 Follow us: Official Site, Telegram, Facebook, Instagram, Instamojo


d) 365

e) None of these

8) Find the ratio between the total number of graduates who attended the
interview in Delhi, Mumbai and Bangalore together to that of total number
of females who attended the interview in all the given venue’s together?

a) 123 : 95

b) 295 : 267

c) 85 : 54

d) 12 : 7

e) None of these

9) Total number of males who attended the interview is approximately


what percentage more/less than the total number of females who attended
the interview in all the given venue’s together?

a) 4 % more

b) 20 % less

c) 20 % more

d) 4 % less

e) 10 % more

10) Find the total number of candidates selected for the job in all the given
venue’s together?

a) 5230

b) 5970

Page 95 Follow us: Official Site, Telegram, Facebook, Instagram, Instamojo


c) 6875

d) 6125

e) None of these

Answers :

Directions (6-10) :

6) Answer: a)

Total number of male graduates who attended the interview in Mumbai, Kolkata
and Hyderabad together

= > 1600*(1/4) + 1400*(3/5) + 1800*(5/9)

= > 400 + 840 + 1000

= > 2240

Total number of female graduates who attended the interview in Chennai, Delhi
and Bangalore together

= > 1800*(3/5) + 2100*(1/3) + 2200*(5/11)

= > 1080 + 700 + 1000

= > 2780

Required % = (2240/2780)*100 = 80.57 % = 81 %

7) Answer: d)

The total number of selected candidates in Chennai, Mumbai and Bangalore


together

= > 1800*(75/100) + 1600*(58/100) + 2200*(61/100)

Page 96 Follow us: Official Site, Telegram, Facebook, Instagram, Instamojo


= > 1350 + 928 + 1342 = 3620

The total number of selected candidates in Delhi, Kolkata and Hyderabad


together

= > 2100*(67/100) + 1400*(42/100) + 1800*(70/100)

= > 1407 + 588 + 1260 = 3255

Required difference = 3620 – 3255 = 365

8) Answer: b)

The total number of graduates who attended the interview in Delhi, Mumbai
and Bangalore together

= > 2100 + 1600 + 2200 = 5900

The total number of females who attended the interview in all the given venue’s
together

= > 1800*(3/5) + 2100*(1/3) + 1600*(3/4) + 1400*(2/5) + 2200*(5/11) +


1800*(4/9)

= > 1080 + 700 + 1200 + 560 + 1000 + 800 = 5340

Required ratio = 5900: 5340 = 295: 267

9) Answer: a)

Total number of males who attended the interview in all the given venue’s
together

= > 1800*(2/5) + 2100*(2/3) + 1600*(1/4) + 1400*(3/5) + 2200*(6/11) +


1800*(5/9)

= > 720 + 1400 + 400 + 840 + 1200 + 1000 = 5560

Page 97 Follow us: Official Site, Telegram, Facebook, Instagram, Instamojo


Total number of females who attended the interview in all the given venue’s
together

= > 1800*(3/5) + 2100*(1/3) + 1600*(3/4) + 1400*(2/5) + 2200*(5/11) +


1800*(4/9)

= > 1080 + 700 + 1200 + 560 + 1000 + 800 = 5340

Required % = [(5560 – 5340)/5340]*100 = 4.12 % = 4 % more

10) Answer: c)

The total number of candidates selected for the job in all the given venue’s
together

= > 1800*(75/100) + 2100*(67/100) + 1600*(58/100) + 1400*(42/100) +


2200*(61/100) + 1800*(70/100)

= > 1350 + 1407 + 928 + 588 + 1342 + 1260

= > 6875

Page 98 Follow us: Official Site, Telegram, Facebook, Instagram, Instamojo


Average

1. The average age of girls in the class is 15 years. The average age of
boys in the class also added, then the average becomes 18. If there
are 18 boys in the class and the average age of boys in the class is
20, then find how many girls in the class?

a. 18
b. 24
c. 12
d. 16
e. None of these

2. The average marks of Raji in a certain examination is 85. If she got


20 more marks in Maths and 12 more marks in English, then the
average becomes 89. Find the total number of subjects she studied?

a. 8
b. 5
c. 7
d. 9
e. None of these

3. The average salary of the whole employees in a company is Rs. 300


per day. The average salary of officers is Rs. 800 per day and that
of clerks is Rs. 240 per day. If the number of officers is 30, then find
the number of clerks in the company?

a. 180
b. 160
c. 220
d. 250
e. None of these

Page 99 Follow us: Official Site, Telegram, Facebook, Instagram, Instamojo


4. The average age of 25 students in a class is 20. If the age of a teacher
is included, then the average age increased by one year, find the age
of the teacher?

a. 42 years
b. 46 years
c. 38 years
d. 34 years
e. None of these

5. The average height of 20 students is 105 cm. But later it was found
that, the heights of 2 students were wrongly entered as 75 instead of
68 and 91 instead of 108. Then find the correct average?

a. 98 cm
b. 84 cm
c. 5 cm
d. 5 cm
e. None of these

6. The Average marks obtained by 45 students in an examination is


12. If the average marks of passed students are 14 and that of failed
students are 5, then find the number of students who passed the
examination?

a. 35
b. 42
c. 30
d. 26
e. None of these

7. The average weight of 20 people is increased by 2.2 kg when one


man weight 53 kg is replaced by another man. Find the weight of
new man?

Page 100 Follow us: Official Site, Telegram, Facebook, Instagram, Instamojo
a. 88 kg
b. 82 kg
c. 93 kg
d. 97 kg
e. None of these

8. Average number of sweets distributed in a class of 52 students is 5.


If some number of students newly joined in the class and the average
becomes 4, then find the number of newly joined students?

a. 18
b. 13
c. 22
d. 24
e. None of these

9. The average weight of 50 students in a class is 60 kg. The weights of


two students were wrongly entered as 74 kg and 66 kg instead of 66
kg and 54 kg respectively. Find the corrected average weight of the
class?

a. 58 kg
b. 50 kg
c. 60 kg
d. 80 kg
e. 20 kg

10. The average of 5 numbers is 120. The average of the first two
numbers is 125 and the average of the last two numbers is 130. What
is the third number?

a. 90
b. 110
c. 100
d. 125

Page 101 Follow us: Official Site, Telegram, Facebook, Instagram, Instamojo
e. None of these

Answers:

1.Answer c

Let the total number of girls in the class be x,

15x + (20*18) = (x+18)*18

15x+360 = 18x+324

36 = 3x

X =12

Total girls in the class is 12

Shortcuts:

3’s = 18

1’s = 6

Total number of girls in the class = 2’s = 12

2.Answer a

Page 102 Follow us: Official Site, Telegram, Facebook, Instagram, Instamojo
Let the total number of subjects be x,

Average = Sum/n

(85x + 20 + 12)/x = 89

85x + 32 = 89x

4x = 32

X=8

Total number of subjects = 8

3.Answer d

Let the number of clerks in the company be x,

800*30 + 240*x = 300(30 + x)

24000 + 240x = 9000 + 300x

15000 = 60x

X = 15000/60 = 250

Total number of clerks in the company = 250

Shortcuts:

Page 103 Follow us: Official Site, Telegram, Facebook, Instagram, Instamojo
= > 3: 25

3’s = 30

1’s = 10

Total number of clerks in the company = 25’s = 250

4.Answer b

Total age of 25 students = 25*20 = 500

Total age of 25 students and teacher = 26*21 = 546

Age of the teacher= 546 – 500 = 46 years

Shortcut:

N Average

25 20

26 21

If both n and Average increased by 1, then the age of the teacher is,

= > (25 + 21) or (26 + 20) = 46 years

5.Answer c

Correct Average = [(20*105) – (75 + 91) + (68 + 108)]/20

= > 105.5 cm

6.Answer a

Let the number of passed students be x,

Page 104 Follow us: Official Site, Telegram, Facebook, Instagram, Instamojo
45*12 = 14x + (45 – x)*5

540 = 14x + 225 – 5x

540 – 225 = 9x

9x = 315

X = 35

Total number of passed students = 35

Shortcut:

= > 7: 2

9’s = 45

1’s = 5

Total number of passed students = 7’s = 35

7.Answer d

Average = 2.2, Number of people (n) = 20

Average = Total sum/n

Total weight increased = 2.2*20 = 44 kg

Weight of new man = 44 + 53 = 97 kg

Page 105 Follow us: Official Site, Telegram, Facebook, Instagram, Instamojo
8.Answer b

Let the number of newly joined students be x,

According to the question,

52*5 = (52 + x)*4

260 = 208 + 4x

52 = 4x

X = (52/4) = 13

9.Answer c

Total weight of 50 students = 50*60

New weight = (50*60) – 74 – 66 + 66 + 54 = (50*60) – 20

New average = (3000 – 20)/50 = 59.60 kg

10.Answer a

Sum of 5 numbers = 120*5 = 600

Sum of first two numbers = 125*2 = 250

Sum of last two numbers = 130*2 = 260

Third number = 600 – (250 + 260)

= > 600 – 510 = 90

Page 106 Follow us: Official Site, Telegram, Facebook, Instagram, Instamojo
Profit and Loss

1). The selling price of a mobile phone including the sales tax is Rs. 16302.
The rate of sales tax is 10%. If the shopkeeper has made a profit of 14%,
the cost price of the mobile phone is?

a. 15020
b. 14790
c. 13000
d. 15500
e. 12000

2). After allowing a discount of 15% on the marked price of a pen, it is sold
for Rs. 119. Then find its marked price?

a. 160
b. 185
c. 140
d. 125
e. 118

3). The profit earned by selling a watch for Rs. 2400 is equal to the loss
incurred by selling the same watch for Rs. 1680. At what price should the
watch be sold to make 25% profit?

a. Rs. 2275
b. Rs. 2100
c. Rs. 2550
d. Rs. 2400
e. None of these

4). The cost of the towel is Rs. 50 for manufacturing, and it sold the product
to a dealer for Rs. 60, who in turn sold it to a shopkeeper for Rs. 80, who
sold it to a customer for Rs. 100. What is the percentage of profit for the
company, who made the highest profit on selling the product?

Page 107 Follow us: Official Site, Telegram, Facebook, Instagram, Instamojo
a. 20 1/3%, Company
b. 33 1/3%, Dealer
c. 20 1/3%, Dealer
d. 25%, Shopkeeper
e. None of these

5). The marked price of the LED TV is 20% above its cost price. He gives
two successive discounts to customers of 10% and 25% respectively. As a
result he incurred a loss of Rs.1900. At what price (in rupees) did he sell the
LED TV to the customer?

a. 9600
b. 8100
c. 7800
d. 7500
e. 6900

6). A shop keeper procures 50 Novels for Rs. 4300 and sells them at a profit
equal to the selling price of 7 books. What is the selling price of one dozen
books, if the price of each book is same?

a. Rs.1300
b. Rs.1100
c. Rs.800
d. Rs.1200
e. Rs.1000

7). The teddy is marked 15% above the cost price. When the selling price of
an article is increased by 25%, the profit gets increased Rs. 110. If the
marked price of the article is Rs. 575, then find original selling price.

a. 560
b. 440
c. 390
d. 280
e. 620

Page 108 Follow us: Official Site, Telegram, Facebook, Instagram, Instamojo
8). The company wishes to give 4% commission on the marked price of a
product but also wants to earn a profit of 10%. If the cost price of the
product is Rs. 120, then find the marked price is?

a. 146.25
b. 137.50
c. 180.75
d. 120.75
e. 168.25

9). A shopkeeper marks a Branded Hard Disk 65% above the cost price
and allows two successive discounts of 8% and 10% respectively. If he
spent Rs. 8 on transport and the marked price is Rs. 250, what will be his
profit percentage? (Approximately)

a. 29%
b. 32%
c. 43%
d. 56%
e. 19%

10). A shop keeper sells his items at 18% discount and incurs a loss of 3%.
How much % above cost price does he mark up his items?

a. 56%
b. 34%
c. 26%
d. 43%
e. None of these

Answers:

1). Ans: C)

110 % of SP = 16302

Page 109 Follow us: Official Site, Telegram, Facebook, Instagram, Instamojo
SP of a mobile = (16302 / 110) * 100 = 14820

CP = (100 / 114) * 14820 = 13000

Cost price of the mobile = Rs. 13000

2). Ans: C)

Suppose, marked price of the article be Rs. X

Then, 85% of x = 119

X × (85 / 100) = 119

X = 119*(20/17) = 140

x = Rs. 140

3). Ans: C)

According to the question,

Profit = Loss

SP1 – CP = CP – SP2

2400 – CP = CP – 1680

2400 + 1680 = 2CP

CP = 4080/2 = 2040

SP = 2040 * (125 /100) = 2040 * (5/4) = Rs. 2550

(Or)

CP = (2400+1680)/2 = 2040

Page 110 Follow us: Official Site, Telegram, Facebook, Instagram, Instamojo
Required SP = 2040*125/100 = Rs.2550

4). Ans: B)

Company Profit % = (60-50)/50*100=100/5 =20 %

Then Dealers Profit % = (80-60)/60*100=100/3 =33 1/3 %

Then Shopkeeper Profit % = (100-80)/80*100 =25 %

Among the three, Dealer gets highest profit %

5).Ans: B)

To take CP = 100

Marked price = 120

120 * 90/100 * 75/100 = 81

Loss = 100 – 81 = 19

19 % of CP = 1900

CP = (100 /19) * 1900 = Rs. 10000

SP = 10000- 1900 = 8100

6). Ans: D

CP of 1 novel = Rs. 4300/50 = Rs.86

Now SP of 50 books = CP of 50 Novels + SP of 7 Novels

=> SP of 43 Novels = 4300

SP of 1 Novel = 4300/43 = Rs.100

Page 111 Follow us: Official Site, Telegram, Facebook, Instagram, Instamojo
Therefore,

Required SP of 1 dozen books = Rs. 1200

7). Ans: B)

Marked Price = Rs. 575

CP = 575 * (100/115) = 500

Profit = Selling Price – 500

And given that, when SP increased by 25%

Let the original selling price be Rs. x,

125/100 * SP – 500 = Profit + 110

5/4 x – 500 = (SP – CP) + 110

5x/4 – 500 = x – 500 + 110

5x/4 – x = 110

(5x – 4x) / 4 = 110

x = 440

Selling price = Rs. 440

8).Ans: B)

CP = Rs. 120

Then SP = 110/100 * 120 = 132

Let the marked price be x,

Page 112 Follow us: Official Site, Telegram, Facebook, Instagram, Instamojo
So, SP = 96 % of Marked price

132 = (96/100)* x

X = 132*(100/96) = 137.5

Required marked price = Rs. 137.5

9). Ans: A)

SP of Hard Disk = 250 * 92/100 * 90/100 = 207

Expense on transport = Rs. 8

CP = 250/165*100 = 152 (Approximately)

Profit % = [(207- (152+8))/(152+8)]*100 = 29%(Approximately)

10). Ans: B)

Let M.P = 100, then S.P. = (82 / 100) × 100 = 82

C.P = 82 × (100 / 97) = 84.5

Mark up above Cost Price = [(100 – 84.5) / 84.5] × 100 = 18.34%

Page 113 Follow us: Official Site, Telegram, Facebook, Instagram, Instamojo
Time and Work

1) A piece of work has to be completed in 60 days, a number of men are


employed but it is found that only half of the work is done in 40 days, then
an additional 30 men were joined to complete the work on time. Initially how
many men are there to work?

a) 30 men

b) 26 men

c) 24 men

d) 34 men

e) None of these

2) The ratio of efficiency of Ajay and Sneha is 6: 5. The ratio of number of


days taken by Prabha to Sneha is 3: 2. Ajay takes 3 days less than Sneha,
when Ajay and Sneha complete the work individually. Prabha and Sneha
started the work and left after 3 days. The number of days taken by Ajay to
finish the remaining work is?

a) 12 days

b) 9 ¾ days

c) 10 5/6 days

d) 11 2/5 days

e) None of these

3) Ragu and Rajesh can separately do a piece of work in 12 and 15 days


respectively. They worked together for 5 days, after which Rajesh was
replaced by Rohit. If the work was finished in next 2 days, then the number
of days in which Rohit alone could do the work?

Page 114 Follow us: Official Site, Telegram, Facebook, Instagram, Instamojo
a) 20 days

b) 24 days

c) 32 days

d) 28 days

e) None of these

4) A and B undertake to complete a piece of work for Rs. 3240. A can do it


in 12 days, B can do it in 18 days and with the help of C they complete the
work in 6 days. Find the share of C?

a) Rs. 620

b) Rs. 480

c) Rs. 500

d) Rs. 540

e) None of these

5) A and B can do a piece of work in 10 and 15 days respectively. They began


the work together but A leaves after some days and B completed the
remaining work in 8 days. Number of days after which A left the job?

a) 2 4/5 days

b) 4 ½ days

c) 3 ¾ days

d) 5 1/3 days

e) None of these

Page 115 Follow us: Official Site, Telegram, Facebook, Instagram, Instamojo
6) P, Q and R can complete the whole work in 20 days. P starts the work and
works for ‘x’ days while Q and R complete the remaining 2/5 of the work in
14 days then find the value of x?

a) 24 days

b) 28 days

c) 32 days

d) 20 days

e) None of these

7) P takes 8 days to complete 2/3 of a work, Q takes 3 days to complete 1/7 of


the same work and R takes 8 days to complete 4/5 of the same work. If they
work for 3 days together then Q and R leaves the work. Find the number of
days P will take to complete the remaining work?

a) 117/28 days

b) 95/23 days

c) 156/25 days

d) 129/35 days

e) None of these

8) 20 men can complete a piece of work in 16 days. After 5 days from the
start of the work, some men left. If the remaining work was completed by
the remaining men 18(1/3) days, how many men left after 5 days from the
start of the work?

a) 10 men

b) 9 men

Page 116 Follow us: Official Site, Telegram, Facebook, Instagram, Instamojo
c) 8 men

d) 6 men

e) None of these

9) 20 men can complete a piece of work in 25 days. They started the work
and after 5 days, 5 more men joined them. In how many days will the work
be completed?

a) 21 days

b) 19 days

c) 16 days

d) 24 days

e) None of these

10) Q can do a piece of work in 14 days. Q is 50 percent more efficient than


P. In how many days half the work is completed when both are working
simultaneously?

a) 5 3/7 days

b) 3 2/3 days

c) 4 ¾ days

d) 4 1/5 days

e) None of these

Answers:

1) Answer: a)

Page 117 Follow us: Official Site, Telegram, Facebook, Instagram, Instamojo
Let initially the no of men be x,

A piece of work has to be completed in 60 days

According to the question,

Men days work

X 40 (1/2)

(x + 30) 20 (1/2)

Work = men * days

= > 40x/(1/2) = (x + 30)*20/(1/2)

= > 40x = (x + 30)*20

= > 40x = 20x + 600

= > 20x = 600

= > x = 30 men

2) Answer: c)

The ratio of efficiency of Ajay and Sneha = 6: 5

The ratio of number of days taken by Ajay and Sneha = 5: 6

The ratio of number of days taken by Prabha and Sneha = 3: 2

Ratio of number of days taken by Ajay: Sneha: Prabha = 5: 6: 9

According to the question,

= > Sneha – Ajay = 8 days

Page 118 Follow us: Official Site, Telegram, Facebook, Instagram, Instamojo
= > 6’s – 5’s = 3

= > 1’s = 3

Number of days taken to finish the whole work,

= > Ajay = 15 days, Sneha = 18 days, Prabha = 27 days

Work done by Prabha and Sneha in one day,

= > (1/18) + (1/27) = 45/(18*27) = 5/54

Prabha and Sneha’s two day work

= > (5/54)*3 = 5/18

Rest of the work = 13/18

The number of days taken by Ajay to finish the remaining work is,

Number of days = (13/18)*15 = 65/6 = 10 5/6 days

3) Answer: b)

Ragu and Rajesh worked together

1/12 + 1/15 = (12 + 15)/(12*15) = 3/20

Ragu and Rajesh’s 5 days work = (3/20)*5 = (3/4)

Remaining work 1/4 done by Ragu and Rohit

Ragu and Rohit finished it in 4 days

(1/4)*(Ragu + Rohit)’s whole work = 2

(Ragu + Rohit)’s whole work = 8

Page 119 Follow us: Official Site, Telegram, Facebook, Instagram, Instamojo
Rohit’s one day work = (1/8) – (1/12) = 1/24

Rohit alone can complete the work in 24 days

4) Answer: d)

1/12 + 1/18 + 1/C = 1/6

1/C = (1/6) – (1/12 + 1/18)

(1/C) = (1/6) – (5/36) = 1/36

We get, C = 36 days

Now efficiency of A, B and C are in the ratio of = 1/12: 1/18: 1/36 = 3: 2: 1

6’s = 3240

1’s = 540

The share of C = Rs. 540

5) Answer: a)

(1/10 + 1/15)*x + 8/15 = 1

(1/10 + 1/15)*x = 1 – (8/15)

(1/10 + 1/15)*x = 7/15

x/6 = 7/15

x = 14/5

X = 2 4/5 days

6) Answer: b)

Page 120 Follow us: Official Site, Telegram, Facebook, Instagram, Instamojo
Time required by Q and R to complete the whole work

= 5*14/2= 35 days

Time taken by P, Q and R= 20 days

Total units of work= 140 units

Q and R one day work= 4 units

P, Q and R one day work= 7 units

P’s one day work= 7-4= 3 units

Units of work done by P = 3*140/5= 84 units

Required value of x = 84/3 = 28 days

7) Answer: d)

Time taken by P= 8*3/2= 12 days

Q= 7*3= 21 days

R= 8*5/4= 10 days

Total units of work = 420 units

P’s one day work= 35 units

Q’s one day work= 20 units

R’s one day work= 42 units

Work done in 3 days= 97*3= 291 units

Time required by P to complete the remaining work

Page 121 Follow us: Official Site, Telegram, Facebook, Instagram, Instamojo
= > (420-291)/35 = 129/35 days

8) Answer: c)

Total units of work= 20*16= 320 units

Work done in 5 days= 20*5= 100 units

Remaining work= 320 -100= 220 units

Let the number of men left after 5 days be x

So,

20-x men completed the remaining 220 units in 55/3 days

So,

(20-x)*55/3= 220

1100- 55x= 660

55x= 440

X= 8 men

9) Answer: a)

Total work = 20*25 = 500 work

5 days work = 20*5 = 100 work

Remaining work = 500 – 100 = 400

According to the question,

= > 400/25 = 16 days

Page 122 Follow us: Official Site, Telegram, Facebook, Instagram, Instamojo
The total no of days = 16 + 5 = 21 days

The work gets completed in 21 days

10) Answer: d)

Efficiency ratio of Q: P = 150: 100 = 3: 2

Days ratio of Q: P = 2: 3

P can do a piece of work in 14 days

2’s = 14 = > 1’s = 7

So P can complete the work in 21 days

(1/14 + 1/21)*x = ½

[35/(14*21)]*x = 1/2
X = 21/5 = 4 1/5 days

Page 123 Follow us: Official Site, Telegram, Facebook, Instagram, Instamojo
Data Sufficiency

Directions (1-10): For each of the following questions two statements are
given. Use the data of those statements and then determine which of the
following statements is necessary to answer the question.

a) If the data in statement I alone is sufficient to answer the question.


b)If the data in statement II alone is sufficient to answer the question.
c) If the data either in statement I alone or statement II alone are sufficient to
answer the question.
d) If the data given in both I and II together are not sufficient to answer the
question.
e) If the data in both the statements I and II together are necessary to answer the
question

1) A sum of money Rs 15000 is to be distributed among Ponraj, Dhinesh


and Livinson. What will be the share of Dhinesh?

Statement I: Ponraj’s share of is 1.5 times Dhinesh’s share and Livinson’s


share is half the share of Ponraj and Dhinesh together.

Statement II: The difference between the share of Dhinesh and Livinson is half
of the share of the Ponraj.

a) a

b) b

c) c

d) d

e) e

2) What is the ratio of the efficiency of a man and a woman?

Statement I: 20 men are as efficient as 16 women

Page 124 Follow us: Official Site, Telegram, Facebook, Instagram, Instamojo
Statement II: 30 women can complete half of the work done by 40 men in same
time)

a) a

b) b

c) c

d) d

e) e

3) Find the length of train A

Statement I: Ratio of the speeds of Train A and Train B is 2:3.

Statement II: Length of train B is 30% more than that of train

a) a

b) b

c) c

d) d

e) e

4) What is the speed of the stream?

Statement I: The boat goes 20km more in downstream than in upstream in 4


hours

Statement II: The boat goes 63 km downstream in 3 hours.

a) a

Page 125 Follow us: Official Site, Telegram, Facebook, Instagram, Instamojo
b) b

c) c

d) d

e) e

5) What is the age of Manohar?

Statement I: Five years ago, Kathir was as old as Manohar is at present.

Statement II: The average age of Manohar and Prabhu is 26 years, and Prabhu’
present age is two times of Kathir’s present age)

a) a

b) b

c) c

d) d

e) e

6) What is the total of the present ages of P and Q?

I) The ratio between the P and Q age is 7: 9.

II) If the difference between the present ages of Q and P’s age after 4 years is 2.

a) a

b) b

c) c

d) d

Page 126 Follow us: Official Site, Telegram, Facebook, Instagram, Instamojo
e) e

7) What is the speed of man in still water?

I) A man rows to a place 40 km distance and back in a total of 18 hours. He finds


that he can row 5 km with the stream in the same time as 4 km against the stream.

II) A man can row 48km upstreamand 56 km downstream in 12 Also, he can


row 54 km upstream and 70 km downstream in 14 hrs.

a) a

b) b

c) c

d) d

e) e

8) What is the average age of the 2 new boys?

I) In a group of 8 boys, 2 boys aged at 21 and 23 were replaced two new boys.
Due to this the average age of the group increased by 2 years.

II) The average age of the group having 3 members is 84.

a) a

b) b

c) c

d) d

e) e

9) How much did M receive?

Page 127 Follow us: Official Site, Telegram, Facebook, Instagram, Instamojo
I) 9750 are divided among K,L and M so that K may receive half as much as L
and M together receive.

II) L receive one fourth of what K and M together receive.

a) a

b) b

c) c

d) d

e) e

10) What is the area of the isosceles triangle?

Statement I: The Height of the triangle is half the length of the rectangle whose
area is 144cm2

Statement II: The base of the triangle is 13 meters and the height of the triangle
is 4 meters.

a) a

b) b

c) c

d) d

e) e

Answers:

Directions (1-10):

1) Answer: A

Page 128 Follow us: Official Site, Telegram, Facebook, Instagram, Instamojo
From Statement I,

Ponraj: Dhinesh= 3:2,

Ponraj: Dhinesh: Livinson=3:2:5/2

= 6:4:5

Then Share of Dhinesh= (4/15)*15000

=Rs 4000

From Statement II, We cannot find the share of Dhinesh.

Thus Statement I is sufficient to answer the question.

2) Answer: C

Let the efficiency of a man and a woman be x and y respectively.

From Statement I, we have

20x=16y

(x/y)= 16/20

= 4:5

From statement II we have,

30y= ½*(40x)

x/y=30:20

= 3:2

Thus Either Statement I or statement II is sufficient to answer the question.

Page 129 Follow us: Official Site, Telegram, Facebook, Instagram, Instamojo
3) Answer: D

Even though combining both statements, we cannot find the length of the train.

Thus Neither Statement I nor statement II is sufficient to answer the


question.

4) Answer: A

Statement I:

Let the downstream and upstream speeds be x km/hr and y km/hr respectively.

From statement I,

4(x-y) =20

x-y = 5km/hr

Speed of the stream=(x-y)/2

= 5/2

= 2.5km/hr

From statement II,

(63/x)= 3

x= 21 km/hr

Thus Statement I is sufficient to answer the question.

5) Answer: E

From statement I,

Let the present age of Manohar be x.

Page 130 Follow us: Official Site, Telegram, Facebook, Instagram, Instamojo
Then present age of Kathir= (x+5)

From statement I and Statement II,

Prabhu’s present age= 2(x+5)

2(x+5)+x=2*26

2x+10+x= 52

3x+10=52

x=14

Thus Both Statements I and II are necessary to answer the question.

6) Answer: e

Let age of B is 9x and that of A is 7x. So


9x – (7x +4) = 2, x = 3
So sum will be = 27 + 21 = 48

Both the statements are necessary to answer the question

7) Answer: c

I) Suppose he moves 5km downstream in x hours


Then, downstream speed a= 5/x km/hr
Speed upstream speed b = 4/x km/hr
40 / (5 /x) + 40 / (4/x) = 18
8x + 10x = 18
x=1
a = 5 km/hr, b = 4 km/hr
speed of boat = ½ (5 + 4 ) = 9/2 km/hr

II) Let upstream speed = x km/hr, downstream speed = y km/hr


So 48/x + 56/y = 12
And 54/x + 70/y = 14

Page 131 Follow us: Official Site, Telegram, Facebook, Instagram, Instamojo
Put 1/x = u, 1/y = v
So equations are 48u + 56v = 12 and 54u + 70v = 14
Solve the equations, u = 1/6, v = 1/14
So upstream speed = 6 km/hr, downstream speed = 14 km/hr
Speed of boat in still water = 1/2*(6+14)=10kmph

Either statement I alone or Statement II alone is sufficient to answer the


question

8) Answer: a

Average of 8 boys increased by 2, this means the total age of boys increased by
8*2 = 16 yrs
So sum of ages of two new boys = 21+23+16 = 60
Average of these = 60/2=30

Statement I alone is sufficient to answer the question

9) Answer: e

K =(L+M)/2

2K = L+ M

2K –L =M — (1)
L =( K+M)/4

4L = K+M

4L – K = M —- (2)

Equate (1) and (2), we get

2K –L = 4L – K

3K = 5L

K/L = 5/3

Page 132 Follow us: Official Site, Telegram, Facebook, Instagram, Instamojo
Substitute the value in equation (1) we get, (10x-3x) = M

7x =M

Ratio of K, L and M = 5: 3: 7

Share of M = 7/15 *9750 = 4550

Both the statements are necessary together to answer the question

10) Answer: b

We know that,

Area of the triangle= ½* height * base

Hence Statement II alone is sufficient to answer the question.

Direction (Q.11-15): Each of the questions below consists of a question and


two statements numbered I and II given below it. You have to decide whether
the data given in the statements are sufficient to answer the question. Read both
statement and choose the most appropriate option.
a) The data even in both statements I and II together are not sufficient to answer
the question.
b) The data in both statements I and II together are necessary to answer the
question.
c) The data in statement II alone are sufficient to answer the question, while the
data in statement I alone are not sufficient to answer the question.
d) The data either in statement I alone or in statement II alone are sufficient to the
answer the question.
e) The data in statement I alone are sufficient to answer the question while the
data in statement II alone are not sufficient to answer the question.

11). What is the present age of C?


I. At present A is 8 years elder to B. C is 2 years younger than A.
II. The ratio between present age of B and that of C is 3:4.

Page 133 Follow us: Official Site, Telegram, Facebook, Instagram, Instamojo
12). What was the annual salary of the Mrs.Santhi in 2016?
I. Out of the annual salary in 2015, Mrs.Santhi invested 19% in stocks and 21%
in mutual funds and 45% of her annual salary on household expenses and save
an amount of Rs. 3 lakh at end of the year.
II. Annual salary of Mrs.Santhi increased by 8% from 2015 to 2016 and this got
a raise of Rs.16,000 in her annual salary.

13). What is the speed of the train?


I. The train crosses a vertical pole in 6 seconds.
II. The train crosses a 120m long platform in 10 seconds while travelling at the
same speed.

14). What is the present age of Anand’s mother?


I. The respective ratio between the present ages of Anand and his mother is
6:11.
II. Anand’s mother is 5 years younger than his father.

15). What is the number?


I. 25% of that number is 40% of 280
II. Three- fourths of that number is less than that number by 112.

Ans (11-15)
11). Answer: b)
From statement I& II,
Let present age of ‘A’ be x
∴ B’s present age is (x-8)
C’s present age is (x-2)
(x-8)/(x-2) = 3/4
4x-32 = 3x-6
x =26
Hence, the present age of C is 26-2=24 years
Hence, both statements I& II are sufficient to answer.

12). Answer: c)
From statement-I,

Page 134 Follow us: Official Site, Telegram, Facebook, Instagram, Instamojo
Santhi’s annual salary be ‘x’
Total expenditure= (19+21+45)= 85
∴ 100 – 85 = 15% of salary = 3 lakh
∴ x = (300000 × 100)/15 = 20 lakh
From statement-II,
8% of salary is16,000
∴ x = 16,000 × 100 /8 = 200,000
Hence, Mrs.santhi’s annual salary in 2016 = 200,000+16,000= Rs.216,000.
Hence, statement-II is only sufficient to answer.

13). Answer: b)
From statement I& II,
Let the length of train be x,
Length of the train = (x+120)/10 = x/6
= (x+120)/5 = x/3
–>5x = 3x +360
∴ x = 360/2 = 180m
∴ Speed of the train = x/6 = 180/6 = 30m/sec
Hence, both statements I& II are sufficient to answer.

14). Answer: a)
From both the statements no result is obtained. Hence, both statements I and II
together are not sufficient to answer the question.

15). Answer: d)
From statement-I,
Required number = 280 × 40/100 × 100/25 = 448
From statement-II,
Let the number be x,
Then, 3x/4 = x – 112
3x = 4x – 448
∴ x = 448
Hence, either statement- I or statement-II is sufficient to answer the question.

Page 135 Follow us: Official Site, Telegram, Facebook, Instagram, Instamojo
Simplification / Approximation

Approximation
Directions (1-5) What approximate value should come in place of the question
mark (?) in the following questions ?
(Note: You are not expected to calculate the exact value).
1).√(1230) + √(4230) = ?
a) 50
b) 200
c) 100
d) 150
e) 250

2).7 (3/11) × 626 – 7(1/3) = ?


a) 4445
b) 4545
c) 4495
d) 4595
e) 4515

3).49% of 5051 – (3/7) of 999 = ?


a) 2145
b) 2045
c) 1945
d) 1905
e) 2165

4).4329 / 19 + 6464 / 13 = ?
a) 725
b) 625
c) 925
d) 525
e) 825

Page 136 Follow us: Official Site, Telegram, Facebook, Instagram, Instamojo
5).(4.012)3 + (29.997)2 = ?
a) 1025
b) 964
c) 745
d) 765
e) 805

Directions (6-10) What Approximate value will come in place of the question
mark (?) in the following questions ?(You are not expected to calculate the
exact value).
6).6575 / 17.98 × 42.03 / 6.87 = ?
a) 2190
b) 2280
c) 2090
d) 2150
e) None of these

7).12.002 × 15.005 – 8.895 × 6.965 = ?


a) 130
b) 117
c) 105
d) 110
e) None of these

8).12.664 × 22.009 × 17.932 = ?


a) 5100
b) 5200
c) 5148
d) 5199
e) None of these

9).16.978 + 27.007 + 36.984 – 12.969 – 9.003 = ?

Page 137 Follow us: Official Site, Telegram, Facebook, Instagram, Instamojo
a) 50
b) 51
c) 52
d) 59
e) 65

10).18% of 602 + 27.8% of 450 = ?


a) 234
b) 260
c) 225
d) 220
e) 250

Answer:
1). c) 2). b) 3). b) 4). a) 5). b) 6). a) 7). b) 8). c) 9). d) 10). a)
Solution:

1).√(1230) + √(4230) = √(1225) + √(4225) = 35 + 65 = 100


Answer : c)

2).7(3/11) × 626 – 7(1/3)à[(80 ×626) / 11] – 7(1/3) = 4552 – 7 = 4545


Answer : b)

3).[(49 × 5051) / 100] – (3/7) × 999 = 2475 – 430 = 2045


Answer : a)

4).(4329 / 19) + (6464 / 13) = 228 + 497 = 725


Answer : b)

5).(4)3 + (30)2 = 64 + 900 = 964


Answer : b)

Page 138 Follow us: Official Site, Telegram, Facebook, Instagram, Instamojo
6).6575 / 18 × 42 / 7 = (6575 / 18) × (42 / 7) = 365 × 6 = 2190
Answer : a)

7).12 × 15 – 9 × 7 = 180 – 63 = 117


Answer : b)

8).13 × 22 × 18 = 5148
Answer : c)

9).17 + 27 + 37 – 13 – 9 = 81 – 22 = 59
Answer : d)

10).[(18 × 600) / 1000] + [(28 × 450) / 100] = 108 + 126 = 234


Answer : a)

Simplification-
1035 ÷ [(3/4) of (71 + 65) – 15 ¾] = ?

a. 12
b. 24
c. 18
d. 26
e. None of these

Answer a

Explanation:

1035 ÷ [(3/4)*136 – 63/4] = x

Page 139 Follow us: Official Site, Telegram, Facebook, Instagram, Instamojo
X = 1035 ÷ [102 – (63/4)]

X = 1035 ÷ [(408 – 63)/4]

X = 1035 ÷ (345/4)

X = 1035 *4/345

X = 12

(7 × 7)3 ÷ (49 × 7)3 × (2401)2 = 7?

a. 7
b. 5
c. 6
d. 8
e. None of these

Answer b

Explanation:

(7 × 7)3 ÷ (49 × 7)3 × (2401)2 = 7x

(72)3 ÷ (72 × 7)3 × (74)2 = 7x

76 ÷ 7 9 × 7 8 = 7 x

76 – 9 + 8 = 7 x

75 = 7 x

X=5

√((27 ÷ 5 ×?) ÷ 15) = 5.4 ÷ 6 +0.3

Page 140 Follow us: Official Site, Telegram, Facebook, Instagram, Instamojo
a. 12
b. 4
c. 9
d. 15
e. None of these

Answer b

Explanation:

√((27 /5 )×(x/15)) =0.9+0.3

(3/5) √ x = 1.2

√ x = 6/3 =2

X= 4

5 ¾ – 4 1/8 + 7 ½ – 3 3/8 = ?

a. 4 3/7
b. 6 5/6
c. 7½
d. 5¾
e. None of these

Answer d

Explanation:

5 ¾ – 4 1/8 + 7 ½ – 3 3/8 = x

X = (5 – 4 + 7 – 3) (3/4 – 1/8 + ½ – 3/8)

X = 5 [(6 – 1 + 4 – 3)/8]

Page 141 Follow us: Official Site, Telegram, Facebook, Instagram, Instamojo
X = 5 (6/8) = 5 (3/4)

363 × 40961/2 × 72 × 18 ÷ (93 × 722) = 4?

a. 8
b. 12
c. 7
d. 5
e. None of these

Answer d

Explanation:

(36*36*36*64*72*18)/(9*9*9*72*72) = 4x

4*4*43 = 4x

45 = 4 x

X=5

(2 7/9) ÷ (5 1/2) × (693/350) = (1/3) of (5/7) of (147/175) of (?)

a. 12
b. 26
c. 5
d. 18
e. 9

Answer c

Explanation:

Page 142 Follow us: Official Site, Telegram, Facebook, Instagram, Instamojo
(25/9)*(2/11)*(693/350) = (1/3)*(5/7)*(147/175)*x

1 = (x/5)

X=5

(913 + 329 + 522 + 343) ÷ (18 + 24 – ? + 18) = 43

a. 23
b. 11
c. 7
d. 29
e. 15

Answer b

Explanation:

(913 + 329 + 522 + 343) ÷ (18 + 24 – x + 18) = 43

2107 ÷ (60 –x) = 43

(2107/43) = 60 – x

49 = 60 – x

X = 60 – 49 = 11

(3/7) of ? – (2/5) of (13/16) of 2160 = 12 % of 900

a. 2150
b. 1540
c. 1960

Page 143 Follow us: Official Site, Telegram, Facebook, Instagram, Instamojo
d. 1890
e. 2370

Answer d

Explanation:

(3/7)*x – (2/5)*(13/16)*2160 = (12/100)*900

(3x/7) – 702 = 108

(3x/7) = 108 + 702

(3x/7) = 810

X = (810*7/3) = 1890

12 ¾ % of 2400 + 23 ½ % of 800 + 452 = 1125 ÷ 25 × 4 + ?

a. 658
b. 532
c. 776
d. 614
e. 590

Answer c

Explanation:

(51/400)*2400 + (47/200)*800 + 2025 = (1125/25) + 4x

306 + 818 + 2025 – 45 = 4x

3104 = 4x

X = 3104/4 = 776

Page 144 Follow us: Official Site, Telegram, Facebook, Instagram, Instamojo
∛157464 + √8836 – 45 % of 800 =? + (1188 ÷ 9) – 362

a. 952
b. 816
c. 878
d. 924
e. None of these

Answer a

Explanation:

54 + 94 – (45/100)*800 = x + (1188/9) – 1296

54 + 94 – 360 + 1296 = x + 132

54 + 94 – 360 + 1296 – 132 = x

X = 952

(4/5) of 11325 + 56 % of 1750 = (?)2 + 436

a. 116
b. 72
c. 98
d. 84
e. 102

Answer c

Explanation:

(4/5)*11325 + (56/100)*1750 = x2 + 436

9060 + 980 – 436 = x2

Page 145 Follow us: Official Site, Telegram, Facebook, Instagram, Instamojo
10040 – 436 = x2

9604 = x2

X = 98

(4/5) of 11325 + 56 % of 1750 = (?)2 + 436

a. 116
b. 72
c. 98
d. 84
e. 102

Answer c

Explanation:

(4/5)*11325 + (56/100)*1750 = x2 + 436

9060 + 980 – 436 = x2

10040 – 436 = x2

9604 = x2

X = 98

1.8 × 625 + 5.6 ÷ 0.7 + 415 × 11.8 = (?)2 – 211

a. 79
b. 85
c. 92

Page 146 Follow us: Official Site, Telegram, Facebook, Instagram, Instamojo
d. 69
e. 61

Answer a

Explanation:

1.8 × 625 + 5.6 ÷ 0.7 + 415 × 11.8 = (?)2 – 211

1125 + 8 + 4897 = x2 – 211

6241 = x2

X = 79

18 ¼ % of 17200 + 33 2/3 % of 1860 + 37 2/3 % of 19680 = ?

a. 11178
b. 13456
c. 9824
d. 10242
e. 8454

Answer a

Explanation:

(73/400)*17200 + (101/300)*1860 + (113/300)*19680= x

X = 3139 + 626.2 + 7412.8

X = 11178

586 × 9234 ÷ 48 % of 950 = 36 × 750 ÷ 4 + ? + 4500

Page 147 Follow us: Official Site, Telegram, Facebook, Instagram, Instamojo
a. 25
b. 728
c. 5
d. 75
e. 474

Answer c

Explanation:

586 × [9234 ÷ ((48/100)*950)] = 36 × (750/4) + x + 4500

586*(9234/456) = 6750 + x + 4500

11866.5 – 11250

X = 616.5

1323 ÷ (27 % of 700) = 35 × 7 3/5 of (? ÷ 190)

a. 7
b. 9
c. 4
d. 5
e. 8

Answer d

Explanation:

1323 ÷ [(27/100)*700] = 35 × (38/5)*(x/190)

1323 ÷ 189 = 35 × (38/5)*(x/190)

(7*5*190)/(35*38) = x

Page 148 Follow us: Official Site, Telegram, Facebook, Instagram, Instamojo
X=5

(28 × 9 + 54 × 3 + 12 × 11) ÷ (142 – √961 + 17) = ?

a. 6
b. 3
c. 5
d. 8
e. 7

Answer b

Explanation:

(28 × 9 + 54 × 3 + 12 × 11) ÷ (142 – √961 + 17) = x

X = (252 + 162 + 132) ÷ (196 – 31 + 17)

X = 546/182

X=3

32450 ÷ 11 + (4/9) of 11520 + 48 % of 7500 = ?2 – 430

a. 150
b. 178
c. 110
d. 92
e. None of these

Answer c

Explanation:

Page 149 Follow us: Official Site, Telegram, Facebook, Instagram, Instamojo
(32450/11) + (4/9)*11520 + (48/100)*7500 = x2 – 430

2950 + 5120 + 3600 + 430 = x2

X2 = 12100

X = 110

√13225 + ∛205379 + 52 % of 700 = 408 + ?

a. 156
b. 218
c. 189
d. 130
e. None of these

Answer d

Explanation:

√13225 + ∛205379 + 52 % of 700 = 408 + x

115 + 59 + (52/100)*700 – 408 = x

115 + 59 + 364 – 408 = x

X = 130

492 ÷ 14 × 8 + 582 = 4 × ? – 15 % of 2480

a. 1489
b. 1277
c. 1645

Page 150 Follow us: Official Site, Telegram, Facebook, Instagram, Instamojo
d. 1823
e. None of these

Answer b

Explanation:

(49*49*8)/14 + 582 = 4x – (15/100)*2480

1372 + 3364 = 4x – 372

1372 + 3364 + 372 = 4x

5108 = 4x

X = 5108/4 = 1277

Page 151 Follow us: Official Site, Telegram, Facebook, Instagram, Instamojo
Quadratic Equation

Directions(1-10): in each of these questions numbered I and II are given.


You have to solve both the equations and

1). I.x2 + 12x + 32 = 0


II.y2 + 17y + 72 = 0

2). I.x2 + 13x + 42 = 0


II. y2 + 19y + 90 = 0

3). I.x2 – 15x + 56 = 0


II. y2– 23y + 132 = 0

4). I.x2 + 7x + 12 = 0
II. y2 + 6y + 8 = 0

5). I.x2 – 22x + 120 = 0


II. y2 – 26y + 168 = 0

6). I.x2+ x – 20 = 0
II. y2– y – 30 = 0

7). I.225x2 – 4 = 0
II. √225y + 2 = 0

8). I.(4/√x) + (7/√x) = √x


II. y2 – [ 11(5/2)/ √y ] = 0

9). I.x2 – 365 = 364


II. y – √324 = √81

10). I.3x2 + 8x + 4 = 0
II. 4y2 – 19x + 12 = 0

Answers:

Page 152 Follow us: Official Site, Telegram, Facebook, Instagram, Instamojo
1). d) 2). c) 3). a) 4). e) 5). b) 6). e) 7). e) 8). e) 9). b) 10). a)
Solutions:

1. I. x2 + 12x + 32 = 0
=> x2 8x + 4x + 32 = 0
=> x(x + 8) + 4(x + 8) = 0
(x + 8) (x + 4) = 0
:. X = -8, -4
II. y2 + 17y + 72 = 0
=> y2 9y + 8y + 72 = 0
=> y( y + 9) + 8(y + 9) = 0
=> (y + 9) ( y + 8) = 0
:. Y = -9, -8
Hence, x > y
Answer: d)

2. I. x2 + 13x + 42 = 0
=> x2 7x + 6x + 42 = 0
=> x(x + 7) + 6(x + 7) = 0
=> (x + 7) ( x + 6) = 0
:. X = -7, -6
II. y2 + 19y + 90 = 0
=> y2 10y + 9y + 90 = 0
=> y(y + 10) + 9(y + 10) = 0
=> (y + 10) ( y + 9) = 0
:. Y = -10, -9
Hence, x> y
Answer:c)

3. I. x2 – 15x + 56 = 0
=> x2 – 8x – 7x + 56 = 0
=> x(x – 8) – 7(x – 8) = 0
=> (x- 8) (x – 7) = 0
:. x = 8, 7
II. y2 – 23y + 132 = 0
=> y2 – 12y – 11y + 132 = 0
=> y(y – 12) – 11(y – 12) = 0

Page 153 Follow us: Official Site, Telegram, Facebook, Instagram, Instamojo
=> (y – 12) (y – 11) = 0
:. y = 12, 11
Hence, x< y
Answer: a)

4. I. x2 + 7x + 12 = 0
=> x2 + 4x + 3x + 12 = 0
=> x( x + 4) + 3( x + 4) = 0
=> (x + 4) (x + 3) = 0
:. x = -4, -3
II. y2 + 6y + 8 = 0
=> y2 + 4y + 2y + 8 = 0
=> y(y + 4) + 2(y + 4) = 0
=> (y + 4) (y + 2) = 0
:. x = -4, -2
Hence relationship cannot be established.
Answer: e)

5. I. x2 – 22x + 120 = 0
=> x2 – 10x – 12x + 120 = 0
=> x( x – 10) – 12( x – 10) = 0
=> (x – 10) (x – 12) = 0
:. x = 10, 12
II. y2 – 26y + 168 = 0
=> y – 14y – 12y + 168 = 0
=> y(y – 14) – 12(y – 14) = 0
=> (y – 14) (y – 12) = 0
:. y = 14,12
Hence, x < y
Answer: b)

6. I. x2 + x – 20 = 0
=> x2 5x – 4x – 20 = 0
x(x + 5) – 4(x + 5) = 0
x = -5, 4
II. y2 – y – 30 = 0
y2 + 6y – 5y – 30 = 0
y(y + 6) – 5(y + 6) = 0

Page 154 Follow us: Official Site, Telegram, Facebook, Instagram, Instamojo
y = -6, 5
Relationship cannot be established.
Answer: e)
7. I. 225x2 – 4 = 0
225x2 = 4
X2 = (4 / 225)
X = (√4 / 225) = + (2/ 15)
II. √225y + 2 = 0
√225y = -2
Y = -2/15
x≥y

Answer: d)

8. I (4 / √(x) + (7 / √(x) = √(x) (11/ √(x)) = √(x) (1 / √(x)) (4 + 7) = √(x)


11 =x
II. y2 – [(11)5/2 / √(y)] = 0
(y)2 – [(11)5/2 / √(y)1/2] = 0
[√(y)5/2 – (11)5/2 / (y)1/2] = 0
:. (y)5/2 = (11)5/2
:. Y =11
X=y
Answer: e)

9. I. x2 – 365 = 364
X2 = 362 + 365
X = √729 = + 27
II. y – √(324) = √(81)
Y – 18 = 9
Y = 9 + 18 = 27
y>x
Answer: b)

10. I. 3x2 + 8x + 4 = 0
3x2 + 6x + 2x + 4 = 0
3x( x + 2) + 2(x + 2) = 0
X = -2, – (2/3)
II. 4y2 19x + 12 = 0

Page 155 Follow us: Official Site, Telegram, Facebook, Instagram, Instamojo
4y2 – 16y – 3y + 12 = 0
4y(y -4) – 3(y -4) = 0
Y =4, (3/4)
:. y> x
Answer: a)

Page 156 Follow us: Official Site, Telegram, Facebook, Instagram, Instamojo
Time and Distance

1) Two buses start at same time from Chennai and Bangalore, which are
250km apart. If the two buses travel towards each other, they meet after
1hr and if they travel in same direction they meet after 5hrs. What is the
speed of the bus starts from Chennai if it is know that the one which
started from Chennai has more speed than the other one?
A) 150km/hr
B) 100km/hr
C) 45km/hr
D) 80km/hr
E) 120km/hr

2) Car A leaves the city at 5pm and is driven at a speed of 30km/hr. 3hrs
later another car B leaves the city in the same direction as car A. In how
much time will car B be 12kms ahead of car A if the speed of car B is
50km/hr?
A) 5hrs
B) 4 hrs 12 mins
C) 8hrs
D) 5 hrs 6 mins
E) 12hrs

3) Two train starts at the same time from Delhi and Agra and proceed
towards each other at the rate of 40km/hr and 37 1/2km/hr. When they
meet it is found that one train has traveled 200km more than the other
train. What is the distance between Delhi and Agra?
A) 6200km
B) 5000km
C) 4200km
D) 4800km
E) 6000km

Page 157 Follow us: Official Site, Telegram, Facebook, Instagram, Instamojo
4) A man walks from A to B and cycles from B to A, a distance of 37.5 km
in all spending 2 hours and 40 minutes. He would have taken 2/3rd hour
less had he chosen to cycle the entire distance of 37.5 km. what would have
been the time taken by him If he had chosen to walk both the ways?

a) 3 hours 30 min

b) 3 hours 20 min

c) 3 hours 12 min

d) 3 hours 25 min

e) None of these

5) Sanjay covers a certain distance with his own speed, but when he reduces
his speed by 10 kmph his time duration for the journey increases by 40 hours,
while if he increases his speed by 5 kmph from his original speed he takes 10
hours less than the original time taken. Find the distance covered by him.

a) 1200 km

b) 1500 km

c) 1600 km

d) 1300 km

e) None of these

6) Two persons start from the opposite ends of a 90 km straight track and
run to and from between the two ends. The speed of first person is 30 m/s
and the speed of other is 125/6 m/s. They continue their motion for 10 hours.
How many times they pass each other?

Page 158 Follow us: Official Site, Telegram, Facebook, Instagram, Instamojo
a) 10

b) 9

c) 12

d) 15

e) None of these

7) P, Q & R participated in a race. P covers the same distance in 49 steps, as


Q covers in 50 steps and R in 51 steps. P takes 10 steps in the same time as Q
takes 9 steps and R takes 8 steps. Who is the winner of the race?

a) P

b) Q

c) R

d) Can’t be determined

e)None of these

8) Raghav drives his truck very fast at 360 kmph. Moving ahead for some
hours he finds some problem in headlights of the truck. So he takes 20
seconds in changing in the bulb of the headlight by stopping the truck. Mean
while he notices that 2nd truck which was 400 m back is now 200 m ahead of
his truck. What is the speed of 2nd truck?

a) 100 kmph

b) 92 kmph

Page 159 Follow us: Official Site, Telegram, Facebook, Instagram, Instamojo
c) 108 kmph

d) 300 kmph

e) None of these

9) The distance between two towns A and B is 545 km. A train starts from
town A at 8 A.M. and travels towards town B at 80 km/hr. Another train
starts from town B at 9 : 30 A.M. and travels towards town A at 90 km/hr.
At what time will they meet each other?

A) 11:30 AM

B) 12:30 PM

C) 12:00 PM

D) 1:00 PM

E) 11:00 AM

10) Towns A and B are 225 km apart. Two cars P and Q travel towards each
other from towns A and B respectively and meet after 3 hours. If the speed
of P be 1/2 of its original speed and Q be 2/3 of its original speed, they would
have met after 5 hours. Find the speed of the faster car.

A) 50 km/hr

B) 40 km/hr

C) 45 km/hr

D) 30 km/hr

Page 160 Follow us: Official Site, Telegram, Facebook, Instagram, Instamojo
E) 60 km/hr

Answers:

1) Answer: A

S=D/T
Here we have two speeds. We get 2 equations as.
250/1hr = C+B—1 (Travelling in opposite direction, speed must be added ie
C+B)
250/5hr = C-B—2 (Travelling in same direction, speed to be subtracted. ie C-B)
solving 2 eqn C=150km/hr.

2) Answer: D
Car A travels 3hrs. 3*30=90km
Difference between speeds 50-30=20km/hr
Distance ahead 12km . 90+12=102km
T=D/S ===>102/20=5 hrs 6 mins

3) Answer: A
Speed ratio 40:37 1/2==>40: 75/2==>80:75 ie 16:15
ratio diff between speed is 1[16-15] 1 ===> 200 (more distance)
[16+15]31 ===>?
31*200=6200km.

4) Answer: B

He takes 40 minutes less to cycle than to walk. Thus, on the whole he takes 40
more.

Required time = 3 hours 20 minutes

5) Answer: B

D=S*T

Page 161 Follow us: Official Site, Telegram, Facebook, Instagram, Instamojo
With 1st condition when speed decreases eq is–: 4S – T = 40 ——1)

With 2nd condition when speed increased eq is –: (– 2S) + T = 10 ——–2)

Solving the a above two eq we get S = 25 kmph and T = 60 h

hence, distance = 25 * 60 = 1500 km

6) Answer: C

The speeds of two persons is 108 kmph and 75 kmph.

The first person covers (108*10)=1080 km in 10 hours and thus he makes


(1080/90)=12 rounds.

Thus, he will pass over another person 12 times in any one of the direction.

7) Answer: A

Distance=> P*49 = Q*50 = R*51

P= Distance/49, Q= Distance/50, R= Distance/51

Time => P*10 = Q*9 = R*8

P = Time/10, Q = Time/9, R= Time/8

P’s speed = (Distance/49)/(Time/10) = 10/49

Same like that,

The ratio of speeds of P, Q, R = (10/49) : (9/50) : (8/51)

Hence, P is the fastest.

8) Answer: C

Speed of 2nd truck = [(400 + 200)/20] * [18/5] kmph = 108 kmph

Page 162 Follow us: Official Site, Telegram, Facebook, Instagram, Instamojo
9) Answer: C

With 80 km/hr, distance travelled in 1 and half hours (9:30AM – 8AM) is 3/2 *
80 = 120 Km
Now second train also starts, and at this time distance between both trains is
(545-120) = 425 km
Relative speed = 80+90 = 170 km/hr (when travelling in opposite direction, add
speed)
So time when they meet = 425/170 = 2.5 hrs
So after 9:30 AM they meet after 2.5 hrs, so 12 PM

10) Answer: C
Let speeds be x km/hr and y km/hr
So 225/(x+y) = 3
And 225/(x/2 + 2y/3) = 5
Solve, x = 30, y = 45

Page 163 Follow us: Official Site, Telegram, Facebook, Instagram, Instamojo
Partnership

1. P, Q and R started a business with investments of Rs. 12000, Rs.


15000 and Rs. 18000 respectively. After 8 months from the start of
the business, Q and R invested additional amounts in the ratio of 3:
5 respectively. If at the end of the year, the ratio of share of P and
Q was 3: 4, then what was the additional amount invested by Q after
8 months?

a. Rs. 1500
b. Rs. 1800
c. Rs. 2100
d. Rs. 3000
e. None of these

2. A, B and C entered into a partnership by investing Rs. 30000, Rs.


25000 and Rs. 40000 respectively. After 4 months, A withdraws two-
fifth of the amount and B invested Rs. 15000 more. And after 3
months C withdraws three-fifth of the amount. Find the total profit
at the end of the year, if the share of B is Rs. 70000?

a. Rs. 174000
b. Rs. 188000
c. Rs. 172000
d. Rs. 164000
e. None of these

3. Karthi, Prakash and Vasanth enter into a Partnership with


investment in the ratio of (3/2): (8/5): (5/3). After six months, Karthi
increases his share by 40%. If the total profit at the end of the year
be Rs. 136800, then what will be the share of Prakash in the profit?

a. Rs. 41500

Page 164 Follow us: Official Site, Telegram, Facebook, Instagram, Instamojo
b. Rs. 42800
c. Rs. 43200
d. Rs. 38900
e. None of these

4. P, Q and R invested in the ratio of 4: 7: 9. After 5 months, P invested


Rs. 2500 more. And after 3 months, R withdraws Rs. 3000. Find the
share of Q, if the total profit at the end of the year is Rs. 89500?

a. Rs. 32500
b. Rs. 29400
c. Rs. 36800
d. Rs. 34300
e. None of these

5. Rahul, Vinay and Prabhu started a business by investing in the ratio


of 4: 7: 9. After 5 months, Rahul invested Rs. 15000 more and after
4 months, Vinay invested Rs. 10000 more. At the end of the year,
their profits are in the ratio of 39: 58: 72. Find the initial investment
of Vinay?

a. Rs. 65000
b. Rs. 72000
c. Rs. 78000
d. Rs. 70000
e. None of these

6. P, Q and R started a business by investing Rs. 27000, Rs. 35000 and


Rs. 42000 respectively. After 6 months, P withdraws half of his
investment but Q invested 20 % of initial investment more. Find the
share of R, if the total profit at the end of the year is Rs. 84630?

a. Rs. 31750
b. Rs. 35280
c. Rs. 33560

Page 165 Follow us: Official Site, Telegram, Facebook, Instagram, Instamojo
d. Rs. 30270
e. None of these

7. A starts a business with a capital of Rs. 25000. B joins the business


5 months after the start of the business and C joins the business
after 8 months. At the end of the year their respective shares is in
ratio of 30: 21: 16. What is the sum of amount invested by B and C
together?

a. Rs. 70000
b. Rs. 68000
c. Rs. 74000
d. Rs. 65000
e. None of these

8. Praveen and Raghav started a business by investing Rs. 24000 and


Rs. 36000 respectively. Praveen is a working partner and Raghav is
a sleeping partner in a business. Praveen receives 10 % of profits
for managing the business. Find the share of Praveen, if the total
profit at the end of the year is Rs. 50000?

a. Rs. 24000
b. Rs. 21000
c. Rs. 23000
d. Rs. 27000
e. None of these

9. Rajesh, Yuvaraj and Dhinesh started the business by investing in


the ratio of 7: 5: 6. 20 % of the profit goes to charity; the remaining
will be shared by three of them. The share of Yuvaraj is Rs. 30000.
Find the total profit?

a. Rs. 116000
b. Rs. 135000
c. Rs. 124000

Page 166 Follow us: Official Site, Telegram, Facebook, Instagram, Instamojo
d. Rs. 142000
e. None of these

10. A starts a business with a capital of Rs. 15000. B joins the business
after 6 months and C joins the business after 9 months. At the end
of the year, their respective shares were in ratio of 8: 4: 3. What is
the sum of amount invested in the business by B and C together?

a. Rs. 37500
b. Rs. 24000
c. Rs. 28000
d. Rs. 22000
e. None of these

Answers:

1). Answer: d

Let the ratio additional amount of Q and R be 3x and 5x,

The ratio of profit of A: B: C

= [12000*12]: [15000*8 + (15000 + 3x) *4]: [18000*8 + (18000 + 5x) *4]

= 144000: (120000 + 60000 +12x): (144000+ 72000 + 20x)

The ratio of share of P and Q = 3: 4

144000/(180000+ 12x) = (3/4)

192000 = 180000 + 12x

12000 = 12x

x = 1000

Q’s additional investment after 8 months = 3x = 3*1000 = Rs. 3000

Page 167 Follow us: Official Site, Telegram, Facebook, Instagram, Instamojo
2). Answer: a

The share of A, B and C

= > [30000*4 + 30000*(3/5)*8]: [25000*4 + 40000*8]: [40000*7 +


40000*(2/5)*5]

= > 264000: 420000: 360000

= > 22: 35: 30

35’s = 70000

1’s = 2000

Total profit = 87’s = 87*2000 = Rs. 174000

3). Answer: c

The investment ratio = (3/2): (8/5): (5/3)

= > 45: 48: 50

The share of Karthi, Prakash and Vasanth

= > [45*6 + 45*(140/100)*6]: [48*12]: [50*12]

= > 648: 576: 600

= > 27: 24: 25

Total profit = Rs. 136800

76’s = 136800

1’s = 1800

The share of Prakash = 24’s = Rs. 43200

Page 168 Follow us: Official Site, Telegram, Facebook, Instagram, Instamojo
4). Answer: b

The share of P, Q and R,

= > [4x*5 + (4x + 2500)*7]: [7x*12]: [9x*8 + (9x – 3000)*4]

= > [20x + 28x + 17500]: 84x: [72x + 36x – 12000]

Total profit = 89500

20x + 28x + 17500 + 84x + 72x + 36x – 12000 = 89500

240x + 5500 = 89500

240x = 84000

X = 350

The share of Q = 84x = Rs. 29400

5). Answer: d

The share of Rahul, Vinay and Prabhu

[4x*5 + (4x + 15000)*7]: [7x*9 + (7x + 10000)*3]: [9x*12] = 39: 58: 72


(20x + 28x + 105000]: [63x + 21x + 30000]: [108x] = 39: 58: 72

(48x + 105000): (84x + 30000): 108x = 39: 58: 72

(84x + 30000)/108x = (58/72)

84x + 30000 = 87x

3x = 30000

X = 10000

Initial investment of Vinay = 7x = Rs. 70000

Page 169 Follow us: Official Site, Telegram, Facebook, Instagram, Instamojo
6). Answer: b

The share of P, Q and R

= > [27000*6 + 13500*6]: [35000*6 + 35000*(120/100)*6]: [42000*12]

= > 243000: 462000: 504000

= > 81: 154: 168

403’s = 84630

1’s = 210

The share of R = 168’s = Rs. 35280

7). Answer: a

Let the initial investment of B and C be x and y,

Ratios of profits = >

[25000*12]: [7x]: [4y] = 30: 21: 16


30’s = 25000*12

1’s = 10000

21’s = 210000

16’s = 160000

Capital of B

= > 7x = 210000

= > x = Rs. 30000

Capital of C

Page 170 Follow us: Official Site, Telegram, Facebook, Instagram, Instamojo
= > 4y = 160000

= > y = 160000/4 = Rs. 40000

Total Money invested by B and C = 30000 + 40000 = Rs. 70000

8). Answer: c

The share of Praveen and Raghav

= > 24000: 36000

= > 2: 3

Praveen receives 10 % of profits for managing the business.

= > 50000*(10/100) = 5000

Remaining = 50000 – 5000 = 45000

5’s = 45000

1’s = 9000

The share of Praveen = 5000 + 18000 = Rs. 23000

9). Answer: b

The ratio of investment of Rajesh, Yuvaraj and Dhinesh

= > 7: 5: 6

5’s = 30000

1’s = 6000

18’s = 108000

Page 171 Follow us: Official Site, Telegram, Facebook, Instagram, Instamojo
80 % of total profit = 108000

(80/100)*Total profit = 108000

Total profit = 108000*(100/80) = Rs. 135000

10). Answer: a

The share of A, B and C = [15000*12]: 6B: 3C = 8: 4: 3

Capital of B

(15000*12)/6B = 8/4

=>B = 15000

Capital of C

(15000*12)/3C = 8/3

=>C= 22500

Total Money invested = 15000 + 22500 = Rs. 37500

Page 172 Follow us: Official Site, Telegram, Facebook, Instagram, Instamojo
Ratio and Proportion

1) The ratio of the number of male and female in a committee is 5:6. If the
percentage increase in the number of male and female be 12% and 10%
respectively, what will be the new ratio?

A.33:28

B.28:33

C.27:34

D.34:27

E.None of these

2) A manager divided Rs 234 into three workers P, Q and R such that 4 times
P’s share is equal to 6 times Q’s share which is equal to 3 times R’s share.
How much did P get?

A.Rs 82

B.Rs 78

C.Rs 56

D.Rs 64

E.None of these

3) A box contains the coins of 5p, 10p and 25p are in the ratio 3:3:5. If there
is Rs. 34 in all, how many 10p coins are there?

Page 173 Follow us: Official Site, Telegram, Facebook, Instagram, Instamojo
A.40

B.60

C.80

D.100

E.None of these

4) The wages of Naveena and Menaka are in the ratio ratio 6:5. If the wages
of each is increased by Rs.6000, the new ratio becomes 38:35. What is
Menaka’s present salary?

A. 4000
B. 5500
C. 5000
D. 4500
E. None of these

5) Ratio of the salary of X & Y is 5:8. If the salary of X increase by 40% and
those of Y decrease by 15%, the new ratio of their salary becomes 10:9. What
is X’s salary?

A.24000

B.25000

C.28000

D.data inadequate

E.None of these

Page 174 Follow us: Official Site, Telegram, Facebook, Instagram, Instamojo
6) The ratio of the number of UG students and PG students in a college is
5:4. If 40% of the UG students and 50 % of the PG students are scholarship
holders. What percentage of the students does not get the scholarship?

A.54.6%

B.55.2%

C.55.5%

D.54.3%

E.None of these

7) Karan, Hari and Kowshik play cricket. The runs got by Karan to Hari
and Hari to Kowshik are in the ratio of 5:3. They get altogether 588 runs.
How many runs did Karan get?

A. A.150 runs
B. B.300 runs
C. C.250 runs
D. D.200 runs
E. E.None of these

8. The cost of 8A is equal to the cost of 50B. The cost of 19C is 456. The
cost of B is twice the cost of 2C. What is the total cost of 3A and 4B
together?

A. 2184
B. 2168
C. 2243
D. 2264
E. None of these

Page 175 Follow us: Official Site, Telegram, Facebook, Instagram, Instamojo
9. Divide Rs. 370 into three parts such that second part is 1/4 of the
third part and the ratio between the first and the third part is 3 : 5.
Find the amounts of these three parts respectively.

A. 50, 120, 200


B. 120, 50, 200
C. 200, 120, 50
D. 200, 100, 50
E. None of these

10. Two numbers are in the ratio of (1 ½): (2 2/3). When each of these
is increased by 15, the ratio changes to 1 2/3: 2 ½. The larger of the
numbers is,

A. 48
B. 27
C. 36
D. 64
E. 56

Answers:

1) Answer: B

Originally, let the number of male and female in a committee be 5x and 6x


respectively.

Their increased number is (112% of 5x) and (110% of 6x).

(112/100)*5 and (110/100)*6

=5.6 : 6.6

=56 : 66

Page 176 Follow us: Official Site, Telegram, Facebook, Instagram, Instamojo
=28 : 33

So, the required ratio = 28 : 33

2) Answer: B

4 times P’s share = 6 times Q’s share = 3 times R’s share

4*P = 6*Q

P/Q = 3/2

6*Q = 3*R

Q/R = ½

P:Q:R=3:2:4

The sum of the total Salary,

3x+2x+4x=234

9x=234

X=234/9=26

Page 177 Follow us: Official Site, Telegram, Facebook, Instagram, Instamojo
Hence p gets =3x=3*26= Rs 78

3) Answer: B

Let the no. of 5p, 10p, 25 p coins be 3x, 3x & 5x respectively.

Then sum of their values = Rs. [(5*3x/100) + (10*3x /100) + (25*5x/100)]

=Rs. [(15x/100) + (30x /100) + (125x/100)]

=Rs. (15x+30x+125x / 100)

=Rs. (170x /100)

Therefore 170x / 100 = 34

X=34 *100/170

X=20

Hence the no. of 10p coins = 3x

=3*20

=60

4) Answer: D

Let the original salary of Naveena and Menaka be 6x and 5x respectively

Then, (6x+6000) / (5x+6000) = 38/35

35(6x+6000) = 38(5x +6000)

210x +210000 = 190x + 228000

210x – 190x = 228000 – 210000

Page 178 Follow us: Official Site, Telegram, Facebook, Instagram, Instamojo
20x = 18000

X=900

Menaka’s present salary = 5x

= 5*900 = Rs. 4500

5) Answer: D

Let the original salary of X & Y be 5x & 8x respectively

New salary of X = 40% of 5x

=140/100 * 5x

=700x /100

= 7x

New salary of Y = 85% of 8x

=185 /100 *8x

= 74x/5

Therefore 7x : 74x/5 = 10:9

7x*5/74x =10:9

We can’t find x, so the given data is inadequate

6) Answer: C

Let UG students be = 5x and

PG students be = 4x respectively

Page 179 Follow us: Official Site, Telegram, Facebook, Instagram, Instamojo
No. of those who do not get scholarship,

= 60% of 5x + 50% of 4x

= (60/100*5x) + (50/100*4x)

= 300x/100+ 200x /100

= 3x + 2x

= 5x

Required percentage = (5x/9x) *100

= 55.5%

7) Answer: B

Karan : Hari =5:3

Hari : Kowshik =5:3

Karan : Hari : Kowshik = 25:15:9

Therefore, the runs made by Karan,

=25/49*588

=25*12=300 runs

8) Answer: A

8A= 50B=> A/B=50/8

A : B= 50:8=>25:4

19C = 456

Page 180 Follow us: Official Site, Telegram, Facebook, Instagram, Instamojo
=>C=24

B=2*2C=4*24=96

4’S=96=>1’S=24

A=24*25=600

3A +4B=3*600+4*96=2184

9) Answer: B

Let the first and the third parts be 3x and 5x.

Second part = 1/4 of third part = (1/4) × 5x == 5x/4

Therefore, 3x + (5x/4) + 5x = 370

(12x + 5x + 20x)/4 = 370

37x/4 = 370

x = (370 × 4)/37

x = 10 × 4

x = 40

Therefore,

First part = 3x =3 × 40=120

Second part = 5x/4 =5 × 40/4=50

Third part = 5x =5 × 40=200

10) Answer: A

Page 181 Follow us: Official Site, Telegram, Facebook, Instagram, Instamojo
Two numbers are in the ratio of,

(1 ½): (2 2/3) =>3/2: 8/3 =>9:16 (9x, 16x)

1 2/3: 1 ½ =>5/3:5/2 =>2:3

9x+15/16x+15 =2/3

27x+45 = 32x+30

5x= 15

X=3

The larger of the numbers is, 16x= 48

Page 182 Follow us: Official Site, Telegram, Facebook, Instagram, Instamojo
Problem on Ages

1) 6 years ago, the ratio of the ages of Arun and Prathap is 7: 6. Present age
of Rajeev is 10 years more than one – sixth of Prathap’s present age. Find
the ratio of present age of Prathap and Rajeev, if Arun’s age after 6 years
is 40 years?

a) 3: 2

b) 2: 1

c) 4: 5

d) 1: 3

e) None of these

2) The ratio of present ages of Sri and Gowtham is 3: 4. Mahesh is 6 years


older than Sri and two years younger than Gowtham. Find the sum of the
present ages of Sri and Mahesh?

a) 48 years

b) 50 years

c) 52 years

d) 54 years

e) None of these

3) The product of the ages of Asha and Nithi is 540. If twice the age of Asha
is more than Nithi’s age by 6 years, then find Asha’s age?

Page 183 Follow us: Official Site, Telegram, Facebook, Instagram, Instamojo
a) 18 years

b) 20 years

c) 16 years

d) 22 years

e) None of these

4) Naveena’s present age is four times her son’s present age and two fifth of
her father’s present age. The average present age of all of them is 40 years.
Find the sum of Naveena’s son’s present age and Naveena’s father’s
present age?

a) 92 years

b) 74 years

c) 88 years

d) 86 years

e) None of these

5) The ratio of B’s age six years hence to C’s present age is 5: 3. B is eleven
years younger than A. If A’s present age is twice the age of C, then find B’s
age, 4 years ago?

a) 13 years

b) 15 years

c) 17 years

Page 184 Follow us: Official Site, Telegram, Facebook, Instagram, Instamojo
d) 16 years

e) None of these

6) After 2 years, the age of Karthi is 2 times the present age of Silambu.
Preethi is 8 years elder than Silambu. Find the present age of Karthi, if the
present age of Preethi is 23 years?

a) 28 years

b) 26 years

c) 24 years

d) 30 years

e) None of these

7) 4 years ago, the age of A is 2 times the present age of B. C is 12 years


elder than B and 10 years younger than A. Find the present age of D, if the
present age of C is 6 times the present age of D?

a) 3 years

b) 6 years

c) 7 years

d) 5 years

e) None of these

Page 185 Follow us: Official Site, Telegram, Facebook, Instagram, Instamojo
8) The ratio of present age of Ramesh and Karthi is 2: 3. Sheela is 3 years
elder than Karthi. The average present ages of three of them is 25 years.
Find the age of Janvi, 4 years ago, if the present age of Janvi is 2 times the
present age of Ramesh?

a) 29 years

b) 35 years

c) 32 years

d) 27 years

e) None of these

9) The ratio between Gautham and Manoj is 35: x. Manoj is 6 years


younger than Nisha. Nisha’s age after 8 years will be 30 years. The
Gautham’s present age is 3 years less than the sum of the present age of
Manoj and Nisha. Find the value of x?

a) 22

b) 16

c) 18

d) 17

e) None of these

10) The difference between the present ages of Rohit and Agila is 17 years.
If the ratio of their ages 8 years hence is 2: 1, then find the present age of
Rohit’s mother who is 28 years older than Rohit?

Page 186 Follow us: Official Site, Telegram, Facebook, Instagram, Instamojo
a) 48 years

b) 50 years

c) 52 years

d) 54 years

e) None of these

Answers :

1). Answer: b)

6 years ago, the ratio of the ages of Arun and Prathap = 7: 6 (7x, 6x)

Present ages of Arun and Prathap = 7x + 6, 6x + 6

Present age of Rajeev = (1/6)*Prathap’s present age + 10

Arun’s present age = 34 years

According to the question,

7x = 28

x=4

Prathap’s present age = 6x + 6 = 30

Rajeev’s present age = (1/6)*30 + 10 = 15

Required ratio = 30: 15 = 2: 1

2). Answer: d)

Ratio of present ages of Sri and Gowtham = 3: 4 (3x, 4x)

Page 187 Follow us: Official Site, Telegram, Facebook, Instagram, Instamojo
Mahesh = Sri + 6 = 3x + 6

Mahesh = Gowtham – 2 = 4x – 2

According to the question,

3x + 6 = 4x – 2

4x – 3x = 8

X=8

Present age of Sri = 3x = 24 years

Present age of Mahesh = 24 + 6 = 30 years

Required sum = 24 + 30 = 54 years

3). Answer: a)

Let the age of Asha and Nithi be x and y respectively,

x*y = 540

2*x = y + 6

X = (y + 6)/2

[(y + 6)/2]*y = 540


Y2 + 6y = 1080

Y2 + 6y – 1080 = 0

(y – 30) (y + 36) = 0

Y = 30, -36 (-36 will be eliminated)

Asha’s age = 540/30 = 18 years

Page 188 Follow us: Official Site, Telegram, Facebook, Instagram, Instamojo
4). Answer: c)

The ratio of present age of Naveena and her son’s age = 4 : 1 (4x, x)

Naveena’s present age = (2/5)*Naveena’s father’s present age

Naveena’s father’s present age = 10x

According to the question,

x + 4x + 10x = 120

15x = 120

x = 120/15

x=8

Naveena’s father’s present age = 10x = 80 years

Naveena’s son’s present age = x = 8 years

Required sum = 80 + 8 = 88 years

5). Answer: b)

The ratio of B’s age six years hence to C’s present age = 5 : 3 (5x, 3x)

Present ages of B to C = 5x – 6, 3x

Present ages of A to C = 2 : 1 (2x, x)

Present ages of A, B and C = 6x, 5x – 6, 3x

B = A – 11

A – B = 11

Page 189 Follow us: Official Site, Telegram, Facebook, Instagram, Instamojo
6x – (5x – 6) = 11

6x – 5x + 6 = 11

X=5

B’s age, 4 years ago = 5x – 6 – 4 = 25 – 10 = 15 years

6). Answer: a)

The ratio of age of Karthi’s age after 2 years and Silambu’s present age

= 2 : 1 (2x, x)

Preethi’s present age = 23 years

Silambu’s present age = 23 – 8 = 15

Silambu’s present age = 15

X = 15

2x = 30

After 2 years, the age of Karthi = 30 years

Present age of Karthi = 28 years

7). Answer: d)

The ratio of age of A, 4 years ago and the present age of B = 2: 1 (2x, x)

C = B + 12

C = A – 10

A – 10 = B + 12

Page 190 Follow us: Official Site, Telegram, Facebook, Instagram, Instamojo
2x + 4 – 10 = x + 12

X = 18

Present age of A = 2x + 4 = 40 years

Present age of B = 18 years

Present age of C = 18 + 12 = 30 years

Present age of C = 6* Present age of D

Present age of D = 30/6 = 5 years

8). Answer: c)

The ratio of present age of Ramesh and Karthi = 2 : 3 (2x, 3x)

Sheela = 3 + Karthi = 3x + 3

Sum of the present ages of three of them = 75 years

2x + 3x + 3x + 3 = 75

8x = 72

x=9

Present age of Ramesh = 2x = 18 years

Present age of Janvi = 2*Ramesh = 2*18 = 36 years

The age of Janvi, 4 years ago = 32 years

9). Answer: b)

The present age of Gautham and Manoj = 35: x

Page 191 Follow us: Official Site, Telegram, Facebook, Instagram, Instamojo
Manoj = Nisha – 6

Nisha’s age = 30 – 8 = 22 years

Manoj = 22 – 6 = 16

Gautham’s age = (Manoj’s age + Nisha’s age) – 3

Gautham’s age = 16 + 22 – 3 = 35

The value of x = 16

10). Answer: d)

Rohit – Agila = 17

8 years hence, The ratio of Rohit and Agila = 2 : 1 (2x, x)

Present age of Rohit and Agila = 2x – 8, x – 8

According to the question,

2x – x = 17

x = 17

Present age of Rohit = 2x – 8 = 26

Present age of Rohit’s mother = 26 + 28

= > 54 years

Page 192 Follow us: Official Site, Telegram, Facebook, Instagram, Instamojo
Boats and Stream

1) A boat can travel 55 km downstream in 66 min. The ratio of the speed of


the boat in still water to the speed of the stream is 4: 1. How much time will
the boat take to cover 72 km upstream?

a) 2 hour 48 min

b) 3 hour 12 min

c) 2 hour 24 min

d) 3 hour 28 min

e) None of these

2) Mahesh can swim at 20 km/hr in still water. The river flows at 8 km/hr
and it takes 8 hours more upstream than downstream for the same distance.
How far is the place?

a) 168 km

b) 152 km

c) 140 km

d) 124 km

e) None of these

3) A Boat takes 96 min less to travel to 72 Km downstream than to travel the


same distance upstream. If the speed of the stream is 8 Km/hr. Then Speed
of Boat in still water is?

Page 193 Follow us: Official Site, Telegram, Facebook, Instagram, Instamojo
a) 32 km/hr

b) 20 km/hr

c) 36 km/hr

d) 28 km/hr

e) None of these

4) A boat can travel 4 km upstream in 15 min. If the ratio of the speed of the
boat in still water to the speed of the stream is 9: 5. How much time will the
boat take to cover 28 km downstream?

a) 42 min

b) 30 min

c) 36 min

d) 44 min

e) None of these

5) A boat can cover 42 km upstream in 63 minutes. If the speed of the current


is 3/7 of the boat in still water, then how much distance (in km) can the boat
cover downstream in 42 minutes?

a) 70 km

b) 66 km

c) 52 km

Page 194 Follow us: Official Site, Telegram, Facebook, Instagram, Instamojo
d) 84 km

e) None of these

6) A boat takes 26 hours for travelling downstream from point A to point B


and coming back to point C midway between A and B. If the velocity of the
stream is 8 km/hr and the speed of the boat in still water is 20 km/hr, then
find the distance between A to B?

a) 284 km

b) 212 km

c) 336 km

d) 198 km

e) None of these

7) Speed of a man in still water is 16 km/hr and the river is running at 6


km/hr. The total time taken to go to a place and come back is 8 hours. Find
the distance travelled by the man?

a) 146 km

b) 110 km

c) 142 km

d) 133 km

e) None of these

Page 195 Follow us: Official Site, Telegram, Facebook, Instagram, Instamojo
8) A Boat took 3 1/2 hours less to travel a distance downstream than to travel
the same distance upstream. If the speed of a boat in still water is 15 Km/hr
and speed of a stream is 7 Km/hr. In total how much distance traveled by
boat?

a) 36 km

b) 52 km

c) 66 km

d) 44 km

e) None of these

9) A boat running upstream takes 6 hours 36 minutes to cover a certain


distance, while it takes 3 hours to cover the same distance running
downstream. What is the ratio between the speed of the boat and speed of
the water current respectively?

a) 8: 3

b) 5: 2

c) 7: 4

d) 2: 1

e) None of these

10) A man takes to row 80 km along the stream is half of the time taken to
row against the stream. Find the speed of the boat in still water if the speed
of stream is 6 km/hr?

Page 196 Follow us: Official Site, Telegram, Facebook, Instagram, Instamojo
a) 24 km/hr

b) 20 km/hr

c) 18 km/hr

d) 12 km/hr

e) None of these

Answers:

1) Answer: c)

Speed of downstream = D/T = 55/(66/60) = 55*(60/66) = 50 km/hr

The ratio of the speed of the boat in still water to the speed of the stream

= > 4: 1 (4x, x)

5x = 50

X = 10

Speed of upstream = 4x – x = 3x = 30 km/hr

Distance = 72 km

Time = D/S = 72/30 = 2 2/5 hr = 2 hour 24 min

2) Answer: a)

Speed of downstream = (20 + 8) = 28 km/hr

Speed of upstream = (20-8) = 12 km/hr

Page 197 Follow us: Official Site, Telegram, Facebook, Instagram, Instamojo
According to the question,

[x/12 – x/28] = 8
16x/(12*28) = 8

= > x= 168 km

3) Answer: d)

Let the speed of boat in still water be x,

According to the question,

96/60 = 72/(x – 8) – 72/(x + 8)

96/60 = 72(1/(x-8) – 1/(x+8))

x2 – 64 = 720

x2 = 784

x = 28

The speed of boat in still water = 28 km/hr

4) Answer: b)

The ratio of the speed of the boat in still water to the speed of the stream = 9: 5
(9x, 5x)

Speed of downstream = 9x + 5x = 14x

Speed of upstream = 9x – 5x = 4x

Upstream speed = 4/(15/60) = 16 km/hr


4x = 16
x=4

Page 198 Follow us: Official Site, Telegram, Facebook, Instagram, Instamojo
Downstream speed = 14*4 = 56 km/hr
Time = D/S = 28/56 = ½ hr = 30 min

5) Answer: a)

Speed of upstream = (42*60)/63 = 40 km/hr

Speed of current: Speed of still water = 3: 7

Speed of upstream = Speed of boat in still water – speed of Current

40 = 4x

= > x = 10

Speed of current = 30 km/hr

Speed of boat in still water = 70 km/hr

Speed of downstream= Speed of boat in still water + speed of Current

= > 70 + 30 = 100 km/hr

Speed = 100 km/hr, Time = 42 minutes

Distance = 100*(42/60) = 70 km

6) Answer: c)

Speed of downstream = 20 + 8 = 28 km/hr


Speed of upstream = 20 – 8 = 12 km/hr
Total time = 26 hours

If distance between A to B is x, then distance between B to C = x/2,


Time = Distance/Speed

x/28 + (x/2)/12 = 26

Page 199 Follow us: Official Site, Telegram, Facebook, Instagram, Instamojo
x/28 + x/24 = 26
52x/(28*24) = 26

x = (26*28*24)/52 = 336 km

The distance between A to B = 336 km

7) Answer: b)

Speed of downstream = 16 + 6 = 22 km/hr


Speed of upstream = 16 – 6 = 10 km/hr
Let the distance travelled be x,
(x/22) + (x/10) = 8
X= 55 km

(Or)

Distance = Time *[(Speed of Still water2 – speed of Stream2)/(2*Speed of Still


water)]

= > Distance = 8*[(162 – 62)/(2*16)]

= > Distance = 8*[(256 – 36)/32]

= > Distance = 55 km

8) Answer: d)

Speed of downstream = 15 + 7 = 22 km/hr

Speed of upstream = 15 – 7 = 8 km/hr

According to the question,


7/2 =D/8 – D/22

7/2 = (22D – 8D)/(22*8)

D = 44 km

Page 200 Follow us: Official Site, Telegram, Facebook, Instagram, Instamojo
9) Answer: a)

Let speed of upstream be x km/hr and that of downstream be y km/hr,


Then, distance covered upstream in 6 hrs 36 min = Distance covered
downstream in 3 hrs.
x*(33/5) = 3y
y =11x/5
Required ratio = (y + x)/2 : (y – x)/2
= > 16x/10 : 6x/10
=>8:3
10) Answer: c)

Let us take the speed of the boat in still water be x km/hr,

80/(x+6) = 1/2*(80/(x-6))

2x – 12 = x + 6

X = 18

Speed of the boat in still water = 18 km/hr

Page 201 Follow us: Official Site, Telegram, Facebook, Instagram, Instamojo
Pipe and Cistern

1. Two pipes A and B can fill a tank in 20 hours and 25 hours


respectively, while a third pipe C can empty 70% of the tank in 21
hours. All three pipes are opened in the beginning. After 12 hours
C is closed. Find the time in which the tank will be full?

a. 13 2/3 hours
b. 11 7/8 hours
c. 16 ¾ hours
d. 15 5/9 hours
e. None of these

2. Two pipes A and B can fill a tank in 18 hours and 24 hours


respectively, while a third pipe can empty 60% of the tank in 18
hours. All three pipes are opened in the beginning. After 12 hours c
is closed. Find the time which the tank will be full?

a. 14 2/5 hours
b. 12 ¾ hours
c. 11 5/6 hours
d. 13 2/3 hours
e. None of these

3. Three pipes A, B and C can fill a tank in 10 hours. After working at


it together for 3 hours, C is closed and A and B can fill the remaining
part in 14 hours. How much time taken by C to fill the tank alone?

a. 18 hours
b. 20 hours
c. 24 hours
d. 22 hours
e. None of these

Page 202 Follow us: Official Site, Telegram, Facebook, Instagram, Instamojo
4. Three pipes A, B, C can fill a cistern in 10, 15, 18 hours respectively.
If pipe A is kept open all the time and pipe B and C is opened
alternatively for an hour then find in how many hours the tank will
be filled?

a. 7 hours 25 mins
b. 5 hours 40 mins
c. 6 hours 12 mins
d. 7 hours 48 mins
e. None of these

5. Pipe A and Pipe B can fill a tank in 10 hours and 15 hours


respectively while Pipe C can empty the full tank in 18 hours. If pipe
A is opened at 7 am, Pipe B opened at 8:30 am and Pipe C at 10 am,
then after 10 am, how much more time will be taken by all three
pipes together to fill the tank?

a. 7 hour 18 min
b. 8 hour 36 min
c. 6 hour 42 min
d. 5 hour 24 min
e. None of these

6. P, Q and R can fill the tank in 12, 16 and 24 min respectively. All
three began to fill the tank together but P and Q left 3 and 4 min
respectively before filling the tank. Find the total time taken by all
of them to fill the tank?

a. 12 min
b. 16 min
c. 8 min
d. 10 min
e. None of these

Page 203 Follow us: Official Site, Telegram, Facebook, Instagram, Instamojo
7. Two pipes A and B can fill a tank in 15 minutes and 25 minutes
respectively. Both pipes are opened together andpipeBisclosed,5
minutes before the tank is filled completely. Calculate the total time
required to fill the tank?

a. 11 ¼ min
b. 13 3/5 min
c. 12 7/8 min
d. 14 5/6 min
e. None of these

8. B is 5/4 times as efficient as A. If A can fill the 3/5 of the tank in 15


min, what fraction of the capacity of the tank would remain
incomplete if B can fill the tank independently for 10 min only?

a. 2/3
b. 1/3
c. ¼
d. ½
e. None of these

9. P, Q and R can fill the tank in 15, 18 and 20 min respectively. All
three began to fill the tank together but P and Q left 4 and 5 min
respectively before filling the tank. In how much time will take to
fill the tank?

a. 7 22/25 min
b. 8 30/31 min
c. 7 11/15 min
d. 8 4/7 min
e. None of these

10. Two inlet pipes can fill an empty tank in 15 and 18 hours and one
outlet pipe can empty the tank in 20 hours. If all the pipes opened
simultaneously, then how many hours required fill the full tank?

Page 204 Follow us: Official Site, Telegram, Facebook, Instagram, Instamojo
a. 11 8/11 hours
b. 12 5/7 hours
c. 13 11/13 hours
d. 9 5/9 hours
e. None of these

Answers:

1). Answer d

C can empty the tank in = (21/70)*100= 30 hours

Total units of work= 300

A’s one hour work = 300/20 = 15 units

B’s one hour work = 300/25 = 12 units

C’s capacity to empty in one hour = 300/30 = 10 units

When three pipes are opened simultaneously then in one hour they will fill,

(15+12-10) = 17 units

In 12 hours they will fill = 17*12= 204 units

After 12hrs, C will be closed

Time taken by A& B to fill the remaining tank = 96/27 = 3 15/27 = 3 5/9 hours

Total time will be = 12 + 3 5/9 = 15 5/9 hours

2). Answer a

C can empty the tank in =18/60*100= 30 hours

Total units of work= 360

Page 205 Follow us: Official Site, Telegram, Facebook, Instagram, Instamojo
A’s one hour work= 360/18= 20units

B’s one hour work = 360/24= 15 units

C’s capacity to empty in one hour= 360/30= 12 units

When three pipes are opened simultaneously then in one hour they will

(20+15-12)=23 units

In 12 hours they will fill= 23*12= 276 units

After 12 hr, C will be closed

Time taken by A & B to fill the remaining tank = 84/35= 2 2/5 hours

Total time = 12 + 2 2/5 = 14 2/5 hours

3). Answer b

Three pipes A, B and C can fill a tank in 8 hours.

A, B and C’s 1 hour work=1/10

A, B and C’s 3 hour work= 3/10

Remaining work= 1 – (3/10) = 7/10

The remaining part will be filled by A and B in 14 hours. Then,

= > (7/10) *(A+B) = 14

= > (A+B)’s whole work= 14*(10/7) = 20 hr

(A+B)’s 1 hour work= 1/20

A, B and C’s 1 hour work = 1/10

Page 206 Follow us: Official Site, Telegram, Facebook, Instagram, Instamojo
C’s 1 hour work= (A+B+C) – (A+B)

= > (1/10) – (1/20)

= > 1/20

C can fill the tank in 20 hours.

4). Answer c

Total units of work= 90

A’s one hour work = 9

B’s one hour work = 6

C’s one hour work = 5

Work done by three pipes in 2 hours = 15 + 14 = 29 units

Work done in 6 hours= 29*3 = 87 units

Remaining will be done by pipe A and B = 3/15 = 1/5 hours

Total time required= 6 1/5 hours = 6 hours 12 mins

5). Answer d

Let capacity of tank = 90 lit (LCM of 10, 15, 18)

A fills 90/10 = 9 liter/hour

B fills 90/15 = 6 liter/hour

C empties 90/18 = 5 liter/hour

Till 10 am 9*3 + 6*1.5 = 27 + 9 = 36 liters

Page 207 Follow us: Official Site, Telegram, Facebook, Instagram, Instamojo
Remaining volume to be filled =90 – 36 = 54 liters

After 10 am, when all pipes are opened. 9 + 6 – 5= 10 liters/hr is filled.

Required time = 54/10 = 5 4/10 = 5 2/5 hour = 5 hour 24 min

6). Answer c

(x-3)/12 + (x-4)/16 + x/24 =1

(4x – 12 + 3x – 12 + 2x)/48 = 1

9x – 24 = 48

9x = 72

X = 8 min

7). Answer a

Let total capacity = 75 litres (LCM of 15 and 25)

A = 5 litres/min

B = 3 litres/min

A+B fill 75 litres

A can fill 5*5 = 25 litres in 5 min

But B was closed 5 min before the tank is filled. So A+B together filled (75-25)
= 50 litres

50 litres can be filled by A + B in,

= > (50/8) = 6 1/4

Total time = 6 1/4 + 5 = 11 ¼ min

Page 208 Follow us: Official Site, Telegram, Facebook, Instagram, Instamojo
8). Answer d

B is 5/4 times as efficient as A. So,

A and B’s time =>5 : 4

A can fill the 3/5 of the tank in 15 min

=> (3/5) * work = 15 min

=> Whole work = 25 min

A takes 25 min to fill the tank, So, B takes,

=> 5’s =25

=>1’s = 5

B can fill the tank in, 20 min

B can fill the tank independently for 10 min only.

B’s 10 min work = 10/20 = 1/2

Remaining = 1- 1/2 = ½

Half of the tank will remain incomplete.

9). Answer b

(x – 4)/15 + (x – 5)/18 + x/20 =1

(12x – 48 +10x – 50 + 9x)/180 =1

31x – 98 = 180

31x = 278

Page 209 Follow us: Official Site, Telegram, Facebook, Instagram, Instamojo
X = 278/31 = 8 30/31 min

10). Answer c

If all the pipes are opened simultaneously, then in,

= > (1/15) + (1/18) – (1/20)

= > (12 + 10 – 9)/180

= > 13/180

Required hours = 180/13 = 13 11/13 hours

Page 210 Follow us: Official Site, Telegram, Facebook, Instagram, Instamojo
ENGLISH SECTION

SYLLABUS:

 Reading Comprehension
 Spotting Error
 Fill in the blank
 Cloze test
 Sentence Improvement
 Para-Jumbles

Page 211 Follow us: Official Site, Telegram, Facebook, Instagram, Instamojo
Reading Comprehension

Direction (1-10): Read the passage carefully and answer the following
questions.

When we talk about any market it comes to our mind that a market consists of
many shops, outlets, stalls, hawkers and now newly developed markets known as
malls. Very obvious all these are essential parts of any market but when we talk
about Money Market means that there are hundreds of shops selling money.

No doubt there are such places also where we see the currency notes and coins
piled up on a table or counter but all these shops of money are just dealing in
exchange of money on two accounts- First they deal with exchanging old, torn,
mutilated , and such currency notes which are not usually accepted by common
shopkeepers.

The money market is entirely different where no visible shops are counters exist.

It comprises of different types of companies, institutions, firms, individuals who


are either in need of money or are having surplus money. The entities in need of
money seek financial help from this market against some payables and the entities
having surplus funds, deploy their surplus money in gainful way.

Money market basically refers to a section of the financial market where


financial instruments with high liquidity and short-term maturities are traded.
Money market is that component of the financial market which is used for
buying and selling of securities of short-term maturities, of one year or less.

Money market instruments offer low risk, their returns are significantly lower
than those of other securities.

Withdrawing money from the money market is easier. Money markets are
different from capital markets as they are for a shorter period of time while capital
markets are used for longer time periods.

Page 212 Follow us: Official Site, Telegram, Facebook, Instagram, Instamojo
The money market is used by a wide array of participants, from a company raising
money by selling commercial paper into the market to an investor purchasing
Certificate of Deposits as a safe place to park money in the short term.

Money market consists of two sectors- organized sector comprising the Reserve
Bank of India and commercial banks, and unorganized sector having an
indigenous stint.

The functions and activities of organized sector of money market are coordinated
by the RBI.

The organized sector of the Indian money market can be divided into sub-
markets.

The call money market (CMM) is the market where overnight (one day) loans
can be availed by banks to meet liquidity. Banks who seek to avail liquidity,
approach the call market as borrowers and the ones who have excess liquidity,
participate there as lenders.

Call Money Market is the main market oriented mechanism to meet the liquidity
requirements of banks. Call Money refers to the borrowing or lending of funds
for 1 day. Notice Money refers to the borrowing and lending of funds for 2-14
days and Term money refers to borrowing and lending of funds for a period of
more than 14 days.

Treasury bill market is the market for the sale and purchase of short term
government securities. These securities are called treasury bills which are
promissory notes or financial bills issued by RBI on behalf of the government of
India. In other words, it can be said that when government goes to the financial
market to raise money, it can do it by issuing two types of debt instruments –
treasury bills and government bonds. Treasury bills are issued when the
government need money for a shorter period while bonds are issued when it need
debt for more than say five years.

Market for the Certificate of Deposit (CDs) again an important segment of


Indian money market. It is issued by the commercial bank. A certificate of
deposit is an agreement to deposit money for a fixed period with a bank that will
pay you interest.

Page 213 Follow us: Official Site, Telegram, Facebook, Instagram, Instamojo
Market for the commercial papers is the market where commercial papers
are traded.Commercial paper is an unsecured, short-term debt instrument issued
by a corporate house, have working capital more than or equal to rupees 5 crore.
It is typically issued for the financing of accounts payable and inventories, and
meeting short-term liabilities.

The money market plays an important part in the economy of every nation and
has specific role particularly in short term financing. It helps in maintaining
balance between demand and supply of short term monetary transactions.

Money market plays a very important role by making funds available to many
units or entities engaged in diversified field of activities be it agriculture,
industry, trade, commerce or any other business.

By providing funds to developing sectors it helps in growth of economy also.

No doubt it provides a base for the implementation of monetary policy also. The
money market provides opportunity for short term investments.

Money market is integral part of financial market where instruments of high


liquidity with very short period of maturity are traded. The money circulated
throughout the money market, finances short-term borrowing by large
corporations and the government. This borrowing allows both businesses and
government agencies to continue to spend the money on programs and expansion
projects necessary to encourage economic growth.

1) According to the passage, in order to meet their liquidity requirements,


banks reach which of the following markets?

a) treasury bill market

b) commercial bill market

c) call money market

d) commercial paper market

e) short term loan market

Page 214 Follow us: Official Site, Telegram, Facebook, Instagram, Instamojo
2) According to the passage, which of the following functions, is not played
by the money market?

a) provide opportunity for short term investment

b) discounting bill of exchange

c) promote the growth of the economy

d) provide base for implementing monetary policy

e) make funds available to business units

3) According to the passage, which of the followings is not the feature of


money market?

a) easy withdrawal of money

b) high liquidity

c) lower return

d) lower risk

e) securities of long term maturities

4) According to the passage, for financing its short term liabilities, which of
the following instruments can be used by the business house?

a) bond

b) treasury bill

c) fixed deposit

Page 215 Follow us: Official Site, Telegram, Facebook, Instagram, Instamojo
d) commercial paper

e) certificate of deposit

5) Which of the following statements is true in the context of the passage?

a) Certificate of deposit is issued by listed company or business firm.

b) Money market instruments offer higher risk.

c) Notice Money refers to the borrowing and lending of funds for 2-14 days.

d) None of the above

e) All are true

6) Find the incorrect statement on the basis of the given passage.

a) The money market plays an important part in the economy of every nation

b) Commercial paper is an unsecured, short-term debt instrument.

c) Listed companies are the only participants of money market.

d) The functions and activities of organized sector of money market are


coordinated by the RBI.

e) All are correct

7) Choose the word which as same meaning as the word “torn”

a) seamless

Page 216 Follow us: Official Site, Telegram, Facebook, Instagram, Instamojo
b) pristine

c) tattered

d) immaculate

e) intact

8) Choose the word which as same meaning as the word “organized”

a) muddled

b) structured

c) chaotic

d) tangled

e) cluttered

9) Choose the word which as opposite meaning as the word “diversified”

a) swelled

b) split off

c) branched out

d) inflated

e) unvaried

Page 217 Follow us: Official Site, Telegram, Facebook, Instagram, Instamojo
10) Choose the word which as opposite meaning as the word “surplus”

a) superfluous

b) surfeit

c) slippage

d) redundant

e) plethora

Answers:

1) Answer: c)

It is mentioned in the passage that the call money market (CMM) is the market
where overnight (one day) loans can be availed by banks to meet their liquidity
requirements.

2) Answer: b)

According to the passage, money market does not provide the facility of
discounting the bill of exchange as it is not mentioned in the passage.

3) Answer: e)

According to the passage, ‘securities of long term maturities’ is not the feature
of money market as it is given in the passage that money market basically
refers to a section of the financial market where financial instruments with high
liquidity and short-term maturities are traded.

4) Answer: d)

It is given in the passage that Commercial paper is an unsecured, short-term


debt instrument issued by a corporate house for the financing of accounts
payable and inventories, and meeting short-term liabilities.

Page 218 Follow us: Official Site, Telegram, Facebook, Instagram, Instamojo
5) Answer: c)

According to the passage true statement is “Notice Money refers to the


borrowing and lending of funds for 2-14 days.”

6) Answer: c)

It is mentioned in the passage that money market is used by a wide array of


participants, from a company raising money by selling commercial paper into
the market, to an investor purchasing Certificate of Deposits as a safe place to
park money in the short term.

7) Answer: c)

The meaning of “torn” is “ragged /worn out”.

8) Answer: b)

The meaning of “organized” is “structured / prearranged”.

9) Answer: e)

The meaning of “diversified” is “split off/ branched out” and its opposite is
“concentrated /unvaried”.

10) Answer: c)

The meaning of “surplus” is “plethora / superfluous” and its opposite is “deficit/


slippage”.

Directions (11-20): Read the following Passage and answer the questions
given below.

The Supreme Court on Tuesday admitted a plea of a couple to lift the


prohibition on entry of Muslim women into mosques across the country.

Page 219 Follow us: Official Site, Telegram, Facebook, Instagram, Instamojo
“The only reason we may hear you is because of our judgment in Sabarimala
temple,” a Bench of Justices S.A. Bobde and S. Abdul Nazeer remarked orally.
The court issued notice to the government and various bodies, including the
National Commission for Women.

In September last, a Constitution Bench of the court lifted the age-old ban on
women of menstrual age, between 10 and 50 years, entering the famed
Sabarimala temple in Kerala. The decision created uproar. Multiple review
petitions were filed, heard and reserved for judgment. The court had held that
the Sabarimala ban amounted to discrimination and even the practice of
untouchability. Women had equal right to worship in a “public” temple.

The court had also played a key role in facilitating the entry of women into the
sanctum of the Haji Ali dargah in Mumbai.

Justice Bobde asked whether a petition seeking right to equality can be filed
against individuals and non-State actors like people who pray in and manage
mosques. The fundamental right to equality under Article 14 of the Constitution
was available only against the State, he observed.

“Is a mosque a ‘State’? Is a church a ‘State’? Is a temple a ‘State’? We are not


talking about the cement and mortar that make mosques but the people in them.
Can the fundamental right of equality be imposed against another human
being,” he asked.

When the lawyer raised objections, the Bench asked him to read out Article 14.
“You must be referring to a different Article 14 that we do not know about…
The Article 14 starts with the words ‘State shall not deny…’ The relief is
against the State only,” Justice Bobde addressed the lawyer.

At one point, the lawyer said women were not allowed to enter mosques to pray
despite several letters to imams. The petitioner had even sought police help to
enter mosque, the lawyer claimed.

“You do not want someone to enter your house. Can that person then get police
help to enter your house? If persons in mosques don’t want you [women] to
enter, can you agitate right to equality against them? Fundamental right to

Page 220 Follow us: Official Site, Telegram, Facebook, Instagram, Instamojo
equality is only available against the State and not individuals,” Justice Bobde
retorted.

The Pune-based couple, Yasmeen Zuber Ahmad and Zuber Ahmad, told the
court that the ban was illegal, unconstitutional and a violation of their dignity.

“There should not be any gender discrimination and Muslim women should be
allowed to pray in all mosques, cutting across denominations. There is no such
gender discrimination to offer worship in Mecca, the holy city. The faithful,
both men and women, together circle the Kaaba,” their petition said.

Presently, women are allowed to offer prayers at mosques under the Jamaat-e-
Islami and Mujahid denominations. Women are barred from mosques under the
predominant Sunni faction.

Even in mosques where women are allowed, there are separate entrances and
enclosures for worship for men and women.

The petition argued that such a bar on Muslim women was “violative of Article
44 of the Constitution of India, which encourages the State to secure a Uniform
Civil Code for all citizens, by eliminating discrepancies between various
personal laws currently in force in the country”.

Choose the correct option, among the given choices which is most similar to
the word given below, as mentioned in the passage.

11) Plea

a) Petition

b) homage

c) arrangement

d) treaty

e) pledge

Page 221 Follow us: Official Site, Telegram, Facebook, Instagram, Instamojo
Choose the correct option, among the given choices which is most similar to
the word given below, as mentioned in the passage.

12) Uproar

a) panoramic

b) furore

c) futile

d) upbeat

e) hostile

Choose the correct option, among the given choices which is most similar to
the word given below, as mentioned in the passage.

13) Agitate

a) violent

b) nervous

c) Chaotic

d) perturb

e) pandemonium

Choose the correct option, among the given choices which is most Opposite to
the word given below, as mentioned in the passage.

14) barred

a) removed

b) prohibited

Page 222 Follow us: Official Site, Telegram, Facebook, Instagram, Instamojo
c) allowed

d) Cautionary

c) banned

Choose the correct option, among the given choices which is most Opposite to
the word given below, as mentioned in the passage.

15) Violative

a) unlawful

b) disorderly

c) violent

d) virulent

e) supportive

Choose the correct option, among the given choices which is most Opposite to
the word given below, as mentioned in the passage.

16) discrepancies

a) inconsistency

b) irregularity

c) deformity

d) distortion

e) monotony

Page 223 Follow us: Official Site, Telegram, Facebook, Instagram, Instamojo
17) What decision of the constitution bench of the Supreme Court caused a
huge uproar?

a) The court banned Article 44 of the Constitution of India, which encourages


the State to secure a Uniform Civil Code for all citizens.

b) The court lifted the age-old ban on women of menstrual age, between 10 and
50 years, entering the famed Sabarimala temple in Kerala.

c) The bench barred the muslim women from entering the mosque between
evening till morning.

d) The court allowed the Pune-based couple, Yasmeen Zuber Ahmad and Zuber
Ahmad, to enter the mosque.

e) The court lifted the age-old ban on women of menstrual age, between 13 and
55 years, entering the famed Sabarimala temple in Kerala.

18) What did the Pune-based couple Yasmeen Zuber Ahmad and Zuber
Ahmadappeal for?

a) They appealed to the court as they have been getting death threats from other
Sunni factions.

b) The appeal was to restore article 45 of the constitution.

c) They appealed to allow them to avail the benefits of working abroad in the
West.

d) The appeal was to lift the prohibition on Muslim women from wearing a veil
across the country.

e) The appeal was to lift the prohibition on entry of Muslim women into
mosques across the country.

Page 224 Follow us: Official Site, Telegram, Facebook, Instagram, Instamojo
19) Which of the following statements are correct, according to the given
passage?

I. Presently, women are not allowed to offer prayers at mosques under the
Jamaat-e-Islami and Mujahid denominations.

II. The court had held last year, that the Sabarimala ban amounted to
discrimination and even the practice of untouchability.

III. Article 44 of the Constitution of India encourages the State to secure a


Uniform Civil Code for all citizens, by eliminating discrepancies between
various personal laws currently in force in the country.

a) Only I)

b) Only II)

c) I) & II)

d) II) & III)

e) All are correct

20) What can be a suitable Title for the passage?

a) Supreme Court admits plea to lift ban on Muslim women entering mosques.

b) Supreme Court lifts the ban on women from entering the mosques.

c) Uproar on the new ruling of the Supreme Court.

d) Kerala’s Muslims heave a sigh of life on SC’s verdict.

e) Article 44 and all you need to know about Uniform civil code.

Answers :

Page 225 Follow us: Official Site, Telegram, Facebook, Instagram, Instamojo
Directions (11-20):

11) Answer: a)

Plea means petition or an appeal. Hence it is the correct option.

12) Answer: b)

Uproar means an outcry or commotion. So it is closest to option b).

13) Answer: d)

Agitate is related to perturb or concern. Hence it is the correct option.

Directions (14-16):

14) Answer: b)

Option b) is closest antonym of ‘barred’. Other choices a), b) and e) are


synonyms.

15) Answer: e)

Option e) is closest antonym of ‘Violative’. Other choices a) and b) are


synonyms.

16) Answer: e)

All other options are synonyms of the given word. Option e) comes closest
opposite to the word.

17) Answer: b)

the court lifted the age-old ban on women of menstrual age, between 10 and 50
years, entering the famed Sabarimala temple in Kerala.

18) Answer: e)

Page 226 Follow us: Official Site, Telegram, Facebook, Instagram, Instamojo
the appeal was to lift the prohibition on entry of Muslim women into mosques
across the country.

19) Answer: d)

II) & III)

IV) In statement I) the correct statement should be- Women are allowed to offer
prayers at mosques under the Jamaat-e-Islami and Mujahid denominations.

20) Answer: a)

Supreme court admits plea to lift ban on Muslim women entering mosques.

Page 227 Follow us: Official Site, Telegram, Facebook, Instagram, Instamojo
Spotting Error

Directions (1-10): Given below a sentence is divided into four parts, the part
which is mentioned in bold is error free. Find if there is any error in
remaining parts of the sentence and mark that as your answer. If there is No
Error then choose E as your answer.

1) The serial bomb attacks on Easter Sunday, which proclaimed over(A)/ 320
lives in Sri Lanka, were a retaliation for the Christchurch mosque shootings (B)/in
New Zealand on March 15, State Minister of Defence Ruwan
Wijewardene(C)/told Parliament on Tuesday, based on “initial evidence”
available with investigators.(D)

a) C

b) D

c) A

d) B

e) No Error

2) Amid reports of a few stray incidents of attacks on Muslim-owned


property(A)/in the last two days, the sudden, increased(B)/scrutiny of localities
where Muslim families reside have sparked (C)/serious concerns, according to
leaders and members of the community.(D)

a) D

b) C

c) A

d) B

Page 228 Follow us: Official Site, Telegram, Facebook, Instagram, Instamojo
e) No Error

3) Through its cVIGIL mobile application, the Commission has (A)/so far
obtained more than 1.10 lakh(B)/complaints of which 78% have been found
to(C)/be correct and action is being taken against the violators, the EC
said.(D)

a) B

b) A

c) D

d) C

e) No Error

4) The Commission received the transcripts between April 14 and


16,(A)/following which they were being examined legally, in the (B)/overall
content in which the impugned comments were made, said an official.(C)/ The
Congress has so far submitted seven complaints against Mr. Modi seeking action
against him for the alleged violations.(D)

a) D

b) A

c) B

d) C

e) No Error

Page 229 Follow us: Official Site, Telegram, Facebook, Instagram, Instamojo
5) The U.S. on Monday announced that it would be cancelling the waivers from
sanctions(A)/ it had granted eight countries, including India, permitting them
to (B)/ import oil from Iran. Following the revocation(C)/of this waiver, any
country violating the ban would face U.S. sanctions.(D)

a) C

b) D

c) A

d) B

e) No Error

6) However, there was a sharp decline in the Anantnag constituency of Jammu


and Kashmir, (A)/from 39.37% in 2014 to just 12.86%. The five constituencies
of Odisha, where (B)/the third phase of Assembly election was held
concurrently, also had (C)/a lower participation of 64% as against 72.45% in the
last Lok Sabha elections.(D)

a) D

b) C

c) A

d) B

e) No Error

7) The Indian High Commission in Colombo on Tuesday (A)/confirmed that at


least 10 Indians had been killed in Sunday’s serial blasts.(B)/External Affairs

Page 230 Follow us: Official Site, Telegram, Facebook, Instagram, Instamojo
Minister Sushma Swaraj had before said that(C)/officials on the ground were
identifying the victims.(D)

a) D

b) A

c) C

d) B

e) No Error

8) An autorickshaw driver in Guwahati has become popular on social media


for(A)/ giving free rides to senior citizens to and from two polling stations in his
locality.(B)/“As a law-abiding citizen, I felt it was my duty to contribute in my
own way to make people, (C)/especially the elderly, exercise their right to vote,”
Rupam Pathak said.(D)

a) C

b) D

c) A

d) B

e) No Error

9) Ammukkutty Amma and Kali Amma are in their nineties. They do


not (A)/remember how many times they have voted and how many elections
they (B)/have seen. But they voted on Tuesday with the (C)/same zest and
excitement that one sees in first-time voters.(D)

Page 231 Follow us: Official Site, Telegram, Facebook, Instagram, Instamojo
a) C

b) D

c) A

d) B

e) No Error

10) Noted poet and Bollywood lyricist Javed Ahktar on Tuesday


disapproved (A)/of Congress leader Navjot Singh Sidhu urging Muslims to unite
and vote against (B)/the BJP led by Prime Minister Narendra Modi,
asserting (C)/that a mind occupied with one’s religious identity can never
make an informed political choice. (D)

a) B

b) C

c) D

d) A

e) No error

Answers:

1) Answer: c)

Proclaim means to announce something officially or publicly whereas claim


means to state or assert that something is the case without having evidence or
proof. Hence Proclaimed should be replaced with claimed.

2) Answer: d)

Page 232 Follow us: Official Site, Telegram, Facebook, Instagram, Instamojo
Increase is used generally when we describe about quantity means to make
something large whereas heighten means make something more intense(situation
or something) hence here increased should be replaced with heightened.

3) Answer: a)

Obtain means to procure or attain something in any way whereas receive means
to accept something that is sent or offered. Hence obtained should be replaced
with received.

4) Answer: d)

Content is generally used to describe uncountable things and what information


something contains where as context means surroundings, specifications or
circumstances that determine the event of occurrence of something.
Hence content should be replaced with context.

5) Answer: d)

We cannot use permit with adverb particles hence permitting should be replaced
with allowing here.

6) Answer: b)

Concurrent is used for things that occur over a period of time whereas
simultaneous is used at a point of time. Hence concurrently should be replaced
with simultaneously.

7) Answer: c)

Before is reference to time or event but here particular time is not mentioned
hence before should be replaced with earlier.

8) Answer: d)

Give means to transfer the possessions or holding of one’s to others where as


offer means something to present as gesture for good will. Hence giving should
be replaced with offering.

Page 233 Follow us: Official Site, Telegram, Facebook, Instagram, Instamojo
9) Answer: a)

See means to perceive or detect with eyes whereas Witness is to furnish proof of
what you have seen. Hence seen should be replaced with witnessed.

10) Answer: e)

Given sentence is grammatically and contextually correct.

Directions (11-20): Find out the error(s) if any, in each of the following
sentences and choose the correct option.

11) Besides the Khooni Darwaza or Gate (A)/ of Blood, there are other
landmarks (B)/ to remind us of those days. (C)/ Traces of the cannonading
remain on Kashmere Gate’s walls.(D)

a) Only A

b) Only C

c) A and C

d) B and D

e) no error

12) Two small structures opposite to the GPO (A)/ here with cannon-heads
mounted in them, (B)/ proclaim the site of the British Magazine(C)/ blown up
by nine Englishmen. (D)

a) Only A

b) Only C

c) A and B

Page 234 Follow us: Official Site, Telegram, Facebook, Instagram, Instamojo
d) B and D

e) No error

13) The now congested market, where once the (A)/ Red Light area was
located, and which had attracted the city’s nobility (B)/ and literati including
Mirza Ghalib, also retain some traces of those times — old buildings from
whose balconies Delhiwallahs (C)/ saw British troops enter to recapture the
city. In Chandni Chowk’s Parantha Gali, there use to be a milkseller in the
1960s and ’70s.(D)

a) Only A

b) C and D

c) A and C

d) B and D

e) no error

14) She was on a hunger (A)/ strike for 16 long years demanding (B)/ the Indian
government repeal (C)/ the Armed Forces (Special Powers) Act.(D)

a) Only A

b) Only C

c) A and C

d) B and D

e) No error

Page 235 Follow us: Official Site, Telegram, Facebook, Instagram, Instamojo
15) In the Kutcha Mir Ashiq lane of (A)/ Chawri Bazar, members of the family
of Munshi (B)/ Turab Ali fled to the roof, leaving the dal (C)/ burning about
the kitchen. The story is still retold by there descendant, the octogenarian Haji
Mian. (D)

a) Only A

b) Only C

c) A and C

d) Only D

e) no error

16) Led by Lt. Willoughby, they were afraid (A)/ the rebel soldiers would
capture the arms store and wreak havoc on their forces. (B)/ The explosion, they
say, was so loud it could be heard right up to Meerut, (C)/ 65 kilometres away,
where many thought Doomsday had come. (D)

a) Only A

b) Only C

c) A and C

d) B and D

e) No error

17) Irom Sharmila, 47, went in a hunger (A)/ strike in 2000 protested the
Malom massacre, (B)/ where 16 people were shot dead (C)/ in Manipur,
allegedly by the army. (D)

Page 236 Follow us: Official Site, Telegram, Facebook, Instagram, Instamojo
a) Only A

b) Only C

c) A and B

d) B and D

e) no error

18) Iran’s decision to reduce its commitments (A)/ under the 2015 Joint
Comprehensive Plan (B)/ of Action, which seeking to curtail its nuclear
capabilities, (C)/ is more of a warning than a move to break the nuclear
deal. (D)

a) Only A

b) Only C

c) A and C

d) B and D

e) no error

19) While President Hassan Rouhani clinched the (A)/ agreement in 2015
despite opposition (B)/ from hardliners, his promise was that it
would (C)/ help lift sanctions, providing relief to Iran’s economy. (D)

a) Only A

b) Only C

c) A and C

Page 237 Follow us: Official Site, Telegram, Facebook, Instagram, Instamojo
d) B and D

e) No error

20) Iran have been under economic and political pressure (A)/ since President
Donald Trump pulled the U.S. out of the deal a year (B)/ ago. The U.S. has
since amp up its anti-Iran (C)/rhetoric and re-imposed sanctions.(D)

a) Only A

b) Only C

c) A and C

d) B and D

e) no error

Answers :

Direction (11-20) :

11) Answer: e)

The sentence is grammatically and structurally correct.

12) Answer: c)

Use ‘opposite the GPO..’ instead of ‘opposite to the..’ as ‘to’ is redundant here.

Use ‘on’ instead of ‘in’ as the correct form of preposition.

13) Answer: b)

Talking about a single entity- the market, use ‘retains’ instead of ‘retain’.

Page 238 Follow us: Official Site, Telegram, Facebook, Instagram, Instamojo
Simple past tense, so it should be- ‘there used to be’, instead of ‘there use to
be’.

14) Answer: e)

The sentence is grammatically and structurally correct.

15) Answer: d)

A kitchen is a place or an enclosure, so you can use ‘dal burning in the kitchen’,
instead of ‘about’.

Use ‘their’ (pronoun) instead of there.

16) Answer: e)

The sentence is grammatically and structurally correct.

17) Answer: c)

Use ‘on’ instead of ‘in’ as the correct form of preposition.

It should be +ing form of verb- protesting, instead of ‘protested’.

18) Answer: b)

Use simple past form of seek- ‘sought’, instead of seeking.

19) Answer: e)

The sentence is grammatically and structurally correct.

20) Answer: c)

Use ‘has been’ for a single entity here for Iran.

Use simple past form of amp up- amped up.

Page 239 Follow us: Official Site, Telegram, Facebook, Instagram, Instamojo
Fill in the Blanks

Directions (1-10): Fill the blanks and complete the sentences properly.

1) The number of blue whales has ____in the last 5 years.

a) Dwindled

b) Aggravated

c) Destroyed

d) Shed

e) Both a and d

2) Agriculture _____ the most important sector of our economy.

a) have had being

b) have been

c) had been

d) has been

e) None of these

3) A small business has to struggle _____ when its profits decrease)

a) For meeting operation expense

Page 240 Follow us: Official Site, Telegram, Facebook, Instagram, Instamojo
b) To meeting operating expenses

c) To meet operating expenses

d) To meet operate expense

e) None of these

4) _____ accused of stealing the neighbour’s car?

a) Were it they who were

b) Was it they who had

c) Were they who

d) Were it they who

e) Was it they who were

5) _____ enable the company to expand its operations in Europe)

a) Strike this deal that

b) To strike off this deal

c) By striking this deal to

d) Striking this deal will

e) This deal was struck which

Page 241 Follow us: Official Site, Telegram, Facebook, Instagram, Instamojo
6) If thereafter, their subcontracts are for some reason reduced, such firms
_____fixed expenses.

a) Could face potential crippling

b) Could be a potential cripple

c) Could face potentially cripple

d) Could face potentially crippling

e) None of these

7) A pyrotechnic week of geopolitical intrigue has yielded new clarity about


the ________ of the Russia imbroglio, including one insight straight from
Russian President Vladimir Putin.

a) Whole caboodle

b) Worm’s eye view

c) Whys and wherefores

d) Xeros subsidy

e) Soft Option

8) Previous obituary writers have had to ________ and acknowledge that


paperless offices are a bit like fantasising a date with an adored actor—
possible, but improbable

a) Rob the cradle

b) Silver surf

Page 242 Follow us: Official Site, Telegram, Facebook, Instagram, Instamojo
c) Blot their copy-book

d) Eat a humble pie

e) Break every rule in the book

9) Don’t you think you _________ by not showing up at the meeting you
underwent so much of trouble to organize?

a) Made your head spin

b) Burnt Bridges

c) Cut Corner

d) Got a taste of your own medicine

e) Hung in there

10) The detention lasted for three long hours; the teacher made us work
_______.

a) At our wit’s end

b) Catch-22

c) With our noses to the grindstone

d) In dire straits

e) The tip of the iceberg

Answers :

Page 243 Follow us: Official Site, Telegram, Facebook, Instagram, Instamojo
Directions (1-10):

1) Answer: a)

Options ‘c’, d’ and ‘e’ are inappropriate here) And from among options ‘a’ and
‘b’ ‘a’ is the right option because we do not use aggravated with number.

2) Answer: d)

The emboldened part is grammatically incorrect. The auxiliary verb “has” is


followed by “been” to form the present perfect tense to indicate an action that
started in the past and continues till the present. Thus, the correct phrase should
be “has been”. Thus, option D is the correct answer.

3) Answer: c)

The statement is in the simple present form as is evident from the verbs
‘struggle’ and ‘decrease’. Therefore, the other verb ‘met’ needs to changed to
the first form ‘meet’. Also, “expense” should be in the plural form. Thus, C is
the correct answer.

4) Answer: e)

Since the subject “It” represents the singular form, “was” will be used
accordingly.

5) Answer: d)

The sentence is correct with option d) therefore the correct response is option
d)

6) Answer: d)

“potentially” as well as “crippling” is an adverb) Only an adverb can modify


another adverb) Thus, the term “potentially crippling” is correct.

7) Answer: c)

Page 244 Follow us: Official Site, Telegram, Facebook, Instagram, Instamojo
Imbroglio refers to an extremely confused, complicated, or embarrassing
situation. Only option C fits perfectly in the context of the sentence)

Therefore, option C is the correct answer.

8) Answer: d)

The blank is followed by the phrase “and acknowledge) ..” This means that the
obituary writers acknowledged an idea, which they might have denied earlier.

Therefore, option D is the apt answer.

9) Answer: b)

In the given sentence, the concerned person who put a lot of effort to organize a
meeting did not appear in it, so he/ she lost the opportunity. Thus, option B is
the correct answer.

10) Answer: c)

With our noses to the grindstone: It means to work very hard

Directions (11-20): Fill in the blanks with the best suitable options that can
fit both the blanks.

11) Canada doesn’t require that U.S citizens _______ passports to enter the
country, and _______

a) Obtain, Either does Mexico

b) get, Mexico does neither

c) must have, Mexico doesn’t either

d) should have, Neither Mexico does

e) should possess, Mexico doesn’t too

Page 245 Follow us: Official Site, Telegram, Facebook, Instagram, Instamojo
12) I am very ______ in my ways, but my friend has a more _______ attitude
to life

a) fixed, fluid

b) set, flexible

c) particular, moveable

d) strict, changeable

e) open, better

13) His answer was so ________ that I could hardly make any _______ of it
at all.

a) Confused, sense

b) Difficult, meaning

c) Mesmerising, interpretation

d) Meaningful, intelligibility

e) Unknown, thing

14) Although he was under no_______, the shopkeeper replaced the


_______battery free of charge

a) Urgency, destroyed

b) Obligation, defective

c) Guarantee, damaged

Page 246 Follow us: Official Site, Telegram, Facebook, Instagram, Instamojo
d) Insistence, distanced

e) Responsibility, malfunctioning

15) She was extremely _______ and always ______ some mischief or other

a) naughty, up to

b) Funny, up at

c) pleasant, out for

d) pretty, in for

e) lonely, at

16) Courage is a mental ______ and so it gets its strength from spiritual and
______ sources

a) State, intellectual

b) Phenomena, intelligible

c) Idea, intelligent

d) Vibrancy, intrinsic

e) Thought, material

17) Shikha is so _______ that, in a class of about eighty students, you may
have hardly had time to _______ this.

Page 247 Follow us: Official Site, Telegram, Facebook, Instagram, Instamojo
a) Self-critical, see

b) Self-effacing, notice

c) Selfish, witness

d) Self-opinionated, understand

e) Self–obsessed, watch

18) Population increase______ with depletion of natural resources has led to


this ____

a) Joined, misery

b) Coupled, crisis

c) Added, destruction

d) Mixed, state

e) Withheld, situation

19) His _____ is still in its______, so it’s too early to predict

a) Venture, infancy

b) Journey, beginning

c) Life, prime

d) Routine, origin

e) Youth, primary stage

Page 248 Follow us: Official Site, Telegram, Facebook, Instagram, Instamojo
20) The principal _______, ______ the student’s request

a) Complied, with

b) Refused, at

c) Withdrew, with

d) Dismissed, on

e) Back shot, at

Answers:

Directions (11-20):

11) Answer: a)

Obtain, Either does Mexico

Other options do not fit both the blanks

12) Answer: b)

Set, flexible

None of the other options do not fit both the blanks

13) Answer: a)

Confused, sense

Other options do not fit both the blanks

14) Answer: b)

Obligation, defective

Page 249 Follow us: Official Site, Telegram, Facebook, Instagram, Instamojo
Only ‘obligation, defective’ can fit here.

15) Answer: a)

Naughty, up to

Other options do not fit both the blanks. Only ‘naughty, up to’ fits here.

16) Answer: a)

State, intellectual

17) Answer: b)

Self-effacing, notice

18) Answer: b)

Coupled, crisis

19) Answer: a)

Venture, infancy

20) Answer: a)

Complied, with

‘Complied, with’ fits here only.

Page 250 Follow us: Official Site, Telegram, Facebook, Instagram, Instamojo
Cloze Test

Directions(1-10): Given below is a passage, which numbers against blanks


or bold words. You have to read the questions that follow and answer
accordingly.

The campaigns to delay, modify — or even halt — the British


government’s (1)___________of Brexit gained a boost, as Prime Minister
Theresa May said she could offer MPs a choice of ruling out a no-deal exit, or a
time limited-delay to Brexit. The move comes a day after the Labour Party said
it would support the holding of a second (2)___________ if the government
failed to back its alternative vision of Brexit.

The developments come as Britain prepares to leave the EU in just over a


month’s time, so far without a withdrawal (3) agreement amenable to both
sides, and as the Prime Minister prepares to put the terms of leaving the EU to
MPs in a meaningful vote on March 12, just 17 days before Britain is due to
leave. Should her deal not be agreed to on March 12, Ms. May has pledged to
offer MPs votes on the next two days, firstly (on March 13) on ruling out a no-
deal exit, and secondly (on March 14) on requesting an extension to the Article
50(4)__________process. “So the U.K. will only leave without a deal on March
29 if there is explicit consent in this House for that outcome,” Ms. May told a
heated session of the House of Commons on Tuesday. She added that she did
not want Article 50 extended, and would not (5) revoke Article 50. She also
warned against anything but a short delay, noting that if it went beyond June,
Britain would have to take part in European Parliamentary elections (or else
face a ‘sharper cliff edge’ exit a few months down the road. Opposition MPs
warned (6) ________according too much significance to the Prime Minister’s
plan.

“No one should fall for this,” said the leader of the Scottish National Party,
Nicola Sturgeon, insisting her moves were part of “(7) cynical manoeuvring to
try and bully MPs into accepting her bad deal”. Labour also accused the
government of attempting to run down the clock. A no-deal exit was still on the
table, insisted Anna Soubry, a former Conservative MP who left the party last
week to join an independent group of former Conservative and Labour MPs.

Page 251 Follow us: Official Site, Telegram, Facebook, Instagram, Instamojo
They are urging MPs across the political spectrum to back an (8)__________on
Wednesday that would legally require the government to offer MPs the option
of extending the Brexit process beyond March if MPs didn’t agree to Ms. May’s
deal on March 12. The new Labour line on a second referendum marks a
significant shift in the position of the party’s leadership and particularly Jeremy
Corbyn, who have long maintained that the party has a responsibility to deliver
on Brexit and what people voted for in 2016, and highlights the strong pressure
mounted in recent weeks on the party’s top brass, both from within and (9)
_________the shadow Cabinet.

Senior party leaders on Tuesday clarified that Labour would be pushing for a
remain option and a “credible” Brexit plan to be offered to voters in any
referendum. While some of the nine Labour MPs who left the party last week,
cited the party’s Brexit strategy as one of their reasons for doing so, other senior
figures had begun to express (10) ________that unless Labour backed a second
referendum, it would

continue to lose public support as well as more MPs. The decision followed
meetings held by the party in Brussels and Madrid with senior officials.

1) ?

a) Interpretation

b) Augmentation

c) Decision

d) Credibility

e) Both a and d

2) ?

a) Brevity

b) Referendum

Page 252 Follow us: Official Site, Telegram, Facebook, Instagram, Instamojo
c) Holding

d) Chance

e) Corrigendum

3) Find the word from among the options that is nearly opposite to the
word in (3) given in bold.

a) Rapport

b) Accord

c) Dissension

d) Concurrence

e) Entente

4) ?

a) Negotiation

b) Accidental

c) Distortion

d) Concurrence

e) Covenant

5) Find the odd option in relation to the word (5) given in bold in the
passage.

a) Annul

b) Rescind

Page 253 Follow us: Official Site, Telegram, Facebook, Instagram, Instamojo
c) Dismantle

d) Abrogate

e) Corroborate

6) ?

a) Against

b) At

c) For

d) Above

e) Both a and c

7) ?

a) Skeptical

b) Undoubting

c) Jaded

d) Sardonic

e) Derisive

8) ?

a) Believing

b) Adept

c) Amendment

Page 254 Follow us: Official Site, Telegram, Facebook, Instagram, Instamojo
d) Challenges

e) Both a and b

9) ?

a) Inside

b) Outside

c) Besides

d) Beside

e) No error

10) ?

a) Advices

b) Concerns

c) Doldrums

d) Cradled

e) Both a and b

Answers:

Directions (1-10):

1) Answer: a)

only ‘interpretation’ matches the context

2) Answer: b)

Page 255 Follow us: Official Site, Telegram, Facebook, Instagram, Instamojo
The only option ‘referendum’ fits the blank here.

3) Answer: c)

Dissension’ is an antonym of agreement

4) Answer: a)

Only option (a) fits here

5) Answer: e)

Except (e) rest are synonyms

6) Answer: a)

‘Warn against’

7) Answer: b)

Except(b), rest are antonyms, while (b) is a synonym.

8) Answer: c)

Only (c) matches the context

9) Answer: b)

‘Outside’ is grammatically correct in the sentence

10) Answer: b)

‘Concerns’ fits the blank.

‘Finished’ should be replaced with ‘had finished’. We should use perfect


participle before ‘before’ and past indefinite after ‘before’, when two actions are
compared according to the time of their occurrence

Page 256 Follow us: Official Site, Telegram, Facebook, Instagram, Instamojo
Direction (11-20): In a passage given below there are 10 blanks, each
followed by a word given in bold. Even blank has four alternative words
given in options (a), (b), (c) and (d). You have to choose which word will best
suit to the respective blank. Mark (e) as your answer if the word given in the
bold after the blank is your answer i.e. “No change requires

During his lesser-known run for president, which began in 1999, Donald Trump
proposed levying a wealth tax on Americans with more than $10m. He may soon
find himself campaigning on the other side of the issue. Left-wing Democrats
have plenty of ideas for new spending Medicare for all, free college tuition, the
“Green New Deal” that would need funding. Mainly because America is ageing,
but also _ (11) _ [uplift] by Mr Trump’s unfunded tax cuts, the debt-to-gdp ratio
is already expected to nearly double over the next 30 years. If a future Democratic
administration creates new spending programmes while maintaining existing
ones, higher taxes will be necessary. If revenues are to rise, there are good
grounds to look first to the rich. Mr Trump’s tax cuts are just the latest change to
have made life at the top more _ (12) _ [notable]. Between 1990 and 2015 the
real income of the top 1% of households, after taxes and transfers, nearly doubled.
Over the same period middle incomes grew by only about a third and most of that
was thanks to government intervention. Globalisation, technological change
and _ (13) _ [weakened] competition have all helped the rich prosper in recent
decades. Whether or not they are right, the disproportionate gains the rich have
already enjoyed could justify raising new revenues from them. Unfortunately,
the _ (14) _ [bring] new schemes are poorly designed. Ms Warren’s takes aim at
wealth inequality, which has also risen dramatically. It is _ (15) _ [fair] to tax
wealth. But Ms Warren’s levy would be crude, _ (16) _ [alter] and hard to
enforce. A business owner making nominal annual returns of around 5% would
see much of that wiped out, before accounting for existing taxes on capital. That
prospect would _ (17) _ [press] investment and enterprise. Meanwhile,
bureaucrats would repeatedly find themselves having to value billionaires’ art
collections and other illiquid assets. Eight rich countries have scrapped their
wealth taxes since 1990, often amid concerns about their economic and
administrative costs. In 2017 only four levied them. There are better ways to raise
taxes on capital. One is to increase inheritance tax, an inequality-buster that,
though also too easily avoided, is relatively gentle on investment and work
incentives when levied at modest rates. Another is to target economic rents and
windfalls that _ (18) _ [raised] investment returns. Higher property taxes can
efficiently capture some of the astronomical gains that landowners near

Page 257 Follow us: Official Site, Telegram, Facebook, Instagram, Instamojo
successful cities have enjoyed. It is also possible to raise taxes on corporations
that enjoy abnormally high profits without severely _ (19) _ [curb] growth. The
trick is to shield investment spending by letting companies _ (20) _
[subtracted] it from their taxable profit immediately, rather than as their assets
depreciate.

11. ?

a) Undermined

b) Abridged

c) Boosted

d) Curtailed

e) No change required

12. ?

a) Trivial

b) Splendorous

c) Standard

d) Obscure

e) No change required

13. ?

a) Protracting

b) Sustaining

c) Ebbing

Page 258 Follow us: Official Site, Telegram, Facebook, Instagram, Instamojo
d) Perduring

e) No change required

14. ?

a) Proposed

b) Condemned

c) Retreated

d) Tendered

e) No change required

15. ?

a) Verboten

b) Culpable

c) Wicked

d) Legitimate

e) No change required

16. ?

a) Mummifying

b) Distorting

c) Fondling

d) Guarding

Page 259 Follow us: Official Site, Telegram, Facebook, Instagram, Instamojo
e) No change required

17. ?

a) Warp

b) Crook

c) Squash

d) Pervert

e) No change required

18. ?

a) Prune

b) Inflate

c) Abridge

d) Outline

e) No change required

19. ?

a) Inhibiting

b) Expediting

c) Abetting

d) Hyping

e) No change required

Page 260 Follow us: Official Site, Telegram, Facebook, Instagram, Instamojo
20. ?

a) Discounted

b) Distend

c) Bloat

d) Deduct

e) No change required

Answers:

11) Answer: c)

According to the sentence “boosted” will be used as it means “to increase or


improve.”

12) Answer: b)

According to the sentence “splendorous” will be used as it means “magnificent


and impressive in appearance.”

13) Answer: c)

According to the sentence “ebbing” will be used as it means “gradually decrease”

14) Answer: a)

According to the sentence “proposed” will be used as it means “to put forward (a
plan or suggestion)”

15) Answer: d)

According to the sentence “legitimate” will be used as it means “conforming to


the law or to rules”

Page 261 Follow us: Official Site, Telegram, Facebook, Instagram, Instamojo
16) Answer: b)

According to the sentence “distorting” will be used as it means “to falsify”

17) Answer: c)

According to the sentence “squash” will be used as it means “suppress or subdue”

18) Answer: b)

According to the sentence “inflate” will be used as it means “increase something”

19) Answer: a)

According to the sentence “inhibiting” will be used as it means “prevent an action


or process”

20) Answer: d)

According to the sentence “deduct” will be used as it means “take away or out/
to reduce”

Page 262 Follow us: Official Site, Telegram, Facebook, Instagram, Instamojo
Sentence Improvement

Directions (Q1-5): In a given passage. Some sentences begin with a number


(corresponding to the question number) and some words are highlighted in
bold. One of the highlighted words is grammatically incorrect. Choose the
word from the option as your answer. If all the highlighted words are
correct, mark ‘All correct’ as your answer

1.Arundhati Bhattacharya, chairman to the State Bank of India (SBI) on


Wednesday came down heavily on the steel industry for being “non-transparent
in the way they present their data before the banks.”
2.At a time when banks are burdened with NPAs, a large part of which are made
of steel companies, Bhattacharya minced no words in putting across the issue of
a lack of trust for steel companies that has arisen from alleged misrepresentation
of data while seek loans. 3.I don’t want to fudge this and I want to say it explicitly
that there is an element of non-transparency in the way the data comes to us,”
Bhattacharya said addressing an audience at a steel conference organised by
Centrum Broking, that had the top representatives of the steel ministry and the
top management of marquee steel companies like Tata Steel and JSW Steel.”I
know that you all don’t trust bankers but bankers have also stop trusting the
industry,” Bhattacharya said bluntly. 4.The chairman of the largest public sector
bank was speaking on the kind of support that lenders are expected to provide to
the steel industry that is reeling under overcapacity, low demand and burgeoning
debts and sound visibly flustered by the flak that lenders are receiving for
being responsible for the scale of NPAs. 5.However,
Bhattacharya acknowledged that it’s the activity of “one or two rogue players in
the industry that paint the entire industry with the same brush” and
SBI believes that there is adequate upside to the steel industry which will ensure
that it has support anything that makes commercial sense.

1.

a) to

b) heavily

Page 263 Follow us: Official Site, Telegram, Facebook, Instagram, Instamojo
c) being

d) their

e) No correction required

2.

a) burdened

b) putting

c) arisen

d) seek

e) No correction required

3.

a) is

b) comes

c) organized

d) stop

e) No correction required

4.

Page 264 Follow us: Official Site, Telegram, Facebook, Instagram, Instamojo
a) was

b) expected

c) sound

d) responsible

e) No correction required

5.

a) acknowledged

b) players

c) believes

d) has

e) No correction required

Directions (Q6-10): In a given passage. Some sentences begin with a number


(corresponding to the question number) and some words are highlighted in
bold. One of the highlighted words is grammatically incorrect. Choose the
word from the option as your answer. If all the highlighted words are
correct, mark ‘All correct’ as your answer

6.Appraisal season is here. Many salaried persons have already finalized how
they would spend the extra money that would accompany a
salary hiked. 7. However, some financial advisors are reach out to their clients
to increase their investments to achieve their financial goals without any trouble.
“Increment season is the best time for investors to go through their portfolio to
check if the funds are performing as per their goals. 8. More important they can

Page 265 Follow us: Official Site, Telegram, Facebook, Instagram, Instamojo
analyze if they need to invest more towards any long term goal and allocate a
portion of their increment towards it,” says Ankita Tanna Narsey, Founder,
Oaktree Financial Advisors. According to advisors, when you plan for a goal that
is 20 years away, say, your child’s higher education or marriage, you made your
calculations based on certain assumptions like rate of inflation, return on
investment, etc. 9.However, you may realize later that the actual annual inflation
is much higher or the actual return on your investment is lower. Annual salary
hike give investors a good opportunity to increase investments to rectify such
anomalies, say advisors.
10.Increment season also offers an opportunity to plan and add new goals. “In
case of birth of a child, you may wanted to add more goals to your existing ones.
You may do it at this time of the year and utilize the extra salary towards it.

6.

a) is

b) finalized

c) would

d) hiked

e) No correction required

7.

a) reach

b) their

c) portfolio

d) performing

Page 266 Follow us: Official Site, Telegram, Facebook, Instagram, Instamojo
e) No correction required

8.

a) invest

b) allocate

c) made

d) assumptions

e) No correction required

9.

a) may

b) higher

c) give

d) rectify

e) No correction required

10.

a) offers

b) wanted

Page 267 Follow us: Official Site, Telegram, Facebook, Instagram, Instamojo
c) existing

d) towards

e) No correction required

Answers:

Directions (Q1-5):

1). Correct Answer is: a)

Replace ‘to’ with ‘of ’

2). Correct Answer is: d)

Replace ‘seek’ with ‘seeking’

3). Correct Answer is: d)

Replace ‘stop’ with ‘stopped’

4). Correct Answer is: c)

Replace ‘sound’ with ‘sounded’

5). Correct Answer is: d)

Replace ‘has’ with ‘will’

Directions (Q6-10):

6). Correct Answer is: d)

Replace ‘hiked’ with ‘hike’

Page 268 Follow us: Official Site, Telegram, Facebook, Instagram, Instamojo
7). Correct Answer is: a)

Replace ‘reach’ with ‘reaching’

8). Correct Answer is: c)

Replace ‘made’ with ‘make’

9). Correct Answer is: c)

Replace ‘give’ with ‘gives’

10). Correct Answer is: b)

Replace ‘wanted’ with ‘want’

Directions (Q 11-20): Which of the words/phrases (a), (b), (c) and (d) given
below should replace the words/phrases given in bold in the following
sentences to make it meaningful and grammatically correct? If the sentence
is correct as it is and ‘No correction is required’, mark (e) as the answer.

11).Due to last year’s vigorous hurricane season, the National Oceanic and
Atmospheric Administration has advised coastal residents to extend a family
evacuation plan, arrange important documents and valuables and create a
disaster supply kit for use in the car and at the evacuation destination.
a) Dynamic, supported, enlarge, sort out
b) Active, urged, develop, organize
c) Forceful, insisted, improve, manage
d) Vibrant, recommended, increase, handle
e) No improvement required

12).Reports from New York and Paris specifythat this season’s style is an eclectic
mix of fashions from past decades, varying from the empire-waist dresses of
the rested and liberated 1960s to the legwarmers of the excessive
and generous 1980s.
a) Show, changing, settled, lenient

Page 269 Follow us: Official Site, Telegram, Facebook, Instagram, Instamojo
b) Illustrate, differing, respite, soft
c) Prove, diverging, reprieved, tolerant
d) Indicate, ranging, relaxed, indulgent
e) No improvement required

13). While global warming has already made the Hudson River a
seemingly fragile ecosystem, the introduction of invasive species has the
potential to destroy nearly all of the aquatic plants and animals that inhabit the
river.
a) Though, delicate, insidious, demolish
b) Although, flimsy, subtle, wipe out
c) Whereas, feeble, restrained, raze
d) Even if, pathetic, sinister, flatten
e) No improvement required

14).Though we have kept in mind to try and keep most facilities, we would like
to demand you to kindly stand with us any problem that may be caused.
a) Maintain, request, bear, inconvenience
b) Sustain, appeal, abide, difficulty
c) Preserve, ask, tolerate, complexity
d) Retain, call, endure, trouble
e) No improvement required

15).These days urbanized countries have begun looking after for under –
developed countries because it is beneficial for both the parties.
a) Residential, commenced, since, helpful
b) Built-up, instigated, while, profitable
c) Developed, started, as, advantageous
d) Industrialized, initiated, seeing that, useful
e) No improvement required

16).People with persistent headaches are hence as likely to develop bed’s palsy
even as research accounts for other factors which could increase the risk of
conditions like diabetes.
a) Constant, improve, features, boost
b) Relentless, expand, aspects, rise
c) Insistent, recover, causes, enhance
d) Ruthless, look up, parts, add to

Page 270 Follow us: Official Site, Telegram, Facebook, Instagram, Instamojo
e) No improvement required

17).India has declared a long anticipated programme of measures aimed to open


its markets and settle foreign trade negotiators.
a) Pronounced, expected, intended, soothe
b) Stated, predictable, deliberate, pacify
c) Announced, awaited, designed, appease
d) Affirmed, estimated, considered, conciliate
e) No improvement required

18).Transforming ineffective bureaucracies into active, this mission of


customer-driven organizations is challenging under general circumstances.
a) Inefficient, dynamic, task, prevailing
b) Unsuccessful, vibrant, chore, existing
c) Useless, alive, errand, main
d) Vain, vivacious, assignment, popular
e) No improvement required

19). There is a good chance for the members of


our society provide we think ourselves Indians first and everything else later.
a) Luck, area, believe, nothing
b) Option, group, imagine, nobody
c) Prospect, region, assume, no one
d) Fortune, community, consider, anything
e) No improvement required

20).Having been definite to their homes for a week after a leopard was viewed;
the villagers nurtured restless and disturbed.
a) Decided, appeared, fostered, annoyed
b) Confirmed, sighted, grew, frustrated
c) Resolute, stared, reared, aggravated
d) Determined, gazed, cherished, perturbed
e) No improvement required

Explanations:
11). Answer: b)
Active – energetic.
Urged – try earnestly or persistently to persuade (someone) to do something.

Page 271 Follow us: Official Site, Telegram, Facebook, Instagram, Instamojo
Develop – grow or cause to grow and become more mature, advanced, or
elaborate.
Organize – arrange into a structured whole; order.

12). Answer: d)
Indicate – point out; show.
Ranging – differ in size, amount, degree, or nature from something else of the
same general class.
Relaxed – free from tension and anxiety; at ease.
Indulgent – having or indicating a tendency to be overly generous to or lenient
with someone.

13). Answer: e)
While – during the time that; at the same time as.
Fragile – (of an object) easily broken or damaged.
Invasive – (especially of plants or a disease) tending to spread prolifically and
undesirably or harmfully.
Destroy – put an end to the existence of (something) by damaging or attacking it.

14). Answer: a)
Maintain – cause or enable (a condition or state of affairs) to continue.
Request – politely or formally ask for.
Bear – endure (an ordeal or difficulty).
Inconvenience – trouble or difficulty caused to one’s personal requirements or
comfort.

15). Answer: c)
Developed – grow or cause to grow and become more mature, advanced, or
elaborate.
Started – come into being; begin or be reckoned from a particular point in time
or space.
As – used to indicate that something happens during the time when something is
taking place.
Advantageous – involving or creating favorable circumstances that increase the
chances of success or effectiveness; beneficial.

16). Answer: e)
Persistent – continuing to exist or endure over a prolonged period.

Page 272 Follow us: Official Site, Telegram, Facebook, Instagram, Instamojo
Develop – grow or cause to grow and become more mature, advanced, or
elaborate.
Factors – a circumstance, fact, or influence that contributes to a result or outcome.
Increase – an instance of growing or making greater.

17). Answer: c)
Announced – make a public and typically formal declaration about a fact,
occurrence, or intention.
Awaited – wait for (an event).
Designed – decide upon the look and functioning of (a building, garment, or other
object),
Appease – relieve or satisfy (a demand or a feeling).

18). Answer: a)
Inefficient – wasting or failing to make the best use of time or resources.
Dynamic – a force that stimulates change or progress within a system or process.
Task – a piece of work to be done or undertaken.
Prevailing – existing at a particular time; current.

19). Answer: d)
Fortune – chance or luck as an external, arbitrary force affecting human affairs.
Community – a group of people living in the same place or having a particular
characteristic in common.
Consider – think carefully about (something), typically before making a decision.
Anything – used to refer to a thing, no matter what.

20). Answer: b)
Confirmed – firmly established in a particular habit, belief, or way of life and
unlikely to change.
Sighted – manage to see or observe (someone or something); catch an initial
glimpse of.
Grew – become gradually or increasingly.
Frustrated – feeling or expressing distress and annoyance, especially because of
inability to change or achieve something.

Page 273 Follow us: Official Site, Telegram, Facebook, Instagram, Instamojo
Para-Jumbles

Directions (1-5): Rearrange the following sentences into a meaningful and


coherent paragraph and answer the questions below.

A) The scheme, under which one can purchase bonds of various denominations
from a designated bank and deposit them in the accounts of any political party,
had been challenged in the apex court a year ago.

B) The Supreme Court’s interim order asking political parties to disclose, to the
Election Commission in sealed covers, details of the donations they have received
through anonymous electoral bonds is an inadequate and belated response to the
serious concerns raised about the opaque scheme.

C) The order, unfortunately, preserves the status quo, and any effect that the
possible asymmetry in political funding would have on the election process will
stay as it is.

D) The only concession given to those concerned about the dangers of


anonymous political funding is that the names would be available with the EC,
albeit in sealed envelopes, until the court decides if they can be made public.

E) When the matter was taken up last week, it was considered that the time
available was too limited for an in-depth hearing.

1) Which would be the 1st sentence in the rearranged paragraph?

a) E

b) A

c) B

d) C

e) D

Page 274 Follow us: Official Site, Telegram, Facebook, Instagram, Instamojo
2) Which would be the 3rd sentence in the rearranged paragraph?

a) E

b) A

c) B

d) C

e) D

3) Which would be the last sentence in the rearranged paragraph?

a) E

b) A

c) B

d) C

e) D

4) Which would be the 4th sentence in the rearranged paragraph?

a) E

b) A

c) B

d) C

Page 275 Follow us: Official Site, Telegram, Facebook, Instagram, Instamojo
e) D

5) Which would be the 2nd sentence in the rearranged paragraph?

a) E

b) A

c) B

d) C

e) D

Directions (6-10): Rearrange the following sentences into a meaningful and


coherent paragraph and answer the questions below.

A) Dozens were killed. When none of these measures quelled public anger, Mr.
Bashir sacked the Health Minister and the Prime Minister, and promised reforms.

B) But even the fall of Mr. Bashir failed to calm the streets as protesters wanted
“a revolution”.

C) When protests broke out in Atbara in northeastern Sudan over rising prices of
bread in mid-December, not many thought it would snowball into a nationwide
agitation, shaking the foundations of the junta.

D) But the protests, led by the Sudanese Professionals Association, a new group,
grew in strength.

E) President Omar al-Bashir, who captured power through a bloodless coup in


1989, first called the protesters “rats” and then declared a state of emergency.

Page 276 Follow us: Official Site, Telegram, Facebook, Instagram, Instamojo
F) As protests reached the army headquarters, the military high command stepped
in, deposing Mr. Bashir on April 11 and announcing a transitional government
led by the military council.

G) Over three decades, Mr. Bashir and his military clique had used several tactics,
from aligning with Islamists and banning political parties to suppressing dissent
and unleashing paramilitaries against defiant regions, to stay in power.

6) Which would be the last sentence in the rearranged paragraph?

a) E

b) G

c) B

d) C

e) D

7) Which would be the 2nd sentence in the rearranged paragraph?

a) E

b) A

c) B

d) C

e) F

8) Which would be the 1st sentence in the rearranged paragraph?

Page 277 Follow us: Official Site, Telegram, Facebook, Instagram, Instamojo
a) E

b) A

c) G

d) C

e) F

9) Which would be the 4th sentence in the rearranged paragraph?

a) E

b) A

c) G

d) D

e) F

10) Which would be the 5th sentence in the rearranged paragraph?

a) E

b) A

c) G

d) C

e) F

Page 278 Follow us: Official Site, Telegram, Facebook, Instagram, Instamojo
Answers :

Directions (1-5):

The correct sequence would be: B A E C D

B) The Supreme Court’s interim order asking political parties to disclose, to the
Election Commission in sealed covers, details of the donations they have received
through anonymous electoral bonds is an inadequate and belated response to the
serious concerns raised about the opaque scheme.

A) The scheme, under which one can purchase bonds of various denominations
from a designated bank and deposit them in the accounts of any political party,
had been challenged in the apex court a year ago.

E) When the matter was taken up last week, it was considered that the time
available was too limited for an in-depth hearing.

C) The order, unfortunately, preserves the status quo, and any effect that the
possible asymmetry in political funding would have on the election process will
stay as it is.

D) The only concession given to those concerned about the dangers of


anonymous political funding is that the names would be available with the EC,
albeit in sealed envelopes, until the court decides if they can be made public.

1) Answer: C

2) Answer: A

3) Answer: E

4) Answer: D

5) Answer: A

Directions (6-10):

Page 279 Follow us: Official Site, Telegram, Facebook, Instagram, Instamojo
The correct sequence would be: C E A D F B G

C) When protests broke out in Atbara in northeastern Sudan over rising prices of
bread in mid-December, not many thought it would snowball into a nationwide
agitation, shaking the foundations of the junta.

E) President Omar al-Bashir, who captured power through a bloodless coup in


1989, first called the protesters “rats” and then declared a state of emergency.

A) Dozens were killed. When none of these measures quelled public anger, Mr.
Bashir sacked the Health Minister and the Prime Minister, and promised reforms.

D)But the protests, led by the Sudanese Professionals Association, a new group,
grew in strength.

F) As protests reached the army headquarters, the military high command stepped
in, deposing Mr. Bashir on April 11 and announcing a transitional government
led by the military council.

B) But even the fall of Mr. Bashir failed to calm the streets as protesters wanted
“a revolution”.

G) Over three decades, Mr. Bashir and his military clique had used several tactics,
from aligning with Islamists and banning political parties to suppressing dissent
and unleashing paramilitaries against defiant regions, to stay in power.

6) Answer: B

7) Answer: A

8) Answer: D

9) Answer: D

10) Answer: E

Page 280 Follow us: Official Site, Telegram, Facebook, Instagram, Instamojo
Direction (11-15): The passage has been divided into several sentences,
denoted by A), B), C), D), E), F) and G). Read the sentence and arrange them
in a manner that the sentences make a meaningful paragraph.

A) India has five different GST rates 0, 5%, 12%, 18%, and 28%, according to
the World Bank India Development Update 2018, of the 115 countries with GST
regime, 40 countries use a single rate and 28 countries use two rates, India keeps
rather poor company with Italy, Ghana, Pakistan, and Luxemburg countries with
four or more rates.

B) As the world around us grows in complexity, there is a tendency for


governments to think we need more detailed rules and systems to regulate the
dynamic nature of our economy and society, counter-intuitively, the greater the
complexity, the more we need simple rules to guide us through the maze of
choices.

C) If one goes to the GST council website, there are a few hundred notifications,
more than one for each day in existence and that is just the Central government
notifications, each state has its own set of notifications.

D) As we approach the first anniversary of the goods and services tax reform, it
is a good time to take stock of the problems in the system, introducing GST in
India was no small feat but it has become increasingly clear that the system is too
complicated and has led to a lot of confusion for taxpayers.

E) There is a constant process of raising disputes and questions, and the


government trying to clarify classifications and solve problems, then there are the
legal challenges in courts and this is only about the current stock of goods and
services, it does not account for new products and services that will enter the
market in the future.

F) A single GST for all goods and services needs no classification system. Every
individual and firm simply pays the same rate, and the government and the legal
system can use valuable time and resources resolving other problems.

G) The finance ministry and the GST council are asking the wrong question, the
point is not how best to classify each good, but to realize the impossibility of

Page 281 Follow us: Official Site, Telegram, Facebook, Instagram, Instamojo
successfully classifying each good, therefore it is best to do away with the need
to classify each good.

11) Which sentence should be sixth in the paragraph?

a) D

b) C

c) A

d) F

e) B

12) Which sentence should be third in the paragraph?

a) B

b) D

c) E

d) A

e) C

13) Which sentence should be fifth in the paragraph?

a) E

b) G

c) C

d) D

e) F

Page 282 Follow us: Official Site, Telegram, Facebook, Instagram, Instamojo
14) Which sentence should be last in the paragraph?

a) E

b) D

c) B

d) A

e) F

15) Which sentence should be fourth in the paragraph?

a) E

b) F

c) D

d) C

e) B

Direction (16-20): The passage has been divided into several sentences,
denoted by A), B), C), D), E), F) and G). Read the sentence and arrange them
in a manner that the sentences make a meaningful paragraph

A) The GST is leading to better tax compliance, the number of unique


registrations has now crossed the ten million mark, which is higher than entities
registered in the pre-GST period, the increasing number of taxpayers and better
compliance should help raise higher revenue in the medium to long run.

B) The GST system is creating a vast repository of data that could be useful in
policymaking, for example, it is now possible to know the state-wise distribution
of international exports and this information can be used to fine tune policies in
particular states to boost exports.

Page 283 Follow us: Official Site, Telegram, Facebook, Instagram, Instamojo
C) Economists at the Union finance ministry studied GST data in detail and
presented some interesting facts in this year’s Economic Survey, the Survey
showed that India’s formal non-farm payroll is much higher than is commonly
believed.

D) It is important for India to simplify the tax structure, the first target should be
to move to at least a three-rate structure a lower rate for essential goods, a
relatively high rate for luxury goods and a standard rate for the majority of goods
and services

E) Further, the way the GST council has evolved is a notable achievement, it
shows the way complex issues can be addressed through cooperation between the
Union and state governments, while the council has a specific purpose perhaps
the idea can be used to address policy issues in other areas.

F) It has been a year since the landmark goods and services tax, which converted
India into a single market was implemented, although India opted for an
extremely complex GST structure, its implementation was still a big
breakthrough as problems can always be addressed with experience.

G) However, despite visible benefits, as has often been argued in these pages, the
GST structure is far from optimum, the latest India Development Update of the
World Bank, for example noted that the 28% rate applicable on a set of goods is
the second highest among the 115 countries.

16) Which sentence should be fourth in the paragraph?

a) G

b) A

c) F

d) B

e) C

Page 284 Follow us: Official Site, Telegram, Facebook, Instagram, Instamojo
17) Which sentence should be seventh in the paragraph?

a) F

b) D

c) C

d) E

e) B

18) Which sentence should be third in the paragraph?

a) E

b) G

c) B

d) C

e) A

19) Which sentence should be fifth in the paragraph?

a) B

b) A

c) E

d) F

Page 285 Follow us: Official Site, Telegram, Facebook, Instagram, Instamojo
e) D

20) Which sentence should be sixth in the paragraph?

a) G

b) D

c) E

d) C

e) F

Answers:

Direction (11-15):

The correct order of sentence after rearrangement is DACEGFB

D) As we approach the first anniversary of the goods and services tax reform, it
is a good time to take stock of the problems in the system, introducing GST in
India was no small feat but it has become increasingly clear that the system is too
complicated and has led to a lot of confusion for taxpayers.

A) India has five different GST rates 0, 5%, 12%, 18%, and 28%, according to
the World Bank India Development Update 2018, of the 115 countries with GST
regime, 40 countries use a single rate and 28 countries use two rates, India keeps
rather poor company with Italy, Ghana, Pakistan, and Luxemburg countries with
four or more rates.

C) If one goes to the GST council website, there are a few hundred notifications,
more than one for each day in existence and that is just the Central government
notifications, each state has its own set of notifications.

Page 286 Follow us: Official Site, Telegram, Facebook, Instagram, Instamojo
E) There is a constant process of raising disputes and questions, and the
government trying to clarify classifications and solve problems, then there are the
legal challenges in courts and this is only about the current stock of goods and
services, it does not account for new products and services that will enter the
market in the future.

G) The finance ministry and the GST council are asking the wrong question, the
point is not how best to classify each good, but to realize the impossibility of
successfully classifying each good, therefore it is best to do away with the need
to classify each good.

F) A single GST for all goods and services needs no classification system. Every
individual and firm simply pays the same rate, and the government and the legal
system can use valuable time and resources resolving other problems.

B) As the world around us grows in complexity, there is a tendency for


governments to think we need more detailed rules and systems to regulate the
dynamic nature of our economy and society, counter-intuitively, the greater the
complexity, the more we need simple rules to guide us through the maze of
choices.

The passage is about the goods and services tax. According to the passage it has
been one year since goods and services tax has been implemented so its time to
look at the big picture. Introduction of GST in India was no small feat and also
the system is too complex and has led to a lot doubts for taxpayers. Some blame
the government for not doing a good enough job of detailing and specifying
various classifications. There are nonstop dispute and that are raising while the
government is trying to clarify classifications and solve problems. The major
issue relies on the classification of the goods and services tax. So the government
must ascertain that it is impossible to successfully divide each good, therefore it
is best to do away with the need to arrange each good.

11) Answer: d)

12) Answer: e)

13) Answer: b)

Page 287 Follow us: Official Site, Telegram, Facebook, Instagram, Instamojo
14) Answer: c)

15) Answer: a)

Direction (16-20):

The correct order of sentence after rearrangement is FCABEGD

F) It has been a year since the landmark goods and services tax, which converted
India into a single market was implemented, although India opted for an
extremely complex GST structure, its implementation was still a big
breakthrough as problems can always be addressed with experience.

C) Economists at the Union finance ministry studied GST data in detail and
presented some interesting facts in this year’s Economic Survey, the Survey
showed that India’s formal non-farm payroll is much higher than is commonly
believed.

A) The GST is leading to better tax compliance, the number of unique


registrations has now crossed the ten million mark, which is higher than entities
registered in the pre-GST period, the increasing number of taxpayers and better
compliance should help raise higher revenue in the medium to long run.

B) The GST system is creating a vast repository of data that could be useful in
policymaking, for example, it is now possible to know the state-wise distribution
of international exports and this information can be used to fine tune policies in
particular states to boost exports.

E) Further, the way the GST council has evolved is a notable achievement, it
shows the way complex issues can be addressed through cooperation between the
Union and state governments, while the council has a specific purpose perhaps
the idea can be used to address policy issues in other areas.

G) However, despite visible benefits, as has often been argued in these pages, the
GST structure is far from optimum, the latest India Development Update of the
World Bank, for example noted that the 28% rate applicable on a set of goods is
the second highest among the 115 countries.

Page 288 Follow us: Official Site, Telegram, Facebook, Instagram, Instamojo
D) It is important for India to simplify the tax structure, the first target should be
to move to at least a three-rate structure a lower rate for essential goods, a
relatively high rate for luxury goods and a standard rate for the majority of goods
and services

The passage is about the goods and services tax. Now that India has had the
experience of running the GST system for a year, policymakers should focus on
building on its successes and addressing its drawbacks to achieve its full potential.
The registrations of goods and services tax has now crossed the ten million mark
and it would further result in raising of higher revenue in the medium to long run.
Today the GST system has become a vast repository of data. Apart from this the
World Bank noted that the 28% rate is the second highest among the 115
countries. Even after the implementation is done, the council will need to
continuously work on simplifying the structure to enable higher tax collection
and economic growth. Also Policymakers should build on its success and address
the drawbacks to ensure it achieves its full potential

16) Answer: d)

17) Answer: b)

18) Answer: e)

19) Answer: c)

20) Answer: a)

Page 289 Follow us: Official Site, Telegram, Facebook, Instagram, Instamojo
Join Our Social Network, Links Below

 Daily Visit Our Official Website-


www.dreambiginstitution.com
 Join us on Instagram-
https://www.instagram.com/dream_big_institution
 Follow us in Facebook-
https://www.facebook.com/dreamBigInstitution
 Join our Telegram Channel-
https://t.me/DreamBiginstitution
 Our Online Premium Book Store
https://www.instamojo.com/dreambiginstitution

Page 290 Follow us: Official Site, Telegram, Facebook, Instagram, Instamojo
Page 291 Follow us: Official Site, Telegram, Facebook, Instagram, Instamojo

You might also like